Sei sulla pagina 1di 162

from p 133 - child with fever

line 7860

_______________________________________
PC:

---
1o -
---
ddx:

V -
I -
N -
D -
I -
C -
A -
T -
E -
M -
S -
---
OE:

---
ix:

---
mx:

_______________________________________

Headache / 87
migraine
cluster ha
temporal arteritis
sinusitis
tension ha
sah
meningitis
pseudotumor cerebri
trigeminal neuralgia
Confusion/Memory Loss / 89
multi-infarct/vascular dementia
alzheimer
NPH
CJD
hypoglycemia
subdural hematoma
Loss of Vision / 91
retinal artery occlusion
Depressed Mood / 92
bereavement
MDD
bipolar 1
Psychosis / 93
schizophrenia
substance psychosis
schizoaffective disorder
Dizziness / 94
menieres
dehydration related orthostatic hypotension
drug induced orthostatic hypotension
benign positional vertigo
vestibular neuronitis
labyrinthitis
Loss of Consciousness / 95
general tonic clonic
drug induced orthostatic hypotention/syncope
cardiac arrhythmia
Numbness/Weakness / 96
TIA
stroke
guillain barre
MS
diabetic peripheral neuropathy
myasthenia gravis
todds paralysis
Fatigue and Sleepiness / 98
MDD
PTSD
colon cancer
hypothryoidism
OSA
DM
shift work sleep disorder
Night Sweats / 100
night sweats
hyperthyroid
Insomnia / 100
stress induced insomnia
OSA
depression induced insomnia
Sore Throat / 101
infectious mononucleosis
HIV, retroviral syndrome
pharyngitis
Cough/Shortness of Breath / 102
astma
COPD
penumonia
atypical pneumonia
lung cancer
COPD exacerbation
TB
CHF
pulmonary edema
Chest Pain / 105
MI
sickle cell
GERD
stable angina
pericarditis
costochondritis
pulonary embolism
aortic dissection
Helico pylori
Palpitations / 107
hypoglycemia
panic attack
social phobia
MVP
Weight Loss / 108
hyperthyroid
Weight Gain / 109
smoking cessation
insulinoma
Dysphagia / 110
esophageal cancer
plummer vinson syndrome
achalasia
esophagitis
Neck Mass / 111
hodgkins / non-hodgkins
Nausea/Vomiting / 111
pregnancy
Abdominal Pain / 112
nephrolithiasis
pancreatic cancer
acture pancreatitis
acute cholecystitis
ascending cholangiitis
acute hepatitis
peptic ulcer disease
perforated peptic ulcer
splenic rupture
intestinal obstruction
mesenteric ischemia/infarction
ovarian torsion
diverticulitis
acute appendicitis
irritable bowel syndrom
pelvic inflammatory disease
Constipation/Diarrhea / 116
colorectal cancer
low fiber diet
irritable bowel syndrome
gastroenteritis
pseudomembranous colitis
travelers diarrhoea
lactose intolerance
crohns disease
ulcerative colitis
Upper GI Bleeding / 118
bleeding peptic ulcer
gastritis
Blood in Stool / 119
colorectal cancer
ulcerative colitis
diverticulosis
Hematuria / 119
bladder cancer
Polycystic kidney diseaes
renal cell carcinoma
Other Urinary Symptoms / 120
benign prostatic hypertrophy BPH
prostate cancer
urethritis
acute pyelonephritis
stress incontinence
urge incontinence
Erectile Dysfunction / 122
drug related erectile dysfunction
Amenorrhea / 123
pregnancy
polycystic ovary syndrome
amenorrhea secondary to prolactinoma
menopause
sheehans syndrome
anorexia nervosa
anxiety induced amenorrhea
Vaginal Bleeding / 124
dysfunctional uterine bleeding
endometrial cancer
cervical cancer
spontaneous abortion
ectopic cancer
Vaginal Discharge / 126
bacterial vaginosis
vaginitis - candida
vaginitis - trichomonal
Dyspareunia / 126
atrophic vaginitis
endometriosis
Abuse / 127
domestic violence
rape
Joint/Limb Pain / 128
domestic vilence
carpal tunnel
systemic lupus erythematosus
rheumatoid arthritis
psoriatic arthritis
hip fracture
hip dislocation
osteoarthritis
septic arthritis
stress fracture
plantar fasciitis
tennis elbow - lateral epicondylitis
disseminated gonorrhea
peripheral vascular disease - intermittent claudication
DVT
angina/MIA
shoulder dislocation
rhabdomyolysis - statins
Low Back Pain / 132
disk herniation
lumbar muscle strain
lumbar spinal stenosis
malingering
Child with Fever / 133
neonatal sepsis
acute otitis media
measles / viral exanthem
gastroenteritis
Child with GI Symptoms / 134
pyloric stenosis
hisrchsprungs
intussusception
somatoform disorder
colic
Child with Red Eye / 136
bacterial conjunctivitis
Child with Short Stature / 136
constitutional short stature
Behavioral Problems in Childhood / 137
ADHD
adjustment disorder
substance abuse
age appropriate
ankle pain
back pain
sore throat
car accident
left arm weaknes
nosebleed / epistaxis
sudden abdominal pain and syncope
vaginal bleed in 40 yo
fever and weight loss
adolescent weight loss
bed wetting 7 yo
RUQ abd pain
mental status change
abdo pain and fatigue
child w V + D
fatigue
obesity
fever in 70yo
COPD
bizarre behaviour
diarrhea
chest pain
syncope
syncope
ear ache
diabetic complications
diabetic uncontrolled
dka
_______________________________________
VINDICATEMS - vascular, idiopathic/infectious/inflamm, neoplastic, degenerative,
intoxication, congenital/csf, autoimmune/allergic/anatomic, trauma + toxic,
endocrine/enviro, metablic/muscular, seizure/(p)SYCH

_______________________________________
PC:

---
1o -
---
ddx:

V -
I -
N -
D -
I -
C -
A -
T -
E -
M -
S -
---
OE:

---
ix:

---
mx:

_______________________________________

_______________________________________
PC:

Throbbing head pain


Unilateral temporal
hours
flashes
weakness + numb
N/V
---
1o MIGRAINE
---
V - migrain, tension, cluster, vasculitis
I
N
D
I
C - pseudotumor cerebri
A
T
E
M - tension
S - seizure-partial
---
BP, temp, Kernig/brudzinksi, msk H+N
papilledema, retinal hemorrhage, temporal artery, sinus palpation, TMJ
neuro / deficitis
*papilledema,LOC,fever,meningism,focal neuro, trauma
---
CBC - metabolic
ESR - inflamm
CT-B - bleed
MRI-B - masses
LP-CSF - infectious
---
Acute: stop trigger, ASA, NSAID, triptan 5HT(+), ergotamine 5HT(+)
Prophy if >3/mo or refractory: avoid trigger, TCA,
antiepileptic/topiramate/gabapentin, propranolol B(-), CCB/verapamil, stop OCP
(risk of stroke)

_______________________________________
PC:
Rt temporal HA
rhinorhhea + redness
HA clustered in time
---
1o CLUSTER HA
---
ddx:

V - migrain
I -
N - neoplastic
D
I
C - pseudotumor cerebri
A
T
E
M
S
---
OE:
retro-orbital
10min-2hr
daily attacks for grouped periods of time with no symptoms in between clusteres
constant, aching, stabbing
wakes from sleep
trigger: light/etoh
better with walking around
red water eye
nasal congestion, rhinorheea
unilateral horner
---
ix:
CBC
CT-B - rule out bleed
MRI-N - rule out neoplastic
LP - rule out infection
ESR
---
mx:
acute: O2, nasal sumatriptan
prophy: avoid triggers, CCB/verapamil, lithium, methylsergide, prednisolon

_______________________________________
_______________________________________
PC:
intermitten Rt temporal HA
fever
blurred vision in Rt eye
pain in jaw when chewing
---
1o - GIANT CELL arteritis, temporal arteritis
---
ddx:

V - migraine, cluster, tesion, carotid artery dissection


I - meningitis
N -neoplastic
D -
I -
C - pseudotumor cerebri
A -
T - caritid dissection
E -
M - TMJ HA
S -
---
OE:
HA worse when eating
visual acuity/fields
---
ix:
ESR*
temporal artery biopsy**
CBC
Carotid doppler USS
MRI-B - neoplastic
LP - infectious
---
mx:
prednisolone before biopsy to save sight (neurological emergency)
ophtal followup

_______________________________________
_______________________________________
PC:
frontal HA
fever
nasal d/c
sinus pain on palpation
hx allergy

---
1o - sinusitis
---
ddx:

V - igraine, tension,
I - meningitis
N - neoplastic
D -
I -
C -
A -
T -
E -
M -
S -
---
OE:
maxillary toothache
headache
fatigue
cough
ear pain fullness
nasal d/c
halitosis
better with decongestants
abnormal transillumination
purulent excretion
nasal congestion
facial pain/pressure
decre sense of smell
viral < 7d
bacterial > 7d

viral*****
(rhinovirus, inflenza, parainfluenza)

bacterial
S. pneumoniae
H. influenzae
M. catarrhalis
S. aureus
---
ix:
clinical diagnosis
otoscope*
CBC
XR/CT sinus
LP CSF
---
mx:
1o NSAID, NS rinse, decongestants

NOT ANTIHISTAMINES
bacterial:
1o amoxicillin 500 mg PO tid x 5-10 d
2O nasal corticosteroids
3o ENT referralif - AMS, HA, toxic, swelling orbid/visiual, focal neuro,
meningismus, intracranial
_______________________________________
_______________________________________
PC:
bilateral squeezing HA
recurrent x 4/wk
esp end of work day
stress
recently decr consumption caffeine

---
1o - tension headache
---
ddx:

V - migraine, cluster, htn


I -
N - neoplastic
D -
I - withdrawal caffeine
C - pseudotumor cerebri
A -
T -
E -
M -
S - depression
---
OE:
bilateral frontal/nuchal-occipital
minutes - days
band like constant
trigger - depression, anxiety, noise, hunger, sleep deprivation
better with rest
no N/V or photophobia
---
ix:
CBC
EUC
ESR
CT-B
LP-CSF

---
mx:
1o - REST - psych, lifestyle relaxation, massage, avoid triggers
2 - tca, nsaids, ssri

_______________________________________
_______________________________________
PC:
sudden
severe HA
vomiting
confusion
left hemiplegia
nuchal rigidity

---
1o - SAH
---
ddx:

V - migraine, vertebral artery dissection, acute htn


I - meningitis
N - neoplastic
D -
I -
C -
A -
T - SAH, bleed
E -
M -
S -
---
OE:
---
ix:
1o - non-contrast CT**
2o - LP - xanthochromia + RBC + protein
3o - angiography, CTA

---
mx:
Admit ICU
O2/ventilation
bed elevated 30o
BP control sBP 120-150
continuous ECG
catheterisation
IV NS + 20 mmol K at 120 ml/hr
nimodipine 60 mg for vasospams
leveracetam - antiepileptic
corticosteroids - hydrocephalus
mild sedation

monitor follow up bloods for DI, hyponatremia, pulmonary edema


_______________________________________
_______________________________________
PC:
hi fever
severe HA
confusion
photophobia
muchal regidity

---
1o - meningitis
---
ddx:

V - migraine
I - sinusitis, meningitis, abscess
N -
D -
I -
C -
A -
T - SAH
E -
M - SIADH
S -
---
OE:
meningismus - fever, nausea/vomiting, fever, HA, neck stiffness, confusion,
photophobia, AMS, seizure, focal neuro, papilledema
***petechial rash LL = meningococcal
---
ix:
[CT head if - focal neuro, seizure, papilledema, AMS/coma/immunocompromised]

If no CT = septic wkup then ABX empiric/dexamethasone(dont wait for results) then


CT

If yes CT = LP/septic wkup then ABX empiric/dexamethasone


Septic workup =
BC, UC, CSF-culture => acid fast bacilli, fungal, cryptococcal antigen
CBC + differential
UEC - SIADH
PCR for viral
LP - opening pressure, cell count + differential, proten, gram stain, bacterial C+S
MRI/CT
EEG

if crytococcus suspected:
1o antigen titer
2o india ink stain
---
mx:
ACUTE =
< 1mo = ampicillin, cefotaxime, gentamicin IV
> 1mo = dexamethasone + vancomycin + ceftriaxone (+ ampicillin if listeria risk ie
>50 or <3mo or immunocomp ie HIV/steroid/haem cancer/pregnancy)
ADD acyclovir if viral suspected

Cryptococcus (= HIV pt):


induction = 2 wk flucytosine + amphotericin B
maintenance = HAART + fluconazole until CD4 > 100 for 3 mo
lifelong = HAART

TB meningtitis:
RIPE + dexamethasone for 9mo-1yr

IMMUNISE =
N meningitidis, Pneumovax, HiB (+ rifampin for contacts)
_______________________________________
_______________________________________
PC:
obese F
pulsatile HA
vomiting
blurred vision x 3 wk
taking OCP

---
1o - pesudotumor cerebri
---
ddx:

V - tension HA, migraine, cluster, stroke/CVA/VTE, venous sinus thrombosis


I - meningitis
N - neoplastic
D -
I -
C - pseudotumor cerebri
A -
T -
E -
M -
S -
---
OE:
[no LOC, diplopia]
decr vis acuity
papilled3ema,
---
ix:
1o - LP + opening pressure (also therapeutic)
CT-B - normal, rule out bleeed
LP - rule out infection
MRI - rule out venous sinus thrombosis
---
mx:
-rule out ddx (esp venus sinus thromb)
-discontinue steroids, tetracycline, nalixidic acid, retinoic acid
-acetazolimide + thiazide/furuseide
-LP shunt
-optic nerve sheath decompression
-2 yr monitor for neoplasm with MRI
-f/u opthal visual loss
_______________________________________
_______________________________________
PC:
Rt cheek pain 1 mo
Electric + stabbing for a few seconds/minutes

---
1o - trigeminal neuralgia
---
ddx:

V - tension, migraine, cluster


I -
N - neoplastic - cerebello angle tumor
D -
I -
C -
A - multiple sclerosis
T -
E -
M - TMJ
S - trigeminal neuralgia
---
OE:
electric pain - sec-min
neuro exam - NORMAL
triggers - touch face/shaving, talking, cold winde
---
ix:
clinical dx
CBC
ESR - temporal arteritis
MIR- B - rule out structural lesion, MS, vascular abn
---
mx:
1o - carbamazepine
2o - baclofen
_______________________________________
_______________________________________
PC:
elderly
7 yr hx progressive confusion + forgetful + clumsy
hx - htn, DM, CVA/strokes + left hemiparesis
declining cognitive

---
1o - vascular dementia (multi infarct)
---
ddx:

V - vascular/multi infarct dementia, CNS vasculitis


I - neurosyphilis
N - neoplastic
D - alzheimer, B12, hypothryoid
I -
C - NPH
A -
T -
E -
M -
S - depression

VITAMIN D VEST
vitamin def (B12, folate, thiamine)
intracranial tumor
trauma - head injury
anoxia
metabolic /diabetes
infectious - postencephalitis, HIV
NPH
degen - alzehimers, huntingtons, CJD
vascular - multi infarct
endocrne - hypothryoid
space occupying lesion - chronic subdural hematoma
toxic - alcohol, polypharmacy

DEMENTIASS
-degen = parkinsons, huntington, alzheimers
-endocrine = thryoid, pth, pituitary adrenal
-metabolic = etoh, uec. B12, glucose, hepatic, renal, wilsons
-exogenous = heavy metals, co, drugs
-neoplasia
-trauma - subural hematoma
-infection = meningitis, encephalitis, endocarditis, syphilits, HIV, prion dz, lyme
dz
-affective disorder - pseudodementia
-stroke/structural = vascular dementia, vasculitis, NPH
---
OE:
vascular dementia:
stepwise cognitive decline
focal neuro

huntington:
chorea
behavioural

alzheimers:
memory, aphasia, apraxia, agnosis
dementia with lewybodies:
vis hallucinations, parkinsonism, fluctuating cognition

frontotemporal dementia (pick dz):


disihibition, perseveration, decr social awareness, mental rigidity

parkinsons:
TRAP = tremor at rest, rigidity cogwheel, akinesia, postural instability
mask fascies
shuffling gait
response to levodopa
unilateral onset

MMSE
---
ix:

MMSE

CBC + smear - rule out B12/alcohol


BSL - hypoglcyemia
TSH - hypothryoid
B12 levels
LFTs - encephalitis
lipids
LP - CSF 14-4-4 for CJD; NPH opening pressure
STDs - HIV pcr, midstream urine

image if: focal neuro, seizure, gait abn, acute sx, rapid sx.

CT/MRI, spect
ANA, ANCA - rule out vasculitis
MRI angiography

fhx suggestive of huntington:


1o clinical
2o genetic dna- CAG repeat
3o MRI (atrophy caudate nucleus, enlarged ventricles)

---
mx:

alzheimers:
1o - AChEI - donezepil, rivastigmine, galantamine
2o - NMDA(-) = memantine
3o - ssri
4o - trazodone - sleep disturbance
5o - family support + day care facilities

huntington:
1o clozapine
2o only screen family AFTER pt/dr discussion on benefit vs downside

parkinsons:
1o carbidopa (decr DA peripheral degrad) + levodopa DA+ (PO or pump) = SE = on-off
sx/dyskinesia
2o COMT inhib (tolcapone)
3o pamiprexol (DA+)
4o selegiline
5o DBS - anterodorsal subthalamis nucleus

dementia with lewy bodies:


1o AChE-I (donezepil)

Frontotemporal dementia:

social:
failure to cope
limit polypharmacy
fitness to drive - driving limitation - advice
caregiver education + stress
advance directive/power of attorney

---
MMSE:
I would like to ask you some questions to test your orientation.
I would like to check your memory and concentration by asking you some
questions.
Can you tell me your name and age?
Do you know where you are now?
Do you know the date today?
Show the patient your pen and ask, Do you know what this is?
Now I would like to ask you some questions to check your memory.
I will name three objects for you, and I want you to repeat them immediately,
okay? Chair, bed, and pen. (Tests immediate memory.)
I will ask you to repeat the names of these three objects after a few minutes.
(Tests short-term memory.)
Do you remember what you had for lunch yesterday? (Tests recent memory.)
When did you get married? (Tests distant memory.)
Now can you repeat for me the names of the three objects that I mentioned
to you? (Tests short-term memory.)
Are you left-handed or right-handed?
I will give you a piece of paper. I want you to take the paper in your right
hand, fold the paper in half, and put it on the table. (Three-step command.)
Now I want you to write your name on the paper.
I want you to count backward starting with the number 100, or Take 7
away from 100 and tell me what number you get; then keep taking 7 away
until I tell you to stop. (Tests concentration.)
Spell world forward and backward. (Tests concentration.)
What would you do if you saw a fire coming out of a paper basket? (Tests
judgment.)

_______________________________________
_______________________________________
PC:
elderly, forgetful, apraxia, gradual degeneration

---
1o - alzheimer
---
ddx:

V - vascular
I - neurosypghilis
N - intracranial neoplasm
D - alzheimer, B12, hypothryoid
I -
C - NPH
A - hypothryoid
T - chronci subdural
E -
M -
S - depression
---
OE:

---
ix:

---
mx:
donezepil
_______________________________________
_______________________________________
PC:
memory, gait, progressive incontinence 6 mo, elderly

---
1o - normal pressure hydrocephalus
---
ddx:

V - vascular dementia
I - neurosyphgilis
N - intracranial neoplasm
D - B12, alzheimers
I -
C - NPH
A - hypothryoid
T - chronic subdural hematoma
E -
M -
S - depression
---
OE:
wet wacky wobbly
---
ix:
MMSE, TSH (hypothyroid), B12 (wernicke)
(+) response to LP + opening pressure
MRI (periventricular lucency + cortical atrophy + ventriculomegaly)
intracranial pressure monitoring
---
mx:
acute - LP (also dx)
VP***
acetazolamide
_______________________________________
_______________________________________
PC:
rapid progressive mental status
decr concentration, memory impairment
moyclonus, ataxia, startle response
---
1o - CJD
---
ddx:

V - vascular dementia
I - neurosyphilis
N - neoplastic
D - lewy body, B12, wernicke
I -
C - NPH
A -
T - chronic subdural hematoma
E -
M -
S - depression, delirium
---
OE:

---
ix:
criterion = all of --
progressiv dementia
>1 of : myoclonus, visual cerebellar, pyrimidal/extrapyrimidal, akinetic mutism
(+) EEG, or CSF 14-3-3 protein, or MRI-B (putamen, caudate nucleus)
Brain Bx (Sp/Dx**) (PrPSc)
---
mx:
no tx - death within 12 mo

_______________________________________
_______________________________________
PC:
elderly
insulin dependent DM
confusion, dizzy, palpitations, diaphoresis, weakness

---
1o - hypoglycemia
---
ddx:

V - acute vascular dementia, TIA


I -
N -
D - dehydration, delirium
I - etoh
C - arrhythimia, angina
A -
T -
E -
M - hypoglycemia
S -
---
OE:
Palpitations, sweating, anxiety, tremor tachy
BSL < 2.5 mmol/L M, 2.2 F
neuro - cdizzy, confusion, clouded vision, mental dull, fatigue, confusion,
seizures, coma
releif with glucose
---
ix:
precip cause - insulin, alcohol, sulfonylurea
EUC, Cr, LFT, drug/toxin screen, cortisol levels

---
mx:
Acute:
IVF + BSL + glucagon IM
O2, ec, BSL monitor
thiamine 100
psych (counselling/suicide)
diabetes educator
_______________________________________
_______________________________________
PC:
elderly, AMS, HA
Hx hit head to ground + LOC

---
1o - chronic subdural hematoma
---
ddx:

V -
I -
N - neoplastic
D - CJKD
I -
C -
A -
T -
E -
M - SIADH/hyponatremia
S -
---
OE:
etoh, CSF shunt hx, anti-coag, coagulopathy
HA, confusion, dysphasia, TIA like, ICP sx, progressive dementia, gait
GCS
neuro ex + pupillary sx wrt to herniation = CN3 paralysis = down and out
---
ix:
CT - hypodense, crescentic (MRI greater Sn for small)
angiography
---
mx:
antiepilective
tx coag
burr hole sx

_______________________________________
_______________________________________
PC:
elderly
acute unilateral vision loss
palpitations, SOB
[no HA, weakness, numbness]
hx - AF, cataracts

---
1o - retinal artery occlusion
---
ddx:

V - retinal vein occulsion, ACUTE angle closure glaucoma


I - temporal arteritis
N -
D -
I -
C -
A -
T - retinal detachment
E -
M -
S -
---
OE:
retinal artery occlusion: (emboli from arrhythmia/endocardidis/valvular), temporal
artiertis
sudden, painless, monoocular loss vision
RAPD (= marcus gunn pupil = lesion afferent pathway to optic chiasm = decr
direct pupillary response to light but accommodation preserved
light in affected eye = direct + consensual decr
light in unaffected = normal direct/consensual
move from unaffected to affected = paradoxical dilation of both pupils
previous amaurosis fugazx
fundoscopy (cherry red spot, retinal pallor, narrow arterioles/boxcaring,
cotton wool spots (retinal infarcts), cholesterol emboli)

retinal vein occlusion


sudden, painless, monoocular loss vision
RAPD
fundoscopy (blood and thunder = diffse retinal hemorrhages, cotton wool
spots)

---
ix:
fundoscopy
---
mx:
retinal artery occlusion: MUST START WITHIN 2 HR of symptoms =
massage globe/compress eye to dislodge embolus
decr intraocular P (topical BB, O2/CO2, IV acetazolimide, IV mannitorl, drain

retinal vein occlusion: fluorescein angiography, retinal lazer photocoagulation,


intravitreal anti-VEGF
_______________________________________

_______________________________________
PC:
elderly, 2 mo hx crying spells, incr sleep, poor hygiene, weight loss following
wife death
thinks that he sees his wife in the street
---
1o - normal bereavement
---
ddx:

V -
I -
N -
D -
I -
C -
A -
T -
E -
M -
S - adjustment disorder with depressed mood
MDD
schizoaffective

Bereavement
normal = protest, searching/acute anguish, despair/detachment, reorganisation
abnormal or MDD =
- GUILT wrt things but exlcuding acts/omission at the time of death
- THOUGHTS OF DEATH but excluding surivior feeling they wld be better off
dead, should have died, preoccupation with worthlessness
- PSYCHOMOTOR retardation
- HALLUCINATIONS other then hearing voice or images of deceased
- DYSPHORIA pervasive and independent
RF for poor prognosis in bereavement:
- supports (poor)
- unanticipated death
- dependent r/ship on deceased
- hi initial distress
- concurrent stressor/losses
- death of child
- pre-existing psychiatric disorder
---
OE:

---
ix:
exam - mental state exam
TSH
CBC
urine toxicology
---
mx:
counselling
_______________________________________

_______________________________________
PC:
4 wk hx excessive fatigue, insomniia, anhedonia, thinks abot death, 5 episodes bf,
2 prev suicide attempts, incr alcohol

---
1o - MDD
---
ddx:

PHAM
P. parkinsons
H. hypothyroid
A. abuse/alcohol (substance abuse)
M. medications

- parkinsons
levodopa
- hypothyroid
check TSH
thyroxine
- abuse/alcohol (substance abuse)
CAGE - alcohol
amphetamines
detox + ssri
- medications
corticosteroids
BB
antipsychotics
reserpine

---
OE:
Criterion for MDE:
Either depressed mood or anhedonia + >5 other sx for > 2 weeks

MISGECAPS

Mood: depressed
Sleep: increased/decreased
Interest: decreased
Guilt
Energy: decreased
Concentration: decreased
Appetite: increased/decreased
Psychomotor: agitation/retardation
Suicidal ideation

---
ix:
TSH, urine screen, cortisol levels
physical exam
mental status exam
Beck depression inventory
blood alcoohol level
TSH
CBC
urine screen
---
mx:

- admit IF suicidal/homicidal/paranoia
1o - ssri (snri, A2(-) mirtazapine)
2o - incr dose
3o - change class + waiting period 4 wk
4o - add adjuvant - buproprion, quetiapine, aripiprazole, lithium
5o - ECT

Management of depression:

"A SET PACE" =

assessment

safety
education
therapeutic r/ship

psychological therapy (CBT for depression +/- atypical features, CONTRA for
melancholic features or psychotic features)

antidepressant tx (depression 1o = ssri; atypical = ssri/maoi; melancholic =


tca/snri/mirtazapine/ssri; psychoit = tca)

combination (CBT + antidepressant = depression/atypical/melancholic; TCA +


antipsychotic for psychotic features ie no CBT)

ECT (if suicide risk, severe, previously effective)


_______________________________________

_______________________________________
PC:
wt loss over 2 mo, early morning awakening, excess guilt, psychomotor retardation.
episodes of incr activity/energy, sexual promiscuity, irresponsbile spending,
racing thoughts

---
1o - bipolar 1
---
ddx:

V -
I -
N -
D -
I -
C -
A -
T -
E -
M -
S -
---
OE:
Criteria for Manic episode:
>1 wk most of the day OR hospitalised
>3 DIG FAST

DIGFAST

distractible
insomnia
grandiose/goal
flight of ideas
activities/agitiation
sexual
talkative

hypomania criteria:
NO psychotic features - ie no grandiosity (delusions/psychosis)
DIGFAST for > 4 d (less than 1 wk, which is manic)

mania vs hypomania:
hypomania is for bw 4-7d (mania is for >1 wk)
hypomania has no psychotic features of grandiosity
---
ix:
physical exam
mental status exaqm
urine toxicology
---
mx:
Manic symptoms + drug use. 1st step in management ?: UDS for amphetamine
Manic symptoms + htn. ddx ?: pheochromocytoma
Manic symptoms + low tsh. ddx ?: hyperthyroidism

Mx acute mania:
1o admit
2o lithium (takes 1 wk)
3o Rispiridone (or other antipsychotic to control acute mania)
40 phenothiazine IM depot in non compliant, severe patients

BPD Mx:
(acute = benzo)
1o psychoeducation (nature of illness) + emergency plan + lifestyle (sleep,
nutrition, meals)
2o mood stabilisers (lithium, valproate)
3o atypicals (lurasidone, aripirazole, olanzapine)
4o ECT

longer term:
CBT
vocational rehab

4Ls of bipolar depression: lithium, lamotrigine, lurasidone (atypical),


seroqueL (quetiapine)
BP-2-D depression 1o = quetiapine
Mixed/cycling: lithium or valproate + atypical (lurasidone, aripirazole,
olanzapine)

What drug has been shown to prevent suicidal ideation in bipolar


disorder?:Lithium
_______________________________________
_______________________________________
PC:
receiving message from TV
suspects friend bugging telephone
suspects professors saying things about him
rarely leaves room
---
1o - schizophrenia
---
ddx:

psych
brief psychotic = <1mo
schizophreniform = 1-6mo {2/3 progres to schizoophrenia]
Delusional disorder = no impairment of baseline function + non bizarre
delusions
Personality disorder
med
thyroid (tsh)
UEC, CMP
HIV (serology)
syphilic (VDRL)
drugs (UDS)
temporal lobe epilepsy
---
OE:
2+ Sx for >1mo with >6mo of impaired baseline functioning
POSITIVE (incr DA in mesolimbic)
delusion
hallucination
disorganized speech
grossly disorganised or catatonic behaviour
NEGATIVE (decr DA prefrontal, M-receptors) - atypicals antipsychotics best for
these
flat affect,
social withdrawal,
anhedonia,
apathy,
poverty of thought)
---
ix:
mental status exam
urine screen
tsh
cbc
UEC
---
mx:
1o admit IF - delusions bizarre / paranoid
2o benzo (agitation) + antipsychotic (haloperidol, respiridone)
3o adj
- mood stabilisers for aggression/impulsiveness
(lithium/valproate/carbamazepin)
- anxiolytics
4o ECT
5o CBT
6o long term = 2 yr after initial episode, 5 yr if multiple episodes

_______________________________________
_______________________________________
PC:
recent ingestion of substance
sees bugs crawling
hears voices when alone
no previus experience

---
1o - substance induced psychosis
---
ddx:
brief psychotic disorder
schizophreniform
schizophrenia
---
OE:

---
ix:
urine drug screen
mental status exam
tsh
cbc
euc, bun/cr, ast/alt
---
mx:
ensure safety of patient ? admit
decr stimulation
benzodiazepines: lorazepam 2 mg PO/IM/SL
antipsychotics: olanzapine 5 mg PO, haloperidol 5 mg PO/IM
treat underlying medical condition
psychiatry or Crisis Intervention Team consult
_______________________________________
_______________________________________
PC:
35F, 1 wk hx of - intermittent vertigo, tinnitus, nausea hearing loss

---
1o - menieres dz
---
ddx:

V -
I - vestibular neuronitis, labyrinthitis, syphillis
N - acoustic neuroma
D -
I - benign psoitional vertigo
C -
A -
T -
E -
M -
S -
---
OE:
BPPV
lasts seconds at a time
better with dix-hallpike (head 45o moved sitting to supine with head hanging
over bed = onset of vertigo = (+) for that sid
no hearing loss or tinnitis
Menieres
mimutes to hours
hearing loss
tinnitis
drop attacks
Labyrinthitis = inflamm of vestibular part of CN8
hours to days
hearing loss + tinnitus
hx of AOM
vertigo with N/V 5 d
irritative nystagmus (fast phase toward offending ear)
ataxis toward affected side
Acoustic neuroma (vestibular schwannoma)
chronic
progressive hearing loss
tinnitis

---
ix:
CBC
VDRL/RPR ddx syphilis
MRI-B
dix-hallpike maneouver

ix acoustic neuroma
MRI + gad contrast
audiogram
absent stapedial reflex
---
mx:
BPPV
reassure
epley / bradth daroff
antiemetics
menieres
acute = bed rest, antiemetics, meclizine, scopalamine
chronic = low salt, diuretics, betahistine, dexamethasone
monitor opposite side
acoustic neuroma
expectant, surgical excision, gamma knife/radiation
labyrinthitis
acute = rest, methypred, abx if bacterial
chronic = vestibular exercising
_______________________________________
_______________________________________
PC:
55F, acute reduced hearing, N + V, 2d hx of URTI

---
1o - labyrinthitis
---
ddx:

labyrinthitis
vestibular neuronitis
menieres
acoustic neuroma
vertebroinusfficience
---
OE:
hours to days
hearing loss + tinnitus
hx of AOM
vertigo with N/V 5 d
irritative nystagmus (fast phase toward offending ear)
ataxis toward affected side
---
ix:
audiogram
EMG
MRI B
---

mx:
acute = rest, methypred, abx if bacterial
chronic = vestibular exercising
_______________________________________
_______________________________________
PC:
26 M fell, lost consciousness
rhythmic mvment + bit tongue, lost bladder control
witnessed
confused afterwards

---
1o - general tonic clonic seizure
---
ddx:

V - syncope, TIA
I -
N -
D - hypoglycemia, hypoxia, hyperventilation
I - substance withdrawal
C -
A -
T -
E -
M -
S - malingering
---
OE:
? aura
eye rolling
tongue biting
steroptyped

pseudoseizure (rigid, eye closure, head shaking, pelvic thrust, lip bite
---
ix:
CBC
UEC, BSL, Ca/Mg, ESR, Cr, ALT/AST, CK, prolactin
urine drug screen
EEG - normal in syncope and pseudoseizure
MRI
CT
LP - saf
ECG
---
mx:
avoid precip
antiepileptic if (>1 unprovoked seizures, organic brain dz, EEG with epileptiform,
1st status epilepticus)
general - valproate, leviracetam
partial - carbamazepine, gabapentin, phenytoin
absence - ethosuxcimide
education
safety - driving/machinery, swimming
_______________________________________
PC:
55M, fell after dizzy, transiet LOC, pmhx htn + DM

65M, fell + LOC seconda. No warning. Hx palpitations + prev CABG

---
1o - drug induced orthostatic hypotension
---
ddx:

V - stroke, MI, pulmonary embolus


I -
N - vasovagal
D - hypoglycemia
I -
C - arrhythmia
A -
T -
E -
M -
S -
---
OE:
myoclonic jerk = syncope not seizure
postural BP + HR
CV, Resp, neuro exam
"I hate falling"
inflamm jnt, hypotension, auditory, tremor, equilibrium, foot prob,
arrhythmia, leg length, lack fitness, ill, nutrition, gait
---
ix:
details from witness
ECG, BSL
CBC, UEC, BUN, Cr, ABG, troponin, Ca Mg bHCG
urine drug screen

echo, holter monitor


CT head

---
mx:
ABC, IVF, O2, monitor
trauma from fall

_______________________________________
PC:
68 yo M presents following a 20-minute episode of
slurred speech, right facial drooping and numbness,
and right hand weakness. His symptoms had totally
resolved by the time he got to the emergency department.
He has a history of hypertension, diabetes
mellitus, and heavy smoking.
---
1o - TIA
---
ddx:

V - transient ischemic attack, stroke


I -
N - seizure, facial nerve palsy
D - hypoglycemia
I -
C -
A -
T -
E -
M -
S -
---
OE:

---
ix:
ABCs, vitals, BSL
1o - CT-B non contrast (rule out hemorrhage)
2o - ECG (rule out AFib)
3o - cardiac dopplers, echocardiogram
CBC, EUC, Cr, PTT/INR, lipids
Sp/Dx = non contrast CT (distinguish ischemic from hemorrhagic stroke)

---
issues - short term
1. management of strong risk
2. management of hypertension
3. diagnostic and management of htn

issues - long term


4. manage risk factors - DM, obesity, htn
5. manage complications of risk factors
6. Social - impact on income, lifestyle

mx - ischemic - acute
1o. tPA if <4.5hr from sx (and no contraind = seizure, rcent surg, GI bleed, LP)
2o. ASA 81 mg PO (add dipyrimadole if recurrent; ASA allergy use clopidogrel)
3o. TIA/stroke + AF = heparin IV (INR2-3) then warfarin
4o. intra-arterial thromboectomy with interventional radiology
5. Control BSL (hypergylcemia incr infarct)
6. Conrol temperature (mx febrile endocarditis ie septic emboli)
7. dysphagia (NPO)
8. DVT prophylaxis (compression socks, mobilisation, intermittent pneumatic
compression) - UFH 5000 IU SC bid
9. rehab/mobilise early
4o. BP control 170/110 permissive hypertension for penumbra perfusion (vs
hemorrhagic need lower)

mx - secondary prevention / stroke recurrence


- mx RFs (carotid stenosis, AF, hypertension, hypercholesterol, DM, smoking,
inactivity, stroke rehab)
- ASA (or clopidogrel)
-carotid endarterectomy/stenting if stenosis >70%
-AF = CHADS2 score = anticoag (<2 = ASA, else INR 2-3 with warfarin,
dabigatran (reversal with Praxbind), apixaban, rivaroxaban
-stroke/tia hi risk = htn (<140/90 or <130/80 in DM/renal dz) = ramipril 10
mg OD;
- tia/stroke 2nd prevention use ACEI + thiazide diuretic
-hypercholesterolemia = 1o prevention = statins; 2o prevention = hi dose
atorvastatin
-DM HbA1C < 7
-smoking cessation

mx - hemorrhagic
1o supportive

ABCD2 Score
To predict/identify individuals at high risk of stroke following TIA
Age: 1 point for age >60 yr
Blood pressure (at presentation):1 point for HTN (>140/90 mmHg at initial
evaluation)
Clinical features: 2 points for unilateral weakness, 1 point for speech disturbance
without weakness
Duration of symptoms: 1 point for 10-59 min, 2 points for >60 min
DM: 1 point Stroke risk: 0-3: low risk, 4-5: moderate risk,6-7: high ris

The risk for stroke can be estimated from the ABCD2 score as follows:
Score 1-3 (low)
2 day risk = 1.0%
7 day risk = 1.2%
Score 4-5 (moderate)
2 day risk = 4.1%
7 day risk = 5.9%
Score 67 (high)
2 day risk = 8.1%
7 day risk = 11.7%
_______________________________________
PC:
33 yo F presents with ascending loss of strength in
her lower legs over the past 2 weeks. She had a recent
URI.

---
1o - GBS
---
ddx:

V -
I -
N - peripheral neuropathy, tumor canal neoplasm
D -
I -
C -
A - GBS, MS, MG
T -
E -
M - polymyositis
S -
---
OE:
symmetric ascending sensory/motor loss (demyelinating inflamm)
hx - campylobacter jejuni
---
ix:
1o - CSF - hi protein, normal WBC (albumin-cytological dissociation)
2o - EMG/NCS
---
mx:
1o IVIG (plasmapheresis)
20 pain management
3o vitals
4o PFTs

_______________________________________
PC:
30 yo F presents with weakness, loss of sensation,
and tingling in her left leg that started this morning.
She is also experiencing right eye pain, decreased vision,
and double vision. She reports feeling electric
shocks down her spine upon flexing her head.

---
1o - MS
---
ddx:

V - stroke, CNS vasculitis


I - neurosyphilis
N - CNS tumor
D -
I -
C - syringomyelia
A -
T -
E -
M -
S - conversion disorder , malingering
---
OE:
chronic inflamm CNS - infalmm, demyelination, axonal degeneration
numbness
optic neuritis (visual loss)
weakness
spasticity
diplopia + INO
gait
vertigo
bladder dysfunction
Lhermitts sign - flexion causes electric shock down into limbs
Uhtoffs - optic neuritis/symptoms worse in heat

NOT (visual field defect, aphasia, apraxia, progressive hemiparesis)

ix:
dissemninated in time and space:
>2 (gad contrast or non-contrast MRI at any time)
>1 lesions T2 MRI lesions in at least 2 of the 4 CNS regions (periventricular,
juxtacortical, infratentorial, spinal cord)
1o MRI T2 (periventricular, corpus callosum, cerebellar peduncles, brainstem,
juxtacortical)
2o CSF oligoclonal bands in 90% incr IgG concn
3o evoke potentials

---
mx:
acute =
1o methylprednison 1000 mg IV OD x 7 d
2o plasma exchange
Disease modifying therapy
1o teriflunomide, INF-B, glatimer aceta, BG-12
2o natalizumab, fingolimnod (risk PML)
Symptoms
spasticity - baclofen,dantrolene, benzo, botulinum
bladder dysfn - oxybutynin
pain - tca,carbamazepine, gabapentin
fatigue - amantadine, modanafil, methylphenidate
depression - ssri, lithium
constipation - fibre, lactulose
sexual dysfunction - sildenafil
Educaton and counselling, MS society support
_______________________________________
PC:
55 yo M presents with tingling and numbness in his
hands and feet (glove-and-stocking distribution)
for the past 2 months. He has a history of diabetes
mellitus, hypertension, and alcoholism. There is decreased
soft touch, vibratory, and position sense in
the feet.

---
1o - Myasthenia gravis

anti-AChR or anti-MuSK antibodies at the NMJ


---
ddx:

V -
I -
N - horners, neoplasm compressing CN 3, 4, 6, amyotrophic lateral sclerosis
D -
I -
C -
A - MS
T -
E -
M -
S -
---
OE:
decrement, fatiguable
symmetric or asymmetric weakness w/o reflex changes, sensory changes or
coordination abn
ocular = diplopia/ptosis
bulbar = dysarthria/dysphagia
proximal limb weakness
---
ix:
Tensilon test = edrophonium test - improves after injection
EMG = repetitive simulation = decrementing response
spirometry FVC
anti-ACh-receptor antibody assay (90% Sn)
single fibre electromyography shows incr jitter (100% Sn)
CT/MRI screen for thymoma/thymic hyperplasia
---
mx:
crisis = VOG + plasmapheresis

1o Steroids + sparig (azothioprine, cyclophos, mycophenolate)


2o AChE inhibitors pyridosstimgine
3o thymectomy
_______________________________________
PC:
25 yo M presents with hemiparesis after a tonicclonic
seizure that resolved within a few hours.

---
1o - todds paralysis
---
ddx:

V - stroke***/tia, migraine
I -
N -
D -
I -
C -
A -
T -
E -
M -
S - malingering
---
OE:
paralysis after a seizure
postictal period typically include confusion and suppressed alertness. Focal
neurologic deficits may also be present, often referred to as Todd paralysis or
postictal paresis. The classic example of postictal paresis is weakness of a hand,
arm, or leg that appears following a focal motor seizure involving the one side of
the body
---
ix:
eeg
EUC, glucose, CMP
CBC
RFT, LFT, UA, urine drug screen
BHCG
CPK, cortisol, WBC, LDH,
ECG
LP
---
mx:

_______________________________________
PC:
56 yo obese F c/o tingling and numbness of her
thumb, index finger, and middle finger for the past
5 months. Her symptoms are constant, have progressively
worsened, and are relieved with rest. She
works as a secretary. She has a history of fatigue and
a 20-lb (9-kg) weight gain over the same period.

---
1o - carpal tunnel - hypothryoid

median nerve compression at the level of the flexor retinaculum as opposed to


pronator teres syndrome (compressive neuropathy at the level of the elbow
---
ddx:

V -
I -
N -
D -
I -
C -
A -
T -
E -
M - overuse - median nerve, medial epicondylitis (golfers elbow)
S -
---
OE:
wrist pain, paresthesia 3 + 1/2 digits
radiation to elbow
worse at night - better with shaking, dangling, rubbing
tinels/phalens sign
thenar muscle wasting
---
ix:
1o Dx/specific = EMG/NCV
---
mx:
1o splints + OT
2o NSAIDS, local corticosteroid injection, corticosteroid PO
3o surgical decompression (indic = persistent signs and sx of nerve compression not
relieved by conservative management).
_______________________________________
PC:
55 yo M presents with tingling and numbness in his
hands and feet (glove-and-stocking distribution)
for the past 2 months. He has a history of diabetes
mellitus, hypertension, and alcoholism. There is decreased
soft touch, vibratory, and position sense in
the feet.

---
1o - diabetic peripheral neuropathy
---
ddx:

V -
I -
N - alcoholic peripheral neuropathy
D - B12 def
I -
C -
A -
T -
E -
M - hypocalcemia, hyperventialation
S -
---
OE:
128 Hz
screen for orthstatic hypotension (postural)
---
ix:
HbA1c
ESR
Calcium
B12
UA

---
mx:
glycemic control
pain control - TCA (amitriptyline, pregabalin, duloxetine,
gabapentin/carbamazepine)
education - foot cares
screen for retinopathy/orthostatic hypotension
_______________________________________
PC:
44 yo M presents with fatigue, insomnia, and nightmares
about a murder that he witnessed in a mall 1
year ago. Since then, he has avoided the mall and
has not gone out at night.

---
1o - PTSD
---
ddx:

V -
I -
N -
D -
I -
C -
A -
T -
E - hypothyroid
M -
S - MD, GAD, schiz
---
OE:
TRAUMA
traumatic event
re-experience event
avoidance of stimuli assoc with trauma
unable to function
more than a month
arousal increased + negative alterations in congnition and mood
---
ix:
CBC
tsh
urine drug screen
beck depression inventory
---
mx:
1o CBT
2o ssri (paroxetine), prazosin (stops dreams/nightmares), benzos (anxiety),
antipsych (respiridone, olanzapine)
3o eye movement desensitisation and reprocessing EMDR
_______________________________________
PC:
55 yo M presents with fatigue, weight loss, and constipation.
He has a family history of colon cancer.

---
1o - Colon cancer
---
ddx:

V -
I -
N -
D -
I -
C -
A -
T -
E - hypothryoid, hypercalcemia, renal failure
M -
S - MD
---
OE:
familial adenomatous polyposis = AD for APC Ch5q21
.thousands polpys
.hypertrophy of retinal pigment epithelium
.100% risk of clon cancer
.can be assoc with CNS tumor/medulloblastoma, or with sebaceous cysts,
osteomas, desmoid tumor
Lynch (hereditary non polyposis colorectal cancer) = AD, DNA mismatch repair
gene msh2/msh6/mlh1
.assoc endometrial ovarian hepatobiliary small bowel cancer
.amsterdam criterion
-3+ relatives with Lynch (1 first degree)
-2+ generation
-1+ dx before 50 yo
-FAP excluded
Colon cancer:
iron deficiency anemia, wt loss, palpable mass
diet - incr fat, red meat, decr fibre
smoking
DM, acromegaly
hematochezia/melena
abdominal pain
obstruction/constipation, tenesmus, rectal bleeding, constipation,
overflow, decr stool calibre
mets - lymphatc, hematogenous (liver, lung, bone, brain, rectum -> IVC
-> lung
---
ix:
PR, stool, FOB
CBC
electrolytes, BUN/Cr, calcium
AST/ALT
TSH
colonoscopy****(gold standard)
barium enema
CT-Abd/pelvis

FAP:
genetic testing
annual flexible sigmoidoscopy/colonscopy

Lynch:
genetic
colonscopy from age 20 anual
surveillance extracolonic

Colon cancer:
colonscopy
CBC, UA, liver enzymes, LFT, CEA
stage CT-chest/abd/pelvis
---
mx:
follow up endoscopy 1yr then every 3-5 yr

Lynch:
total colectome, ileorectal anastomosis with annual proctoscopy

Colon cancer:
wide surgical resection of lesion + regional lymphatic drainage + colectomy +
anastomosis
5FU or oral capecitabine with oxaliplatin

Rectal cancer:
low anterior resection
_______________________________________
PC:
40 yo F presents with fatigue, weight gain, sleepiness,
cold intolerance, constipation, and dry skin.

---
1o - hypothyroid
---
ddx:
hypothryoid ddx =
hashimotos -autoimmune
post ablation / RAI
lithium
iodine def

V -
I -
N -
D -
I -
C -
A -
T -
E - DM, aenmia
M -
S - MD
---
OE:
HIS FIRM CAP
Hypoventilation
Intolerance to cold
Slow HR
Fatigue
Impotence
Renal impairment
Menorrhagia/amenorrhea
Constipation
Anemia
Paresthesia + slow deep tendon reflex

---
ix:
1. TSH -hi TSH (primary), -lo (secondary/pituitary)
2. free T3, T4 (lo)
3. thyroglobulin
4. antimicrosomal antibodies = (anti-thyroid peroxidase, anti thyroglobulin =
hashimoto)

CBC
fasting Glu - HbA1c
ACTH + cortisol (check fo adrenal insuff)
---
mx:
Levo-thyroxine 0.2 mg PO OD
TSH annual check
_______________________________________
PC:

50 yo obese F presents with fatigue and daytime


sleepiness. She snores heavily and naps 34 times
per day but never feels refreshed. She also has hypertension.
---
1o - OSA
---
ddx:

V -
I -
N -
D -
I -
C -
A -
T -
E - hypothryoid
M -
S - narolepsy, chronic fatiue
---
OE:
daytime sleepiness (epworth sleepiness scale > 9 = risk....fall asleep when
sitting/reading, watching tv, sitting inaactive, passenger in car, lying down to
rest in afternoon, sitting/talking to someone, after lunch, when driving)
fatigue
sleep diary finding
---
ix:
sleep diary
CBC
TSH
polysomnography
ECG
---
mx:
conservative = wt loss, avoid alcohol, sedatives, sleep deprivation
obstructive = CPAP
central =acetazolamide, progesterone, supplemental O2
_______________________________________
PC:
20 yo M presents with fatigue, thirst, increased appetite,
and polyuria.

---
1o - DM
---
ddx:

V -
I -
N -
D -
I -
C -
A -
T -
E - DI
M -
S - MD, polydipsia
---
OE:
polydipsia, polyruia, polyphagia, weight loss, blurry vision

HONK (hyperosmolar nonketotis coma = due to hyperglycemic osmotic diuresis and


dehydration) = weakness, polyuria, polydipsia, lethargy, confusion, convulsions,
coma
---
Dx: 1. Symptoms + (+)BSL tests as below:

FBG > 7mmol/L (200 mg/dL), or


2h 75g OGTT > 11.1 mmol/L (200mg/L), or
random BG > 11.1 mmmol/L (200 mg/dL), or
HbA1c

2. No Sx + 2 x (+)BSL (ie repeated tests)


FBG > 125 mg/L
2hr OGTT > 200 mg
HbA1C > 6.5%

other Ix:
UA > 6.5%
CBC
UEC
BUN/Cr (metformin contra in renal insufficiency ie Cr > 2 bc risk lactic
acidosis)

HONK:
BSL > 700 mg/dL
ser osm > 290... (2xNa + glu/18 + BUN/3)
---
mx:

aim HbA1c < 8.5 if DM2 + CAD/hypos, otherwise aim < 7


1. education, weight loss 4%, diet modif/lo fat (sat < 7% energy, polyun<10), limit
sodium, etoh, caffeine, exercise 150min/wk, smoking cessation
2. - check LFT = ok
- check RFT = ok
- then;
metformin (or GLP-1 - exenatide/liraglutide if metformin not tolerated)
- if abn LFT -> insulin
- if abn RFT -> sulfonylurea/thiazolidinones (glits)
3. add sulfonylurea - glyburide, glipizide, glimepiride (if not controlled)
4. insulin (or glitazone/thiazolidone but contra in CHF/bladder cancer)
-1 x long acting Glargine
-3 x short acting premeal lispro/aspart

HONK:
fluid resusc
EUC correction
insulin

Microvascular
nephropathy:
screen Cr, ACR, UA
BP 130/80 = ACEI, ARB
avoid nephrotoxic
neuropathy (glove/stocking polyneurophathy):
pain - gabapentin, TCA
feet - podisatrist, stocking
retinopathy: annual Ophthal review
gastroparesis: metoclopromide, erythromycin

Macrovascular (CADm PVD stroke)


CAD + DM = hi risk (Framingham) = tight glycemic control, LDL <2 (decr fat +
incr fibre 30 g/d, hi dose atorvastatin)
Stroke =
.BP control - <130/80 = ACEI, .ASA (or clopidogrel)
.carotid dopplers
.ecg for AF -> CHADS2 - warfarin
PAD - ankle/brachial index
smoking cessation: NRT, bupropion (DA uptake (-), varenicline (partial Ni(+))

_______________________________________
PC:
35 yo M policeman c/o feeling tired and sleepy during the day. He changed to the
night shift last week.
---
1o - shift work sleep disorder
---
ddx:

V -
I -
N -
D -
I -
C -
A -
T -
E - anemia
M -
S - OSA, MD
---
OE:

---
ix:
CBC
nocturnal pulse oximetry
polysomnography
---
mx:

_______________________________________
PC:
25 yo F presents with a 3-week history of difficulty
falling asleep. She sleeps 7 hours per night without
nightmares or snoring. She recently began college
and is having trouble with her boyfriend. She drinks
34 cups of coffee a day.

---
1o - stress induced insomnia
---
ddx:

V -
I -
N -
D -
I - caffeine
C -
A -
T -
E -
M -
S - circadian rhythm sleep disorder, MDD
---
OE:

---
ix:
polysomnography
MSE
urine toxicology
CBC
TSH
---
mx:

_______________________________________
PC:
55 yo obese M presents with several months of poor
sleep, daytime fatigue, and morning headaches. His
wife reports that he snores loudly.

---
1o - OSA
---
ddx:

V -
I -
N -
D -
I -
C -
A -
T -
E -
M -
S - circadian rhythm sleep disorder, MDD
---
OE:

---
ix:
CBC
TSH
polysomnography
ECG
---
mx:

_______________________________________
PC:
33 yo F c/o 3 weeks of fatigue and trouble sleeping.
She states that she falls asleep easily but wakes up at
3 A.M. and cannot return to sleep. She also reports
an unintentional weight loss of 8 lbs (3.6 kg) and an
inability to enjoy the things she once liked to do.

---
1o - MD insomnia
---
ddx:

V -
I -
N -
D -
I -
C -
A -
T -
E -
M -
S - primary hypersomnia, circadian rhythm
---
OE:

---
ix:
MSE
TSH
CBC
polysomnography
---
mx:

_______________________________________
PC:
26 yo F presents with sore throat, fever, severe fatigue,
and loss of appetite for the past week. She
also reports epigastric and LUQ discomfort. She has
cervical lymphadenopathy and a rash. Her boyfriend
recently experienced similar symptoms.

---
1o - infectious mono /EBV
---
ddx:

V -
I - EBV, hepatitis, pharyngitis, acute HIV, secondary syphilis
N -
D -
I -
C -
A -
T -
E -
M -
S -
---
OE:
15-90yo
kissing/sex
3d malaise/anorexia prodrome
malaise, fatigue, fever, sore throad abd pain LUQ, HA, myalgia
trid = fever, non tender LNpathy, pharyngitis/tonsilitis (exudative)
HSmegaly
periorbital edema
rash (urticarial,, maculopapular, petechial)

---
ix:
CBC + PBS (reactive lymphocytes, lymphocytosis, downey cells, anemia,
thrombocytopenia)
monospot (false positive - HIV, SLE, lumphoma, rubella, parvovirus)
throat swab/culture for strept pharyngitis
AST/ALT/bilirubin/ALP
HIV-ab + viral load
anti-ebv Ab (titres)
VDRL/RPR

---
mx:
.rest, hydration, saline garge,
.avoid sports (splenic rupture)
.monitor for GBS
_______________________________________
PC:
26 yo M presents with sore throat, fever, rash, and
weight loss. He has a history of IV drug abuse and
sharing needles.

---
1o - HIV retroviral syndrome
---
ddx:

V -
I - EBV mono, hepatitis, pharyngitis, strep tonsil, 2o syphilis
N -
D -
I -
C -
A -
T -
E -
M -
S -
---
OE:
lactic acidosis (due to mitochondrial toxicity) - abd pain, fatigue, N/V, mm
weakness
lipodystrophy = body fat redistribution = dorsal fat pad, breast enlargement, incr
abd fat - due to protease inhibitors
lipodydstrophies
lipohypertrophy = facial thinning, decr adipose tissue in extermities = due
to thymidine analogue NRTIs d4T and AZT
Dyslipidemia - incr LDL, TG
Glucose resistance TSDM
CVD risk

NRTIs - Lact acid, lipodystrophy, rash, N/V/D, BM suppre, neuropathy, drug induce
hypersensitivity, pancreatitis, myopathy

nonNRTI - rash, stevens johnson, CNS/dizzy/insomnia/somnolecence, hepatotoxicity

protease inhibitor - lipodystrophy, metabolic syndrome, N/V/D, nephrolithiasis,


rash, hyperbilirubinemia, CYP3A4 induce
---
ix:
CBC + PBS
HIV Ab + viral load
CD4 count
monospot
throat culture
VDRL/RPR
AST/ALT/bilirubin/ALP
---
mx:

_______________________________________
PC:
46 yo F presents with fever and sore throat.

---
1o - pharyngitis
---
ddx:

V -
I -mycoplasma pneumonia, acute HIV, EBV mono
N -
D -
I -
C -
A -
T -
E -
M -
S -
---
OE:
viral = pharyn, conj, rhino, hoarse, cough, lo fever, malaise, myalgia

GABHS Centor = NO cough, >38C, tonsillar exudate, swollen anterior nodes

redflag = >1wk, stridor/croup, dribbling, dsphagia, pain, palpable mass, blood in


pharynx/ear
---
ix:
throat swab - culture + strep antigen
monospot
CBC
serologic (cold agglutinin titre for mycoplasma)
HIV Ab and viral load
---
mx:
viral = none
Strept GABHS = penicillin V

_______________________________________
PC:
30 yo M presents with shortness of breath, cough,
and wheezing that worsen in cold air. He has had
several such episodes in the past 4 months.

---
1o - asthma
---
ddx:

V -
I - bronchitis, pneumonitis,
N -
D -
I -
C -
A -
T -
E -
M - GERD , FO
S -
---
OE:
eti:
- airways hypersensitivity (SM hypertrophy, increased mucous production) ->
obstruction -> V/Q -> hypoxema -> incr ventilation RR -> alkalosis -> muscle
fatigue -> decr vent + acidosis

triggers:
- atopy + eczema + allergic rhinitis
- allergy
- responsiveness - cold air, exercise

SOB
chest tightness
pulsus paradoxus
tracheal tug
accessory muscles
cyanosis
? silent chest
tachypnoea
expiratory wheeze

control ? good = 0, partial <3, poor >3


- Daytime symptoms >2 days per week
- Need for reliever >2 days per week
- Any limitation of activities
- Any symptoms during night or on waking

Severe/arrest ?
- cannot speak in sentences - only words
- no relief from SABA
- O2 < 90, FEV1 < 50, RR > 30, pCO2 > 45
- silent chest, diaphoretic, cyanotic

---
ix:
O2 sat
ABG

CBC
CXR
PFT
peak flow
methacholine challenge
---
mx:

confirm diagnosis
- Fixed (irreversible or incompletely reversible) airflow limitation (post-
bronchodilator FEV1/FVC < lower limit of normal for age and FEV1 <80% predicted)
assess asthma control
educate - self monitorying (peak expiratory flow meter, smoking, healthy eating,
physical activity, healthy weight and immunisation, avoid triggers
review use of bronchodilators
< 2/mo - SABA as needed
> 2/mo - SABA + lo dose ICS (or montelukast or cromone)
uncontrolled - SABA + hi dose ICS
flare = repeated SABA w/in 3 hr period OR reduced activity OR waking up at
night/SOB
manage flare ups - asthma action plan

Acute Mx:
100% O2 + cardiac monitor + IV access
? propofol induction
SABA 5 mg nebulier continuous
short acting anticholinergic atrovent 0.5 mg nebuliser x 3
methylprednisone 125 mg IV
MgSO4 2g IV

_______________________________________
PC:
56 yo F presents with shortness of breath and a productive
cough that has lasted for at least 3 months
each year over the past 2 years. She is a heavy
smoker.

---
1o - COPD - chronic bronchitis
---
ddx:

V -
I - bronchiectasis, TB
N - lung cancer
D -
I -
C -
A -
T -
E -
M -
S -
---
OE:

---
ix:
clinical diagnosis
CBC
sputum gram stain and culture (usually not helpful)
CXR - if pneumonic
PFT + methacholine challenge
-FEV1 >15% / 200mL incr after bronchodilator
CT-chest
PPD
---
mx:
primary prevention - handwash, smoking stop, avoid trigger
Sx relief - rest, fluids, analgesia, bronchodilator
mx chronic COPD
1o (these all prolong survival) =
smoking cessation (NRT, bupropion, varenicline),
vax (influenza, pneumococcal),
home O2 (>15 h/d) indic if paO2<55 or <60+corpulmonale

2o (for sx only) =
ICS + LABA (Fluticasone + salmeterol)
Bronchdilators = M(-) ipratropim bromide + B(+) SABA
3o Nurse educator = education, avoid irritants,
PT + OT = exercise rehab improve endurance
4o Review past admission notes + contact GP + review medications

prognosis COPD = BODE index


BMI < 21 = 1pt
Obstructive FEV1 50-64 = 1pt, 36-49 = 2, <35 = 3pt
Dyspnoea : walks slow + stops = 1, stops = 2, too breathless at rest for adls
= 3
Exercise capacity (6 min walk) = 250-349 = 1pt, 150-249 = 2pt, < 149m = 3pt

_______________________________________
PC:
58 yo M presents with 1 week of pleuritic chest pain,
fever, chills, and cough with purulent yellow sputum.
He is a heavy smoker with COPD.

---
1o - typical pneumonia
---
ddx:

V -
I - COPD exacerbation (bronchitis), lung abscess, TB
N - lung cancer
D -
I -
C - pericarditis
A -
T -
E -
M -
S -
---
OE:
- cough +/- sputum
- fever
- pleuritic chest pain
- SOB, tachypnoea, tachycardia
- consolidation (dull percussion, crackles)
- AMS

---
ix:
Bedside
O2 sats
UA
Labs
CBC + diff
UEC
ABG
trop/CK
LFT
Cultures - sputum gram stain and culture
CXR -
lobar = S pneunomiae
interstitial = PCP, viral, atypical (chlamydia, mycoplasma, legionella)
PFT
CT-chest
PPD

---
mx:
1o - ABC, O2, IVF, SABA
2o - CURB 65 > 1 (confusion, Urea > 7 or BUN > 20, RR > 30, BP <90/60, Age >
65
3o -
. CAP =
outpt - azithromycin (macrolide) and ADD ceftriaxone if comorbid
(CHF, CKD, CLD, DM, etoh, Ca, asplenic, immunocompro) or abx used within 3 mo
inpt - CURB65 > 1 = Ceftriaxone + Azithromycin
. HAP =
pseudomonas ? = Group A drug + Group B + Group C(MRSA) + Group
D(legionella)
Group A for antipseudomonas:
- cephalosporin
(cefepime/ceftazidime), or
- carbapenem (imipenem/mero), or
- piperacillin/tazobactam
Group B drug for other bug:
- fluoroquinolone
(ciprofloxacin/levofloxacin)
- aminoglycoside (amikacin,
gentamicin, tobramycin)
Group C drug for anti MRSA
- linezolid
- vancomycin
Group D drug for anti-legionella
- need at least a macrolide or
fluoroquinolone
4o - pneumovax + influenza vax
_______________________________________
PC:
25 yo F presents with 2 weeks of nonproductive
cough. Three weeks ago she had a sore throat and a
runny nose.

---
1o - atypical pneumonia
---
ddx:

V -
I - post nasal drip
N -
D -
I -
C -
A -
T -
E -
M -
S - GERD
---
OE:

---
ix:
CBC
sputum gram staina nd culture
CXR
IgM for mycoplasma pneumoniae
urine legionella antigen
---
mx:
ABCs, O2, IVF
CURB65 admit ?
atypicals = azithromycin or doxycycline or fluoroquinolone
_______________________________________
PC:
65 yo M presents with worsening cough for the past
6 months accompanied by hemoptysis, dyspnea,
weakness, and weight loss. He is a heavy smoker.

---
1o - lung cancer
---
ddx:

V -
I - TB, lung abscess, COPD, vasculitis/wegeners
N -
D - ILD
I -
C - CHF
A -
T -
E -
M -
S -
---
OE:
dry cough
dyspnoea
chest pain
hemoptysis
clubbing
constitutional

mets - pleural effusion, atelectasis, wheeze, dysphagia/hoarse, SVC syndrome,


horners
MSK - NSCLC - club, HPOA
Derm - bronchogenic Ca - Acanthosis, dermatomyositis
Endocrine - SCLC - SIADH, cushing
- Sq CC - hyperCa, hypoPO4
Neuro - SCLC - Lambert-eaton, polymyositis, spinocerebellar, peripheral neuro
Vasc - NSCLC - migratory thrombophlebitis, DIC
---
ix:
sputum cytology
central (squamous, small cell) = bronchoscopy
bronchoscope negative OR peripheral (adenocarcinoma*, large cell) = needle biopsy
effusion cytology
mediastinoscopy

broader:
CBC
sputum gram stain culture and cytology
CXR
CT-chest
PPD
ANCA
bronchoscopy
echocardiography
---
mx:
non-resectable = met, pleura(effusion), + other lung, main stem bronchus /carina
- baseline PFT
- palliative RadTx
resectable
- baseline PFT
- NSCLC - RadTx + CAP (cyclcophos, adriamycin, platinum)
- SCLC - VP16 (etoposide and platinum)
_______________________________________
PC:
34 yo F nurse presents with worsening cough of 6
weeks duration accompanied by weight loss, fatigue,
night sweats, and fever. She has a history of contact
with tuberculosis patients at work.

35 yo M presents with shortness of breath and cough.


He has had unprotected sex with multiple sexual
partners and was recently exposed to a patient with
active tuberculosis.

---
1o - Tb
---
ddx:

V - CHF, vasculitis
I - Tb, pneumonia (JVC), bronchitis, acute HIV, COPD
N - lymphoma, mets
D -
I - post nasal drip
C -
A - asthma, pulmonary fibrosis, sarcoid
T -
E -
M -
S -
---
OE:
mycobacterium tuberculosis
RF - immigrant, etoh, healthcare worker, prison, homeless, nursing home, immune
suppressed
- productive cough/hemoptysis, sputum, fever, wt loss, night sweats, adenopathy,
meningitis, pericarditis, hepatitis, peritonisis, osteomyeltis (vertebral = potts
disease)
extra-lung: adenitis (LN)***, meningitis, GI, GUT
---
ix:
- CBC (infection, anemia)
- PPD/INFG
- CXR
- Bronchoscopy
- HIV antibody
- LN biopsy
- Dx Tb
1o - CXR - infiltrates/vacitation, effusion, calcified nodules, lwr
lobe = primary, apical = secondary, hilar adenopathy
2o - 3 x sputum stain (acid fast bacilli)

Sn - pleural biopsy
Sp - sputum culture + sensitivities
- other

- Dx latent ix:
- either (+)PPD or (+)INF-G but (-) CXR = give INH for 9 mo
- PPD > 5 mm = (+) in hi risk groups (immunosuppressed, close contact)
- PPD > 10 mm = (+) in risk group
- PPD > 15 mm = (+) in low risk
---
mx:
- Reportable disease
- Latent (+)PPD/INF-G but (-)CXR = 9 mo INH (isoniacid) + B6 (pyridoxine vs
neuropathy of INH)
- Active = 2 mo x RIPE + 4 mo RI
.RIPE = rifampin, isoniazid, pyrazinamide, ethambutol
.RI = rif, isoniazid
- MDR Tb =
1o - streptmycin/kanamycin
2o - levofloxacin

_______________________________________
PC:
50 yo M presents with a cough that is exacerbated by
lying down at night and improved by propping up on
3 pillows. He also reports exertional dyspnea.

---
1o - systolic CHF (HFrEF, ischemic cardiomyopathy, dilated cardiomyopathy)
---
ddx:

V - CHF, vulvular disease


I - COPD, post nasal drip
N - pulmonary fibrosis, GERD
D -
I -
C -
A -
T -
E -
M -
S -
---
OE:
dyspnoea
orthopnoea, PND
dependent edema
JVP, hepatojugular reflex
S3
tachycardia
tachypnoea
Apical displace (cardiomegaly)
pulmonary edema (crackles, frothy white/blood speckled sputum)
CXR - cardiomegaly, pulmonary edema (transudate)
Echo - dilated ventricles, EF< 45%

Precipitating = FAILURE (forgot meds, arryhtmia/anemia, ischemia/infarct,


lifestyle/salt, upreg (pregnancy, hyperthyroid), renal failure, embolism
---
ix:
1o - ECG + CXR
2o - BNP > 500
3o - 2d echo = single best test for CHF
4o - mx precipitation

---
mx:
1o - ACEI (captopril, enalopril, lisinopril) or ARB (or hydralazine +
isosorbide dinitrate)
2o - BB (metoprolol, carvedilol, bisoprolol)
3o - if AF - start digoxin + warfarin (bridging heparin)
4o - spironolactone (eplerenone if painful gynecomastia)
5o - AICD if EF<35%, Biventricular pacing if QRS > 120 ms
_______________________________________
PC:
60 yo M presents with worsening dyspnea of 6 hours
duration and a cough that is accompanied by pink,
frothy sputum.

---
1o - pulmonary edema
---
ddx:

V - pulm edema, CHF, MS, arrhythmia,


I - pneumonia
N -
D -
I -
C -
A - asthma
T -
E -
M -
S -
---
OE:
pulmonary edema (crackles, frothy white/blood speckled sputum)
dyspnoea
orthopnoea, PND
tachypnoea
chest pain
dependent edema
JVP, hepatojugular reflex
S3
tachycardia

Apical displace (cardiomegaly)

CXR - cardiomegaly, pulmonary edema (transudate)


Echo - dilated ventricles, EF< 45%
---
ix:
bedside:
ECG
lab:
ABG
CBC
EUC
ALT/AST
bilirubin
BUN
Trop, BNP
cxr: interstitial edema, cardiomegaly, infiltrates
echo: ventricular dilatation, reduced ejection fraction
PFT
---
mx:
ABCs + LMNOP
lasix (furosemide)
morphine
nitroglycerin (decr preload)
O2
position upright
_______________________________________
PC:
60 yo M presents with sudden onset of substernal
heavy chest pain that has lasted for 30 minutes and
radiates to the left arm. The pain is accompanied by
dyspnea, diaphoresis, and nausea. He has a history of
hypertension, hyperlipidemia, and smoking.

---
1o - MI
---
ddx:

V - angina, pulm embolism, pneumothorax, aortic dissection


I - pericarditis,
N - GORD
D -
I -
C -
A -
T -
E -
M - costochondritis
S -
---
OE:
substernal, radiating jaw/shoulder
dizzy, N/V
diaphoresis
SOB
---
ix:
bedside - ECG - ST elevation 2 contiguous leads, new LBBB
Labs - trop x 3, CKMB, Ddimer, CBC, EUC,
CXR, echo, CT
cardiac catheterisatin

---
mx:
MONA + IV access (consider BB, statin, tirafiban)
tPA if >90 min to PCI or <12 hr since onset
PCI if <90 min to PCI
CABG if > 12 hr
_______________________________________
PC:
20 yo African American F presents with acute onset
of severe chest pain for a few hours. She has a history
of sickle cell disease and multiple hospitalizations for
pain and anemia management.

---
1o - sickle cell crisis
---
ddx:

V - pulmonary embolism, MIA, pneumothorax, dissection


I - pneumonia
N -
D -
I -
C -
A -
T -
E -
M -
S -
---
OE:
THIS IS AN EMERGENCY due to vaso-occlusion

precip - hypoxia, dehydration, acidosis, infection, fever

acute chest - (neg pulm cultures, severe chest pain, fever, leukocytosis, hypoxia,
infiltrates on CXR)

chronic: hematuria, ulcerations, bilirubin gallstones, AVN femoral head,


osteomyelitis (salmonella), retinopathy,
---
ix:
CBC + retic + PBS
LDH
ABG
d-dimer
CXR
trop + CK-MB
ECG
CTA - chest with contrast
---
mx: amoi
1o - ABCs

2o - IVF
Analgesia
O2
Abx for infection (asplenia) - ceftriaxone for pneumococcus, HiB

3o - echange transufsion - 5 units via pheresis - THIS SHOULD BE FIRST STEP IN


EMERGENCY

4O - chronic - folic acid, genetic counselling, BM HSCT, vax


_______________________________________
PC:
45 yo F presents with a retrosternal burning sensation
that occurs after heavy meals and when lying
down. Her symptoms are relieved by antacids.

---
1o - GERD
---
ddx:

V - MI, angina
I - esophagitis, peptic ulcer dz
N - esophageal spasm
D -
I -
C -
A -
T -
E -
M -
S -
---
OE:

sore throat
heart burn
acid taste in morning
recent started CCB
wheeze

worse at night/supine
chronic cough (aspiration)
dysphonia/hoarseness (vocal cord irritation)

assoc: etoh, peppermint, choco, CCB, nitrate, tight garments, pregnancy, heavy meal
bf bed

red flags: dysphagia, odynophagia, wt loss, blood stool, anemia


---
ix:
1o - clinical + f/u response to tx
2o - pH monitoring - gold std
3o - endoscopy if red flags
---
mx:
1o - lifestyle: avoid smoking, etoh, caffeine, wt loss, late night meals, use
elevated bed
2o - omeprazole PPI
3o - nissen fundoplication
_______________________________________
PC:
55 yo M presents with retrosternal squeezing pain
that lasts for 2 minutes and occurs with exercise. It
is relieved by rest and is not related to food intake.

---
1o - chronic stable angina
---
ddx:

V - stable angina
I - esophagitis
N - esophageal spasm
D -
I -
C -
A -
T -
E -
M -
S -
---
OE:
pain on exertion relieved by rest/nitrates
hx CAD risk factors - smoking, dm, +fhx, hyperlipidemia, htn, obesity
O2 supply risk factors - COPD, anemia

[esophagitis - pain on swwallowing only - HIV/candidiasis (tx fluconazole,


endoscopy], caustic pills (alendronate, quinine, risedronate, vit C, KCl,
doxycycline, NSAID, iron sulfate - tx - upright + water)
[esophageal spasm - dysphagia + pain not assoc with swallowing or exertion - Dx w
barium (corkscrew), Sp = manometry - Mx - CCB nifedipine)

---
ix:
bedside: ECG
Labs : Hb (anemia), fBSL, fLipid
CXR: pulm dz, heart failure
echo
stress testing*
---
mx:
1o - lifestyle CAD RFs - smoking, DM, obesity, lipids
2o - SAND BARC =
S. statin / smoking - HDL > 40, LDL < 100, total chol < 200
A. aspirin
N. nitrogylcerin
D. DM control < 7
B. beta blockers
A. ACEI
R. ranolazine (refractory)
C. CCB (2nd line to BB)
_______________________________________
PC:
34 yo F presents with retrosternal stabbing chest pain
that improves when she leans forward and worsens
with deep inspiration. She had a URI 1 week ago.

---
1o - pericarditis
---
ddx:

V - aortic dissection, MI
I - pericarditis
N - GERD, esophageal spasm
D -
I -
C -
A - dressler
T -
E -
M - costochondritis
S -
---
OE:
CARDIAC RIND: collagen/marfans, aortic dissection, radiation, drugs, infections,
AKI, cardiac MI, rheumatic fever, injury, neoplasm, dressler syndrome
Viral * - coxsackie
---
ix:
Dx 3 = chest pain, friction rub, ECG
ECG - PR depression, concave ST elevation on all/most leads
CXR - pulm dz
Echo - effusion
---
mx:
treat etiology
1o - ASA, NSAID, corticosteroids, colchicine
_______________________________________
PC:
33 yo F presents with stabbing chest pain that worsens
with deep inspiration and is relieved by aspirin.
She had a URI 1 week ago. Chest wall tenderness is
noted.

---
1o - costochondritis
---
ddx:

V - MI, embolism
I - pneumonia, pericarditis
N -
D -
I -
C -
A -
T -
E -
M - muscle strain
S -
---
OE:

---
ix:
ECG
CK-MB, troponin
CXR
CBC
---
mx:
Reassurance, advice to followup or reappear if worsen or red flags
NSAIDS (check of allergies)
_______________________________________
PC:
70 yo F presents with acute onset of shortness of
breath at rest and pleuritic chest pain. She also presents
with tachycardia, hypotension, tachypnea, and
mild fever. She is recovering from hip replacement
surgery.

---
1o - pulmonary embolism
---
ddx:

V - MI, CHF, dissection


I - pneumonia
N -
D -
I -
C -
A -
T -
E -
M - costochondritis
S -
---
OE:
dyspnoea, pleuritic chest pain, hemoptysis, tachypnoea, cyanosis, hypoxia,
fever
---
ix:
wells score

if Wells = Lo (0-2)
1o - D-dimer (neg = ruled out) - but if pos do CTPA
2o - CTPA (pos = ruled in)

if Wells > Lo
1o - CTPA

ABG
CK-Mb, troponin
CBC
EUC, BUN/Cr, BSL
Dx/Sp = CT-PA (pulm angiogram is gold standard)
ECG = sinus tachy + RV strain + S1-Q3-T3inv
CXR = atelecatsis/collapse, effusions
V/Q scan (hi Sn) = do it if there is no COPD and CXR normal or contr to CT
Venous dopplers
baseline INR

---
mx:
1o - anticoagulation 6 mo if unprovoked (3mo if transient)- LMWH eg enoxaparin 1.5
mg/kg SC q24hr + warfarin started at same time 5mf PO OD + baseline INR; IVC filter
if anticoag contraindicated
2o - thrombolysis if > moderate PE (eg massive) or compromise/shock
3o - admit if unstable, requiring O2, comorbiditis, lack social supports, unable to
ambulate

Wells criterion:
clinical DVT
immobilisation or surgery within 1 mo
previous PE/DVT
HR > 100
hemoptysis
malignancy

PERC = PE rule out criteria = rules out if all met


<50yo
HR<100
O2sat>95
no hemoptysis
no E2
no prev DVT/PE
no unilateral leg swell
No surg/trauma req hospital w/in last 4 wk

_______________________________________
PC:
55 yo M presents with sudden onset of severe chest
pain that radiates to his back. He has a history of
uncontrolled hypertension.

---
1o - aortic dissection
---
ddx:

V - dissection, MI, pericarditis, fat embolus


I - pancreatitis
N - GORD
D -
I -
C -
A -
T -
E -
M - esophageal rupture or spasm
S -
---
OE:
acute onset
severe tearing retro-sternal
midscapular pain
focal neurological loss
strong history of HTN

pulse deficit or sBP difference bw arms


AR
---
ix:
CT-A
CXR - widened mediastinu
Left pelural effusion
Indistinct aortic knob
ECG (LVH, ischemic, pericarditis, heart block)
Bloods:
- FBC
- XM
- lactate (ishcemic gut, shock)
- amylast (rule out pancreatitis)
- troponin (rule out MI)
---
mx:
ABCs
reduce BP and HR
BB (2nd ACEI)

classify type w CTA


COnsult vascular
_______________________________________
PC:
70 yo diabetic M presents with episodes of palpitations
and diaphoresis. He is on insulin.

---
1o - hypoglycemia
---
ddx:

V - arrhythmia, angina
I -
N - carcinoid syndrome, pheochromoyctoma
D -
I -
C -
A -
T -
E - hyperthyroid
M -
S - panic attack, hyperventilation
---
OE:
(whipples triad)
- low plasma glucose < 2.5 mmol/L
- sx (HA, confusion, seizure, coma, diaphoreis, nausea, hunger, tachycardia,
palpitations)
- resolution with glucose
Hx:
-last meal, prev episodes, meds, compliance
-comorbid liver, renal, endocrine, neoplastic
-depression, alcohol, durg

ETI:
- EXCESS INSULIN
-etoh intox
- sepsis
- liver dz
- oral anti-hyperglycemics
- insulinoma
- hypopituitarism
- adrenal insuff

---
ix:
- BSL
- EUC
- Cr
- LFT
- drug screen
- cortisol
- blood alcohol level
---
mx: ACUTE
IV access and rapid blood glucose measurement
D50W 50 mL IV push, glucose PO if mental status permits
if IV access not possible, glucagon 1-2 mg IM, repeat x 1 in 10-20 min
O2, cardiac, frequent blood glucose monitoring
thiamine 100 mg IM
full meal as soon as mental status permits
if episode due to long-acting insulin, or sulfonylureas, watch for
prolonged hypoglycemia due to
long t1/2 (may require admission for monitoring)
search for cause (most often due to exogenous insulin, alcohol, or
sulfonylureas)

mx: AWAKE pt
- 15 g carbohydrates (3 packets sugar dissolved in water or 1 cup juice)
- wait 15 min
- retest BSL
- repeat CHO/15min/BSL cycle until BSL > 5
- educate on hypoglycemia
_______________________________________

PC:
35 yo M presents with several episodes of palpitations,
sweating, and rapid breathing. Episodes occur
unexpectedly, and he does not recall any triggers. He
has had 45 episodes per month for several months.
Each episode lasts 23 minutes. He does not have
any history of psychiatric illness except for separation
anxiety as a child.

---
1o -
---
ddx:

V -
I -
N -
D -
I -
C -
A -
T -
E -
M -
S -
---
OE:

reaches peak in minutes


anxiety about repeat attacks

Criteria for Panic Disorder (>4)


STUDENTS FEAR the 3 Cs
Sweating
Trembling
Unsteadiness, dizziness
Depersonalization, Derealization
Excessive heart rate, palpitations
Nausea
Tingling
Shortness of breath
Fear of dying, losing control, going crazy
3 Cs: Chest pain, Chills, Choking
---
ix:
CBC
TSH
EUC
UA
Urine drug screen
neuro consult
CXR
CT-B
---
mx:
CBT
SSRI fluoxetine
short term benzo if needed
_______________________________________
PC:
19 yo F presents with episodic palpitations, especially
during presentations in front of her class. Episodes
include heart pounding, facial blushing, and hand
tremor. She also experiences excessive sweating and
rapid breathing. She complains of intense worry and
trouble sleeping for days or weeks before an upcoming
social situation. Now she avoids all social events
because she is afraid of humiliating herself.

---
1o -
---
ddx:
Social phobia
Avoidant personality
disorder
Agoraphobia/specific
phobia
Panic attack
Generalized anxiety
disorder
Substance abuse/
dependence
Hyperthyroidism

---
OE:

> 6mo
fear of social/performance, being scrutinised
fear of acting in way that will humiliate, embarass (public speaking, starting
conversation, dating, eating in public)

---
ix:
CBC
Electrolytes
ECG
Echocardiography
TSH, FT4
Mental status exam
---
mx:
avoid caffeine, alcohol
sleep hygiene
CBT
SSRI fluoxetine
BB in acute situations - public speaking
_______________________________________
PC:
34 yo F presents with episodic palpitations accompanied by lightheadedness and
sharp, atypical chest pain.

---
1o - MVP
---
ddx:

V - arrhythmia
I -
N - pheo
D -
I -
C -
A -
T -
E -
M -
S - panic attack
---
OE:
- mid-systolic click = louder with valsalva or on standing (decr venous
return)
- marfan, pectus excavatum
- stabing CP, SOB, anx/panic, palpitation, fatigue
---
ix:
- ECG (parox SVT)
- Echo (displaced leaflet)
- holter monitor
- 24 hr urinary catecholamines
---
mx:
- BB
- avoid stimulants, caffeine
- rivaroxaban for AFib
- surgical repair/consult
_______________________________________
PC:
42 yo F presents with a 15.5-lb (7-kg) weight loss
within the past 2 months. She has a fine tremor, and
her pulse is 112.

---
1o - hyperthyroid (graves, toxic multinodular goitre, toxic adenoma, subacute
thyroiditis, iodine, thyroxine, hCG pregnancy stimulation)
---
ddx:

V -
I - HIV
N - cancer
D - diet/drugs, malabsorption
I -
C -
A -
T -
E -
M -
S - anorexia
---
OE:
"hyper THYROIDISM"
-tremor
-heart rate up, AFib
-yawning (fatigued)
-restlessness
-oligomenorrheoa/amenorrhea
-intolerance to heat
-diarrhoea
-irritable
-sweat
-muscle wasting - PROXIMAL mm /weight loss - OSTEOPOROSIS

other: Fine hair, skin moist and warm, vitiligo, soft nails with onycholysis
(Plummers nails), palmar erythema, pruritis

Graves:
- leukopenia, lymphocytosis, splenomegaly, lymphadenopathy (occasionally)
- lid lag, retraction, proptosis, diplopia/extraocular, decreased acuity,
perioribital puffiness, conjuctival injection, corneal abrasions, sight loss
- clubbing (acropachy), pretibial myxedema (rare)
---
ix:
TSH (inc = secondary, decr = primary_
fT4 (incr = secondary + primary)
Abs - thryoid stimulating Ig
Radio uptake (incr = graves, multinodular goitre, toxic adenoma), (de r = subacute
thryoiditis, iodine load, exogenous thryoid hormones)
Radioisotope thyroid scan = (graves = diffuse; mutlinodular = heterogenous; toxic
adenoma = nodal uptake)
---
mx:
1o - methimazole (thionamides inhibit thyroid peroxidase) -
.contra in pregnancy (use propylthiouracil)
.use PTU for thyroid storm
2o - sx
. BB - tachy, tremor
.eyes: smoking cessation, eye drops, prendnisone, surgical decompression
3o - radioactive iodine thyroid ablation
4o - thyroidectomy (hemi/complete)

Thyroid storm:
1o - hi dose PTU
2o - after PTU - give Sodium Iodide / lugol / potassium iodide/SSKI after 1 hr
(inhibit T3/4 release
3o - dexamethasone - (lower body temperature and T3 to 4 conversion)
_______________________________________
PC:
44 yo F presents with a weight gain of > 25 lbs (11.3
kg) within the past 2 months. She quit smoking 3
months ago and is on amitriptyline for depression.
She also reports cold intolerance and constipation.

---
1o - smoking cessation
---
ddx for wt gain

V -
I -
N -
D -
I -
C -
A -
T -
E - hypothryoid, PCOS, DM
M -
S -
---
OE:
habits
previous attempts
---
ix:
CBC
UEC
BSL
TSH
24 hr urine free cortisol
Dex suppression test
---
mx:
5As -
. ask if they smoke
. advise to quit
. assess willingness
. assist = STAR to quit plan = set quit date, tell family, anticiate
withdrawal/temptation, remove smoking related products
. arrange follow up
Combined counselling + smoking cessation
.NRT
.buproprion (NE+DA reuptake inhibitor, - also nicotine antagnoist) - contra =
reduces seizure threshold, eating disorder
.varenicline (partial nicotinic agonist - reduces cravings and response to
nicotine) - avoid in depression, suicide
Educate on withdrawal sx - lo mood, insomnia, irritable, anxiety, concentration,
resltless, decr HR, incr app
Unwilling to quit ? = 5Rs
- relevant to pt - children at home
- risks -
.short-term: SOB, asthma exacerbation, impotence, infertility,
pregnancy complications, heartburn, URTI
.long-term: MI, stroke, COPD, lung CA, other cancers
.environmental: higher risk in spouse/children for lung CA, SIDS,
asthma, respiratory infections
- rewards - health, money, food
- roadblocks - withdrawal, wt gain, failure
- repetition - its ok to try more than once

_______________________________________
PC:
30 yo F presents with weight gain over the past 3
months. She also reports tremor, palpitations, anxiety,
and hunger that is relieved by eating. She exhibits
proximal muscle weakness and easy bruising.

---
1o - insulinoma
---
ddx:

V -
I -
N -
D -
I -
C -
A -
T -
E - postprandial hypoglycemia, cusshing syndrome
M -
S -
---
OE:
whipples triad: palpitations, trembling, diaphoresis, confusion, seizure,
personality changes, weight gain
---
ix:
EUC, Cr, LFTs, toxins/drugs, cortisol
Glucose, serum ketone, insuline, pro-insulin, C-peptide, insulin antibodies
BSL
insulin levels
GTT
24 hr cortisol

USS
CT
---
mx:
pancreatectom, pancreaticoduodenectomy

_______________________________________
PC:
75 yo M presents with dysphagia that started with
solids and progressed to liquids. He is an alcoholic
and a heavy smoker. He has had an unintentional
weight loss of 15 lbs (6.8 kg) within the past 4
months

---
1o - esophageal cancer
---
ddx:

Esophageal cancer
Achalasia
Esophagitis
Systemic sclerosis
Esophageal stricture
ALS
---
OE:
dysphagia progressive - solids to liquids
wt loss

Key Questions in Dysphagia


Difficulty in starting swallowing?
Associated symptoms? (regurgitation,
change in voice pitch, weight loss)
Solids, liquids, or both?
Intermittent or progressive?
History of heartburn?
Change in eating habits/diet?

regurgitation and aspiration (aspiration pneumonia)


hematemesis, anemia
direct, hematogenous, or lymphatic spread
trachea (coughing), recurrent laryngeal nerves (hoarseness, vocal paralysis),
aortic, liver, lung, bone,
celiac and mediastinal nodes
---
ix:
1o - barium swallow
2o - biopsy endoscope
3o - biopsy LN

Metastatic workup:
. CXR
. bone scan
. CT head
. CT chest/abd/pelvis
. LFT
. PET

Bloods:
. FBC
---
mx:
Surgery

_______________________________________
PC:
45 yo F presents with dysphagia for 2 weeks accompanied
by mouth and throat pain, fatigue, and a
craving for ice and clay.

---
1o - Plummer-Vinson
---
ddx:

Plummer-Vinson syndrome
Esophageal cancer
Esophagitis
Achalasia
Systemic sclerosis
Mitral valve stenosis
---
OE:
iron deficiency anemia
crave ice, clay
---
ix:
1o - barium swallow
---
mx:
Iron supplements
Dilation procedure
_______________________________________
PC:
48 yo F presents with dysphagia for both solids and
liquids that has slowly progressed in severity within
the past year. It is associated with difficulty belching
and regurgitation of undigested food, especially
at night. She has lost 5.5 lbs (2.5 kg) in the past 2
months.

---
1o - Achalasia
---
ddx:

Achalasia
Plummer-Vinson
syndrome
Esophageal cancer
Esophagitis
Systemic sclerosis
Mitral valve stenosis
Esophageal stricture
Zenkers diverticulum
---
OE:
Achalasia = failure of SM relaxation at LES
Can be 2nd to malignancy, chagas
. constitutional
. trip to south america
---
ix:
CXR - no air in stomach + dilated esophagus
Dx = Barius studies - esophagus terminates in birds beak
Endocscopy - done if alarm sx ( > 60yo, anemia, FOB, >6mo, wt loss)
Sp = manometry
---
mx:
Ballon dilatatation
PPI
Botox ?
_______________________________________
PC:
38 yo M presents with dysphagia and pain on swallowing solids more than liquids.
Exam reveals oral thrush.

---
1o - Esophagitis (CMV, HSV, HIV, pill-induced)
---
ddx:

Esophagitis (CMV, HSV,


HIV, pill-induced)
Systemic sclerosis
GERD
Esophageal stricture
Zenkers diverticulum
---
OE:
hx hiv

hx aledronate, corticosteroids
---
ix:

---
mx:
CBX + differential + HIV ELISA + VIRAL LOAD, cd4 COUNT
fluconazole
endoscopy biopsy

_______________________________________
PC:
39 yo F presents with a single 2-cm mass on the
right side of her neck along with night sweats, fever,
weight loss, loss of appetite, and early satiety. The
mass is painless and movable and has not changed
in size. She does not report heat intolerance, tremor,
palpitations, hoarseness, cough, difficulty breathing,
difficulty swallowing, or abdominal pain. Her husband
was recently discharged from prison, and her
mother has a history of gastric cancer.

---
1o - Hodgkins/non- Hodgkins lymphoma
---
ddx:

Hodgkins/non- Hodgkins lymphoma


Tuberculosis
Thyroid nodule
Gastric carcinoma
---
OE:
LN-pathy
Constitutional

Hodgkins:
LNpathy - non tender, rubber
HS-megaly,
pruritus, systemic

NON hodgkins: [follicular, diffuse large B cell, Burkitt, mantle cell]


immunodeficiency - HIV
autoimmune disease - SLE
infections - EBV

HS-megaly, oropharyngeal/sorethroat/OSA
pancytopenia
---
ix:

Hodgkins = reed steinberge cells = classically painless, non-tender, firm, rubbery


enlargement of superficial lymph nodes, most often in the cervical region
Labs:
CBC with differential - anemia, eosinopilia, lymphopenia, incr
platelets
Biochem:
.HIV serology
.LFTs (liver involvement)
.KFT (baseline before chemo)
.ALP, Ca (bone involvement)
.ESR + LDH disease progression
.PPD/tb
.TSH
Image:
neck USS
CT-chest, CXR - LN, mediastinal mass
CT-abd - liver/spleen
PET scans
cardiac function (pre-chemo) - angiography
PFTs
Biopsy
FNA
endoscopy

Non-hodgkin = NO reed-steinberg = both B or T cell progenitors


Labs:
CBC - normochromic normocytic anemia, pancytopenia
Smear = lymphoma cells
Flow cytometry
Biochem:
.incr uric acid
.LFTs in liver emts
.rapidly incr LDH
stage:
CT neck, chest, abd, pelvis
BM Bx
Monitor progression: PET
Dx:
Excisional LN biopsy
BM biopsy

Ann Arbor staging


1. 1 x LN
2. same side diaphragm - 2+ or extra lumphatic
3. both sides diaphragm
4. diffuse + BM

t(2;5) ALK1 mutation Anaplastic


large cell lymphoma
t(8;14) c-myc activation Burkitts lymphoma
t(14;18) bcl-2 activation Follicular lymphoma
t(11;14) Overexpression of cyclin D1 protein Mantle cell lymphoma
t(11;18) MALT1 activation MALTOMA

---
mx:
Hodgkins: - Treatment of HL depends on stage; treatment of NHL depends on
histologic subtype
1o = St 1-4 = ABVD + RadTx [adriamycin, bleomycin, vinblastine, dacarbazine]
2o = Chemo ablation + Autologous stem cell transplant
Monitor response with PET scans

Poor prognosis = low albumin (liver involvement), low Hb (BM involvement),


Male, Stage 4, age > 45yo, leukocytosis (WBC>1.5x10^9), lymphocytopenia

NON-hodgkins:
local - RadTx + adjv chemo
Indolent - Radtx local + chemo (rituximab + bendamustine)
Aggressive - R-CHOP (rituximab, cyclophosphamide, hydroxyrubicin =
adriamycin, oncovin = vincristine, prednisone
.CNS prophylaxis with MTX
.autologous SCT

Monitor for complications


-hypersplenism
-infection
-hemolytic anemia
-bowel perforation
-tumor lysis syndrome = IV hydration,allopurinol, dialysis (for hyper
K)
_______________________________________
PC:
20 yo F presents with nausea, vomiting (especially in
the morning), fatigue, and polyuria. Her last menstrual
period was 6 weeks ago, and her breasts are
full and tender. She is sexually active with her boyfriend,
and they occasionally use condoms for contraception.

---
1o - pregnancy
---
ddx:

Pregnancy
Gastritis
Hypercalcemia
Diabetes mellitus
UTI
Depression
---
OE:
amenorrhoea - nagals rule (LMP-3mo+7d)
N/V
breast tenderness
urinary frequency
fatigue
O+G hx
.GTPAL
Year
Sex
Weight
Gestational age (GA)
Mode of delivery
Length of labour
Complications
Abuse - safe at home
DM ?
---
ix:
bHCG
USS - pelvus

Bloods:
- FBC, ABO + Rh, Antibody screen,
- infection screening
. rubella,
. HBsAg,
. VDRL,
. Pap smear,
. gonorrhea/chlamydia,
. HIV,
. TB testing
. varicella or vaccination,
. parvovirus immunity if exposed to small children,
. cytomegalovirus
. toxoplasmosis serology
. pertussis vaccine
Smoking, alcohol, drug use, domestic violence
Pelvic exam - pap smear

CBC

UEC, CMP, Glucose


UA, urine culture - bacteriuria
HIV ab
---
mx:

_______________________________________
PC:
45 yo M presents with sudden onset of colicky rightsided
flank pain that radiates to the testicles, accompanied
by nausea, vomiting, hematuria, and CVA
tenderness.

---
1o - Nephrolithiasis
---
ddx:

V -
I - pyelonephritis, appendicitis
N - RCC
D -
I -
C -
A -
T -
E - Nephrolithiasis
M -
S -
---
HX:
- low fluid intake
- HOW MANY KIDNEYS **
- UTI (proteus, pseduo, providencia, klebsiella, mycoplasma, serratia,
SAureua
- myeloproliferative
- IBD
- gout
- DM
- hypercaclemia (PTH, tumor lysis syndrome, sarcoid)
- obesity
- meds: loops, acetazolamide, topiramate, acyclovir
- previous
- fhx+
OE:
- severe flank to groin/testes
- writhing, never comfortable
- N/V
- hematuria
- diaphoresis
- tachyC + tachyP
- fever ? (concurrent uti/pyelo, obstruction)
Eti:
- stasis, saturation, stone nidus
---
ix:
[KUB - calcium, struvite, cystine (cannot see urate)

0. - FBC
- Uric acid
- UA
- Urince mcs
- PTH, CMP EUC, oxalate, cystine
- Abd USS (rule out obstruction + follow up)
1. CT-abd/pelvis - can see all of them
2. KUB XR
3. urgent IF [one kidney, bilateral stones, P/V, AKI]
4. - Uric acid stone dissolution
- other = extracorporeal shock wave lithotripsy, stent, percutaneous
nephrostomy
---
mx:
0. admit if - intractable pain, intract vomiting, fever, compromised renal
function, pregnancy
1. morphine + ondansetron
2. NSAID (lower intra-uretral pressure)
3. phenoxybenzamine (alpha-blocker to incr rate of spontaneous passage of distal
stones)
4. Abx bacteriuria
5. intervention
1st - Ureteric stent via cytoscopy
2nd - USS percutaneous nephrostomy
6. prevention:
- incr fluid 2L/d
- reduce protein, oxalate, sodium, sugar intake
- avoid vit C supplemenet
- thiazide to prevente hypercalciuria
- allopurinol for hyperuricosuria
- potassium citrate for hypocitraturia

_______________________________________
PC:
60 yo M presents with dull epigastric pain that radiates
to the back, accompanied by weight loss, dark
urine, and clay-colored stool. He is a heavy drinker
and smoker. He appears jaundiced on exam.

---
1o - pancreatic cancer
---
ddx:

V - arch aneurysm
peptic ulcer dz
I - acute viral hepatitis (A,E)
cholecystitis
choledolithiasis

N - cholangiocarcinoma
D -
I -
C -
A -
T - alcoholic hepatitis, pancreatitis
E -
M -
S -
---
OE:
Trouseeau - peripheral venous thrombosis
weight loss
obstructive juandice
steattorhea
pain radiation to back
Courvorsier - palpable nontender distended gallbladder

---
ix:

FBC, EUC
Amylase, lipase
LFT - AST, ALT, bili, ALP
hi
ALP
Bilirubin
CA 19-9
USS + CT staging

---
mx:

_______________________________________
PC:
56 yo M presents with severe midepigastric abdominal
pain that radiates to the back and improves when
he leans forward. He also reports anorexia, nausea,
and vomiting. He is an alcoholic and has spent the
past 3 days binge drinking.

---
1o - Acute pancreatitis
---
ddx:

Peptic ulcer disease


Cholecystitis/
choledocholithiasis
Gastritis
Abdominal aortic
aneurysm
Mesenteric ischemia
Alcoholic hepatitis
Boerhaave syndrome

---
OE:
Gallstones + Ethanol
I GET SMASHED

Idiopathic: thought to be hypertensive sphincter or microlithiasis


Gallstones (45%)
Ethanol (35%)
Tumours: pancreas, ampulla, choledochocele
Scorpion stings
Microbiological
bacterial: Mycoplasma, Campylobacter, TB, M. avium intracellulare, Legionella,
leptospirosis
viral: mumps, rubella, varicella, viral hepatitis, CMV, EBV, HIV, Coxsackie
virus, echovirus,
adenovirus
parasites: ascariasis, clonorchiasis, echinococcosis
Autoimmune: SLE, polyarteritis nodosa (PAN), Crohns disease
Surgery/trauma
manipulation of sphincter of Oddi (e.g. ERCP), post-cardiac surgery, blunt trauma
to abdomen,
penetrating peptic ulcer
Hyperlipidemia (TG >11.3 mmol/L; >1000 mg/dL), Hypercalcemia, Hypothermia
Emboli or ischemia
Drugs/toxins
azathioprine, mercaptopurine, furosemide, estrogens, methyldopa, H2-blockers,
valproic acid,
antibiotics, acetaminophen, salicylates, methanol, organophosphates, steroids
(controversial)

eti: activation of trypsin -> enzymes -> inflammatory response

- Pain - severe - constant - thru to back + better with leadning forward (pancreas
is retroperitoneal so leaning forward removies pressure on them)
- rigid abdomen + guarding
- N/V
- distension (ileus)
- fever from inflamm response
- jaudince (compression/obstruction bile duct)
- ARDS
- tetany from hypocalcemia

---
ix:
CBC
- incr WBC
BSL
- incr glucose
CMP
- low calcium
Electrolytes, BUN/Cr
Amylase, lipase**
AST/ALT/bilirubin/alkaline phosphatase
- ALT>150 for biliary cause
U/Sabdomen (for biliary tree)
Dx = CTabdomen + contrast (calcification
Upper endoscopy
ECG
---
mx:
0. admit
1. fluid resusc
2. analgesia
3. O2
4. abx cephalosporine/imipenem if intection
5. aspirate necrotic collections
6. NG decompress stomach
7. nutritional support via nasojejunal feed or TPN
_______________________________________
PC:
41 yo obese F presents with RUQ abdominal pain
that radiates to the right scapula and is associated
with nausea, vomiting, and a fever of 101.5F. The
pain started after she ate fatty food. She has had similar
but less intense episodes that lasted a few hours.
Exam reveals a positive Murphys sign.

---
1o - Acute cholecystitis
---
ddx:

Choledocholithiasis
Hepatitis
Ascending cholangitis
Peptic ulcer disease
Fitz-HughCurtis syndrome
Appendicitis
---
OE:
Constant RUQ pain
anorexia
N/V
low grade fever
Murphy sign
right subscapular pain (Boas's)
---
ix:
CBC
AST/ALT/bilirubin/alkaline phosphatase
Dx - U/Sabdomen, HIDA scan if USS negative
CTabdomen
Blood culture
---
mx:
Admit
IVF
NG-tube decompression.
Abx - cefazolin
ERCP - prior to US for CBD
Cholecystectomy
_______________________________________
PC:
43 yo obese F presents with RUQ abdominal pain,
fever, and jaundice. She was diagnosed with asymptomatic
gallstones 1 year ago. She is found to be hypotensive
on exam.

---
1o - Ascending cholangitis
---
ddx:

Ascending cholangitis
Acute gallstone
cholangitis
Acute cholecystitis
Hepatitis
Sclerosing cholangitis
Fitz-HughCurtis
syndrome
---
OE: = infecton of biliary tree due to obstruction or stricture (Ecoli, KLebsiella,
Enterobacter, Enterococcus, Bacteroides, Clostridia)
- Charcot triad = RUQ pain, fever, jaundice
- Reynolds = charcot + hypotension + AMS

---
ix:
CBC
AST/ALT/bilirubin/alkaline phosphatase
Blood culture
Viral hepatitis serologies
U/Sabdomen
MRCP
ERCP
---
mx:
ERCP - drainage
Abx = Gneg = ampicillin/sulbactam, or piperacillin/tazobactam, or metro/ceftriaxone
_______________________________________
PC:
25 yo M presents with RUQ pain, fever, anorexia,
nausea, and vomiting. He has dark urine and claycolored
stool.

---
1o - acute hepatits
---
ddx:

Acute cholecystitis
Ascending cholangitis
Choledocholithiasis
Pancreatitis
Acute glomerulonephritis
---
OE:
-flu-like prodrome 2 wk
-N/V, anorexia, HA, fatigue, myalgia fever, arthralia, urticaria
Jaundice
Pale stools, dark urine
HS-megalt
RUQ pain
CV LN-pathy
---
ix:
CBC
Amylase, lipase
AST/ALT/bilirubin/alkaline phosphatase
Hepatitis serologies -
- HAV - IgM
- HBV-surface, core,
- HCV - RNA, IgG/IgM
- HDV - IgG/IgM
- HEV - IgG/IgM
- CMV - IgG/igM
- EBV - monospot IgM/IgG, DNA
- Yellow fever - IgG/IgM
LFTs - PT/INR, albumin, bilirubin
UA
U/Sabdomen
---
mx:
1. admit if - encephalopathy, coagulopathy, vomiting, hypoglycemia
2.
HAV - hygiene, vaccine, HA-Ig
HBV
- prevention = vaccine, HB-Ig
- current = entecavir
HCV
-IFN + ribavirin + protease inhibitor
IVF

_______________________________________
PC:
35 yo M presents with burning epigastric pain that starts 23 hours after meals.
The pain is relieved by food and antacids.

---
1o - PUD
---
ddx:
Peptic ulcer disease
Gastritis
GERD
Cholecystitis
Chronic pancreatitis
Mesenteric ischemia

---
OE:
NSAID (NSAID induced)
stress (stress induced)
Weight loss + night sweat + refractory to PPI (Zollinger Ellison)
Cigarette smoking (incr risk, impairs healing, complication)
Liver cirrhosis
COPD
CHronic renal failure
Hiatus hernia

Duodenal picture:
epigastric/xiphoid
burning
develops 1-3 h after meals
relieved by eating and antacids
interrupts sleep
periodicity (tends to occur in clusters over wk with subsequent periods of
remission)

Gastric picture:
Pain WITH eating
---
ix:
Vitals (BP, HR, postural)
CBC
EUC - BUN, Cr
PT/INR, PTT
Blood type + XM
fasting gastrin

Rectal exam
FOB
Amylase, lipase, lactate, AST/ALT/bilirubin/alkaline phosphatase
Upper endoscopy + biopsy for gastric carcinoma**** (Sp/Dx)
Urea breath test (Gneg, flagellated rod in gastric mucosa
Upper GI series
---
mx:
Acute:
IVF/ blood
NG decompression + aspiration
IV pantoprazole 80 mg + 8 mg/hr infusion
Erythromycin 250 mg 30 min before endoscopy
Long term:
Stop NSAID
PPI
HPylori mx - 14d clarithromycin 500 mg bid, amoxicillin 1 g BID,
lansoproazole 30 mg BID
Cease smoking
Avoid caffeine, alcohol, spices
_______________________________________
PC:
37 yo M presents with severe epigastric pain, nausea,
vomiting, and mild fever. He appears toxic. He has
a history of intermittent epigastric pain that is relieved
by food and antacids. He also smokes heavily
and takes aspirin on a daily basis.

---
1o -
---
ddx:

Perforated peptic ulcer


Acute pancreatitis
Hepatitis
Cholecystitis
Gallstone cholangitis
Mesenteric ischemia
---
OE:

---
ix:
Rectal exam
Vitals - Hr, BP + postural
CBC + type + XM
Electrolytes + BUN, Cr
fasting gastrin levels

Amylase, lipase, lactate


AST/ALT/bilirubin/alkaline phosphatase
CXR
KUB
CTabdomen
Upper endoscopy**
(including H pylori
testing)
Blood culture
---
mx:
NG decompression + aspiration
IV pantoprazole
Erythromycine
Endoscopy
_______________________________________
PC:
18 yo M boxer presents with severe LUQ abdominal pain that radiates to the left
scapula. He had infectious mononucleosis 3 weeks ago.

---
1o -
---
ddx:

Splenic rupture
Kidney stone
Rib fracture
Pneumonia
Perforated peptic ulcer
Splenic infarct
---
OE:
EBV hx
Kehrs sign = LEFT shoulder pain due to diaphragmatic irritation from splenic
rupture - worse with inspiration
Hx - blunt trauma
---
ix:
CBC + serial hematocrit levels
Electrolytes
CXR
CTabdomen
U/Sabdomen (if
hemodynamically
unstable)
---
mx:
bed rest
splenic artery embolisation once stable and if indicated (hemoperitoneum)
Splenectomy or splenorrhaphy

indications for splenectomy:


Splenic abscess/splenomegaly
Hereditary spherocytosis
Immune thrombocytopenic purpura
Rupture of spleen
Thrombotic thrombocytopenic purpura
Splenic vein thrombosis
_______________________________________
PC:
40 yo M presents with crampy abdominal pain, vomiting,
abdominal distention, and inability to pass flatus
or stool. He has a history of multiple abdominal
surgeries.

---
1o - Intestinal obstruction
---
ddx:
Intestinal obstruction
Small bowel or colon
cancer
Volvulus
Gastroenteritis
Food poisoning
Ileus
Hernia
---
OE:
Eti = intra-LUMEN, intra-MURAL, extra-MURAL
intra-LUMEN: intussusception, gallstone, bezoar
intra-MURAL: crohn, RadThx stricure, adenocarcinoma
extra-MURAL: surgical adhesion, incarcerated hernia

Top causes: adhesions, bulge/hernia, cancer

Causes SBO = SHAVING = stricture, hernia, adhesions, volvulus, intussusception,


neoplasm, gallstone

Sx = colicky abdominal pain, nausea/vomiting (proximal***), obstipation


SSX = distention, initially hyperactive howel sounds -> none,
Ischemia = pain disproprotionate to findings, tachy, localised tenderness, fever,
Luekocytosis, lactic acidosis

---
ix:
Rectal exam
CBC, HCt, Cr, EUC, ABG (met alkalosis + amylase + leukocytosis + lactate)
Electrolytes
AXR (dilated >3cm, air-fluid levels on upright, no air in distal
CTabdomen/pelvis with
contrast
Colonoscopy

ischemic bowel/strangulation: pneumatosis intestinalis (free air in bowel wall) &


thickened bowel wall, air in portal vein, free intraperitoneal fluids, differential
wall enhancements (poor uptake of IV contrast into the wall of the affected bowel)

---
mx:
IVF
monitor urine outpute + IDC
NG tube decompression
Surgical review
_______________________________________
PC:
70 yo F presents with acute onset of severe, crampy
abdominal pain. She recently vomited and had a
massive dark bowel movement. She has a history
of CHF and atrial fibrillation, for which she has received
digitalis. Her pain is out of proportion to the
exam.

---
1o -
---
Mesenteric ischemia/infarction
Diverticulitis
Peptic ulcer disease
Gastroenteritis
Acute pancreatitis
Cholecystitis
---
Hx - atherosclerotic disease, CVD RFs
severe abdominal pain out of proportion to physical findings,
vomiting,
bloody diarrhea,
bloating,
hypotension,
shock,
sepsis
post prandial pain

Typical: SMA area, watershed (splenic flexure, left colon, sigmoid colon)

Acute abdomen + met acidosis = bowel ischemia unless shown otherwise


---
ix:
Rectal exam
CBC, HCt, Cr, EUC, ABG (met alkalosis + amylase + leukocytosis + lactate)
PT/aPTT
Electrolytes
AXR (portal venous gas, intestinal pneumatosis, free air perforation)
CTabdomen/pelvis with
contrast (thick walls, dilatation, SMA thrombus, portal venous gas, pneumatosis)
Colonoscopy

ischemic bowel/strangulation: pneumatosis intestinalis (free air in bowel wall) &


thickened bowel wall, air in portal vein, free intraperitoneal fluids, differential
wall enhancements (poor uptake of IV contrast into the wall of the affected bowel)

---
mx:
IVF
NBM
NG tube decompression
Broad spectrum abx
exploratory lapartomy

_______________________________________
PC:
21 yo F presents with acute onset of severe RLQ pain, nausea, and vomiting. She has
no fever, urinary symptoms, or vaginal bleeding and has never taken OCPs. Her last
menstrual period was regular, and she has no history of STDs. She has been told
that she had a cyst on her right ovary.

---
1o -
---
ddx:

Ovarian torsion
Appendicitis
Nephrolithiasis
Ectopic pregnancy
Ruptured ovarian cyst
Pelvic inflammatory disease
Bowel infarction or perforation

Urinary tract: UTI, kidney stones


GI: diverticulitis, appendicitis
Gyne: endometriosis, PID, fibroid (degenerating, infarcted, torsion), ovarian
torsion, ovarian neoplasm, ovarian cyst, pregnancy-related
---
OE:

---
ix:
BHCG
CBC + diff, EUC, BSL, Cr, BUN, PTT/INR
Swabs
STI screen - chlamydia, gonorrhea testing, VDRL
USS pelvic, Abdomen
Dopper flow studies for ovarian torsion
Laparoscopy
CT abdomen
---
mx:
Analgesia
refer
admit for surgery, IV abx
_______________________________________
PC:
68 yo M presents with LLQ abdominal pain, fever, and chills for the past 3 days. He
also reports recent onset of alternating diarrhea and constipation. He consumes a
low-fiber, high-fat diet.

---
1o -
---
ddx:

Diverticulitis
Crohns disease
Ulcerative colitis
Gastroenteritis
Abscess
---
OE:
episodic LLQ pain
alternating bloating, flatulence, constipation, diarrhoea
Painless rectal bleeding
fever
leukocytosis
COMPLICATIONS: abscess, fistula, obstruction, perforation
---
ix:
AXR + upright CXR
CT + rectal contrast ****(pericolic fat, fluid, abscess, fistula)

---
mx:
UNCOMPLICATED = outpt - clear fluids + abx (cefazolin + metronidazole 7 d)
COMPLICATED: NBM, IVF, IV abx (ceftriazone, metronidazole) + surgery
SURGERY: hartmann (resection + colostomy + reversal in 3 mo + drain)
Recurrence 30%
_______________________________________
PC:
20 yo M presents with severe RLQ abdominal pain, nausea, and vomiting. His
discomfort started yesterday as a vague pain around the umbilicus. As the pain
worsened, it became sharp and migrated to the RLQ. McBurneys and psoas signs are
positive.

---
1o -
---
ddx:

Acute appendicitis
Gastroenteritis
Diverticulitis
Crohns disease
Nephrolithiasis
Volvulus or other
intestinal obstruction
Perforation
Acute cholecystitis
---
OE:
Murphy
Complications: abscess, stump leak
---
ix:
CBC - leukocytosis + neutrophils
BHCG
UA
Electrolytes
CTabdomen*** BEST (thick wall, appendicolith, inflamm)
AXR
U/Sabdomen
Blood culture
Colonscopy - rule out cancer
---
mx:
IVF
pre op abx - cefazolin + metronidazole
Laparoscopic Appendectomy
If complicated than abx + drain then inverval laparotomy
_______________________________________
PC:
30 yo F presents with periumbilical pain for 6 months. The pain never awakens her
from sleep. It is relieved by defecation and worsens when she is upset. She has
alternating constipation and diarrhea but no nausea, vomiting, weight loss, or
anorexia.

---
Irritable bowel syndrome
Crohns disease
Celiac disease
Chronic pancreatitis
GI parasitic infection
(amebiasis, giardiasis)
Endometriosis
---
OE:
ROME CRITERIA =
>12 wk in last 12 mo with 2 of 3 =
relieved by defecation
change in frequency of stool
change in consistency
sugestive = mucus, tenesmus, bloating

red flags ruling out:


wt loss
fever
nocturnal defecation
anemia
blood/pus in stool
abn flexible sigmoidoscopy
---
ix:
Rectal exam, stool for occult blood
Pelvic exam
Urine hCG
CBC + ESR/CRP
Celiac Tissue transaminase antibodies
Calprotectin - fecal (rule out IBD)
TSH
Electrolytes
Colonoscopy (for alarm features)
CTabdomen/pelvis
Stool for ova and
parasitology, Entamoeba
histolytica antigen
---
mx:
Reassurance + education
relaxation + biofeedback therapy
low FODMAP diet for pain, bloating, irregular bowel movement
Analgesia:
Anti-spasmodic - hyoscine
Incr diet (psyllium)
TCA
_______________________________________
PC:
24 yo F presents with bilateral lower abdominal pain that started with the first
day of her menstrual period. The pain is associated with fever and a thick,
greenish-yellow vaginal discharge. She has had unprotected sex with multiple sexual
partners.
---

---
ddx:

Pelvic inflammatory
disease
Endometriosis
Dysmenorrhea
Vaginitis
Cystitis
Spontaneous abortion
Pyelonephritis
---
OE:
chlamydia, gonorhea
fever, Lw Abd pain, discharge
Dyspareunia

Dx = lwr abd pain + cervical motion tenderness or adnexal tenderneaa + fever /


discharge / culture positive for gonorrhoea or chlamydia or ecoli / free flud /
leukocytosis / ESR CRP

PID Complications
I FACE PID
Infertility
Fitz-Hugh-Curtis syndrome
Abscesses
Chronic pelvic pain
Ectopic pregnancy
Peritonitis
Intestinal obstruction
Disseminated infection (sepsis, endocarditis, arthritis, meningitis)
---
ix:
bhcg (ectopic)
CBC (infecton)
Blood cultures
Urine MCS
Speculum
USS (abscess, free fluid)

---
mx:
Inpatient if - abscess, pregnant, first episode, ovarian torsion

Abx = clindamycin 900 mg IV q8h + gentamicin 2 mg/kg IV loading then 1.5 mg/kg IV
q8h maintenance
COntinue for 24 hr then doxycyclin 100 mg PO

percutaneous drainage of abscess with USS

Laparoscopic

OUTPT - if moderate
- ceftriaxone 250 mg IM x 1 + doxycycline 100 mg PO bid x 14 d
_______________________________________
PC:
67 yo M presents with alternating diarrhea and constipation, decreased stool
caliber, and blood in the stool for the past 8 months. He also reports
unintentional weight loss. He is on a low-fiber diet and has a family history of
colon cancer. His last colonoscopy was 12 years ago.

---
1o -
---
ddx:

Colorectal cancer
Irritable bowel syndrome
Diverticulosis
GI parasitic infection
(ascariasis, giardiasis)
Inflammatory bowel disease
---
OE:
hematochezia, melena
abd pain
changed bowel habits
changed stool gauge
constitutional
---
ix:
colonoscopy (or barium enema)
stool FOBT
CBC
UA
LFT - ast/alt/alp
Albumin + bilirubin
CEA
staging - CT chest/abdomen/pelvis/bone
---
mx:
wide surgical resection + LN drainage + primary anastaomsis + 5FU
_______________________________________
PC:
28 yo M presents with constipation (hard stool) for the past 3 weeks. Since his
mother died 2 months
ago, he and his father have eaten only junk food.

---
Low-fiber diet
Depression
Substance abuse (eg,heroin)
Irritable bowel syndrome
Hypothyroidism
---
OE:
Causes of Constipation
DOPED
Drugs - opioids/antidepressants
Obstruction
Pain - anal fissures
Endocrine dysfunction - DM, hypoThyroid, hypercalcemia,
Depression
pain reflieved by defecation, flatulence, diarrhoea, tenesmus, distension
---
ix:
CBC, TSH, Calcium, glucose
AXR
colonoscopy if alarm sx - rectal bleed, wt loss, anemia
Rectal exam
TSH
Electrolytes
Urine toxicology
---
mx:
fibre 30 g/d
sofeners - docusate, mineral oils
osmotic - lactulose, magensium slats, PEG
stimulants - castor oil, senna (melanosis coli), bisacoyl)
enemas
_______________________________________
PC:
30 yo F presents with alternating constipation and diarrhea accompanied by
abdominal pain that is relieved by defecation. She has no nausea, vomiting, weight
loss, or blood in her stool.

---
1o -
---
ddx:

Irritable bowel syndrome


Inflammatory bowel
disease
Celiac disease
Chronic pancreatitis
GI parasitic infection (ascariasis, giardiasis)
Lactose intolerance
---
OE:
ROME CRITERIA =
>12 wk in last 12 mo with 2 of 3 =
relieved by defecation
change in frequency of stool
change in consistency
sugestive = mucus, tenesmus, bloating

red flags ruling out:


wt loss
fever
nocturnal defecation
anemia
blood/pus in stool
abn flexible sigmoidoscopy
---
ix:
Rectal exam, stool for occult blood + ova and paratsites
Pelvic exam
Urine hCG
CBC + ESR/CRP
Celiac Tissue transaminase antibodies
Calprotectin - fecal (rule out IBD)
TSH
Electrolytes
Colonoscopy (for alarm features)
CTabdomen/pelvis
Stool for ova and
parasitology, Entamoeba
histolytica antigen
---
mx:
Reassurance + education
relaxation + biofeedback therapy
low FODMAP diet for pain, bloating, irregular bowel movement
Analgesia:
Anti-spasmodic - hyoscine
Incr diet (psyllium)
TCA
_______________________________________
PC:
33 yo M presents with watery diarrhea, vomiting, and diffuse abdominal pain that
began yesterday. He also reports feeling hot. Several of his coworkers are also
ill.

---
1o -
---
ddx:

Infectious diarrhea
(gastroenteritis) bacterial, viral, parasitic, protozoal Food poisoning
---
OE:
IF:
fever
bloody (CHESS = campylobacter, hemorrhageic Ecoli o157/H7, entamoeba
histolytica, salmonella, shigella
peritonitis
hypovolemia
recent abx
>65 yr
immunocompromised
>7d
sexual risk
THEN:
fecal leukocytes
stool mcs - campylobacter, salmonella, shigella
shiga or cdifficle toxin in stool
Stool ova/parasites for giardia, cryptosporidium, Entamoeba
ABX if
Typhi, shigella, cdifficle, crypto, histolytica, immunocompromised
OTHERWISE:
IVF if needed
consider - antidiarrhoea - bismuth salicylate, loperimide

volume status
adominal exam - pain, guarding, peritoneal
---
ix:
fecal leukocytes
stool mcs - campylobacter, salmonella, shigella
shiga or cdifficle toxin in stool
Stool ova/parasites for giardia, cryptosporidium, Entamoeba
---
mx:
IF:
fever
bloody (CHESS = campylobacter, hemorrhageic Ecoli o157/H7, entamoeba
histolytica, salmonella, shigella
peritonitis
hypovolemia
recent abx
>65 yr
immunocompromised
>7d
sexual risk
THEN:
fecal leukocytes
stool mcs - campylobacter, salmonella, shigella
shiga or cdifficle toxin in stool
Stool ova/parasites for giardia, cryptosporidium, Entamoeba
ABX if
Typhi, shigella, cdifficle, crypto, histolytica, immunocompromised
OTHERWISE:
IVF if needed
consider - antidiarrhoea - bismuth salicylate, loperimide
_______________________________________
PC:
40 yo F presents with watery diarrhea and abdominal cramps. Last week she was on
antibiotics for a UTI.

---
1o -
---
ddx:

Pseudomembranous colitis (Clostridium difficile)


Gastroenteritis
Cryptosporidiosis
Food poisoning
Inflammatory bowel disease
---
OE:

---
ix:
Stool culture MCS + ova/parasites
Stool PCR + immunoassay for toxin A+B genes C difficile stool toxin
flexible sigmoidoscopy + biopsy
CBC (bi WBC), EUC, CRP, TSH, celiac serology
colonoscopy
trial lactose free
ABG - lactate
AXR (colonic dilatation)
---
mx:
stop responsible abx (fluoroquinolones, cephalosporines)
IVF
metronidazole (severe = vancomycin)
stool transplantation
Toxic megacolon (metronidazole IV + vancomycin PO)

_______________________________________
PC:
30 yo F presents with watery diarrhea, abdominal cramping, and bloating. Her
symptoms are aggravated by milk ingestion and are relieved by fasting.

---
1o -
---
ddx:

Lactose intolerance
Gastroenteritis
Inflammatory bowel disease
Irritable bowel syndrome
Hyperthyroidism
---
OE:

---
ix:
trial lactose free die
watery stool, acid pH, reducing sugards
Rectal exam
Stool leukocytes and culture + MCS
Hydrogen breath test
TSH
transglutaminase erology
72 h stool collection for wt, fat content (statorrhea
fecal elastase (pancreatic insufficiency) + pancreatic enzyme trial
Serum carotene/vit A, folate, Ca, Mg, vit B12, albumin, ferritin, iron, INR/PTT
Stool sudan smear for gat globules
---
mx:
lactose free diet
soy substitution
lactase tablets - lacteeze
_______________________________________
PC:
33 yo M presents with watery diarrhea, diffuse abdominal pain, and weight loss
within the past 3 weeks. He has a history of aphthous ulcers. He has not responded
to antibiotics.

---
1o -
---
Crohns disease
Gastroenteritis
Ulcerative colitis
Celiac disease
Pseudomembranous colitis
Hyperthyroidism
Small bowel lymphoma
Carcinoid syndrome
---
OE:
ileum + ascending clon
linear ulcer
mucosal island
cobblestone
granulomas

- cramps + NON-bloody diarrhoea + weight loss


- ileitis = post-prandial pain, vomiting, RLQ mass, appendicitis
- fistula, fissures, abscess
---
ix:
colonoscope + Bx
CT
CRP
stool cultures - ova/parasite, C-difficile toxin

PR - blood
stool leukocytes, culture
CBC
EUC
colonoscopy
CT abdomen
TSH
small bowel series
5HIAA
---
mx:

lifestyle:
- cease smoking
- vitamine, diet supplementation
Antidiarrheal - loperamide
Sulfasalazine = 5-ASA
prednisone
mercaptopurine or azathioprine - immunosuppression
abx - metronidazole or ciprofloxacin
MTX
infliximab

SLIP VAMP = smoking, loperamide, infliximab, MTX, vitamins, ASA, metronidazole,


prednisome
_______________________________________
PC:
45 yo F presents with coffee-ground emesis for the past 3 days. Her stool is dark
and tarry. She has a history of intermittent epigastric pain that is relieved by
food and antacids.

---
Bleeding peptic ulcer
Gastritis
Gastric cancer
Esophageal varices
---
OE:
Rectal exam
CBC, type and cross
Electrolytes
AST/ALT/bilirubin/
alkaline phosphatase
INR
Upper endoscopy (including H pylori testing if ulcer is confirmed)
---
ix:
ABCs: assess vitals (BP and HR, orthostatic changes)
CBC, lytes, BUN, Cr, INR, blood type, cross and type

---
mx:
ABCs
Resuscitate: crystalloids and blood products if indicated
NGT + aspiration
Erythromycin before endoscopy
OGD to explore upper GIT
IV pantoprazole
establish rsik of rebleeding
Treat underlying
_______________________________________
PC:
40 yo F presents with epigastric pain and coffeeground emesis. She has a history of
rheumatoid arthritis that has been treated with NSAIDs. She is an alcoholic.

---
Gastritis
Bleeding peptic ulcer
Gastric cancer
Esophageal varices
Mallory-Weiss tear
---
OE:
hyplori, etoh, aspirin, nsaid, stress, radiation, celiac, crohns, sarcoid
---
ix:
Rectal exam
CBC, type and cross
Electrolytes
AST/ALT/bilirubin/ alkaline phosphatase
INR
Upper endoscopy
---
mx:
avoid irritatns - etoh, nsaids, foods
_______________________________________
PC:
Peripheral artery disease vs venous ucleration

---
1o -
---
ddx:

V -
I -
N -
D -
I -
C -
A -
T -
E -
M -
S -
---
OE:
PAD:
-atherosclerosis
-smokingm, DM, hyperlipidemia, htn
-intermittent claudication, criti al limb ischemia

Manifetations of PAD:
-cerebral - TIA, amaurosis fugax
-renal arteries, thn, failuyre
-mesenteric angina
-limb claudication

PC PAD:
-pulses diminished/absent
-bruits
-lower skin temperature
-buergers sign - pallor on elevation, rubor on dependency
-muscle wasting
-alopecia

PC arterial ulcers:
-punched out
-painful
-dry
PC venous ulcers
-medial malleolus
-edema
-leg heaviness
-venous eczema
-hemosiderin pigmentation
-lipodermatosclerosis (inverted champagne bottle)
-wet exudative
-no arterial compromise is ABI > 0.9
-eti - incompetent valves - hydrostatic pressure - fibrinogen exudate
---
ix:
ABPI
>1.2 wall calcification
>1 health
0.5-0.9 intermittent claudication
<0.5 critical limb
Claudication
-superficial femoral artery = calf
- iliac arteries = thich, buttock
-subsides with rest, worse with walking
-5% mortality associated with atherosclerosis, MI, stroke
Bloods,fBSL, ABI. CTA, MRA

UA - glycosuria
FBC - anaemia,
bacterial swab
doppler ultrasound for ABPI
duplex scanning
---
mx:
PAD
-Best medical therapy
.lifestyle - smoking cessagion, diet cholesterol, foot care
.monitor/control BSL, DM
.med - statin, DM, ASA, cilostazol
.infection control

-surgical
.BYPASS, STENTING, ANGIOPLASTY

vneous ulcer
-elevation, rest, compression stocking, remove exudate, wash
_______________________________________
PC:
Incontinence

---
1o -
---
ddx:

V -
I -
N -
D -
I -
C -
A -
T -
E -
M -
S -
---
OE:
- urgency, stress, mixed, nocturia, post micturition, overflow
- morbidity assoc : cellulitis, ulcer, UTI, falls, sleep deprivation, ocial
withdrawal, depression,
- urgency = spastic bladder = detrusor overactivity or bladder wall fibrosis
- stress = sphincter deficiency
- overflow = atonic bladder
---
ix:
Urge: Hx + diary + urodynamic
Stress: Hx + diary + urodynamic
Overflow: bladder US, residual volume

---
mx:
urge
Life style: fluid restriction, diet
Bladder retrain
M(-) oxybutynin
botox
stress
wt loss
kegel
bulking agent
surgical sling, tension free vaginal tape
_______________________________________
PC:
65 yo M presents with painless hematuria. He is a heavy smoker and works as a
painter.

---
1o -
---
ddx:

Bladder cancer
Renal cell carcinoma
Nephrolithiasis
Acute glomerulonephritis
Prostate cancer
Coagulation disorder (ie, factor VIII antibodies)
---
OE:
RF = smoking, aromatic amines (napthalines), cyclophosphamide, radiation,
schistosoma hematobium, cystitis
PC = painless hematuria + RF hx, palpable mass, obstruction/retention
---
ix:
UA + MCS, cytology
urine USS
CT + contrast
Dx = cystoscopy (dx + staging)****
Biops
tumor marker = NMP-22, BTA
metastatic workup = CT/MRI, CXR, LFT, CMP EUC
TNM staging
---
mx:
non invasive / T1 = 90% 5 yr
lo grade/ T1 ~resection + mitomycin C
hi grade - resection + BCG
invasiv / T2-3 = 30% 5 yr
- radical cystectomy + LNectomy + urinary diversion + chemo-radiation
- neoadjuvant chemo
mets = <5% 5 yr
-systemic chemo, irradiation, surgery

_______________________________________
PC:
35 yo M presents with painless hematuria. He has a family history of kidney
disease.

---
1o -
---
ddx:

Polycystic kidney disease


Nephrolithiasis
Acute glomerulonephritis
(eg, IgA nephropathy)
UTI
Coagulation disorder
Bladder cancer
---
OE:
flank pain
painless hematuria
nocturia
berry aneurysm
htn
palpable kidneys
UTIs
CKD
nephrolithiasis
chronic back pain
---
ix:
KFT = Cr, BUN, UA MCS
dx = renal USS**
CT abd + contrast
gene analsyis
MRI-B angio (aneurysms)
---
mx:
Educate - genetic counselling (ADPCKD)
TMP-SMX, ciprofloxacin prevention of UTI
Hydration to prevent nephrolithiasis
Avoid contact sport - damage to kidney
BP monitoring
Dialyusis
transplant

_______________________________________
PC:
55 yo M presents with flank pain and blood in his
urine without dysuria. He has experienced weight
loss and fever over the past 2 months. Exam reveals
a flank mass.

---
1o -
---
ddx:

Renal cell carcinoma


Bladder cancer
Nephrolithiasis
Acute glomerulonephritis
Pyelonephritis
Prostate cancer
---
OE:
RF = smoking, htn, obesity, horseshoe kidney/turners, napthalenes
PC = sytemic, gross hematuria, flank pain, palpable mass
paraneoplastic = HELP
-hematopoietic = anemia, polycythemia, ESR
-endocrine = hypercalcemia, erythrocytosis (incr epo), htn (incr renin), Incr
( prolactin, gonadotropin, tsh, insulin, cortiol)
-Liver = abn LFT, decr WBC, fever, necrosis,
-Pressure = htn
---
ix:
UA
renal USS
CT + contrast
renal biopsy
paraneoplastic = CBC, ESR, LFT, CMP/EUC
mets workup = CTR/MRI, CXR, EUC, CMP
Staging = CT, CXR, LFT, bone/head imagining
---
mx:
surg - radical nephrectomy / ablation
palliative radiation for bony lesions
Med -
.sorafenib (tyrosine kinase inhibitors for mets
.bevacizumab (anti VEGF)
.tesirolimus (mTOR inhibitor)
.IL-2 (Tcell inhibitor by upregulating Treg)
.IFN-alpha
_______________________________________
PC:
60 yo M presents with nocturia, urgency, weak
stream, and terminal dribbling. He denies any weight
loss, fatigue, or bone pain. He has had 2 episodes of
urinary retention that required catheterization.

---
1o -
---
ddx:

Benign prostatic hypertrophy (BPH)


Prostate cancer
UTI
Bladder stones
---
OE:
- prostatic transition zone - hypertrophy of smooth muscle (periurethral)
- Symtpoms = FUNWISE
.frequency
.urgency
.nocturia
.weak stream
.intermittency
.straining
.emptying dribbling
-c complications = IN BROH:
.infection
.bladder stones
.retention
.overflow
.hematuria, hydronephrosis
---
ix:
-Hx
-QoL impact = FUNWISE
-DRE
-UA - MCS (UTI)
-Cr (renal)
-renal USS (hydronephrosis)
-Prostate cancer screening
.DRE
.PSA > 4 (glycoprotein produced by epithelial cells of prostate gland)
-cytoscopy, bladder USS
-biopsy
---
mx:
Conservative:
-watchful waiting, fluid restriction, planned voiding
Medical:
A(-) tamsulosin
5alpha reductase inhibitor - finasteride
M(-) oxybutynin
Surgical
indicated for: refractory - HURT = hematuria, utis, renal failure +/- with
obstructive uropathy, sTONES
= TURP
_______________________________________
PC:
71 yo M presents with nocturia, urgency, a weak stream, terminal dribbling,
hematuria, and lower back pain for the past 4 months. He has also experienced
weight loss and fatigue.

---
1o -
---
ddx:

Prostate cancer
BPH
Renal cell carcinoma
UTI
Bladder stones
---
OE:
adenocarcinoma
PSA > 4
Storage / voiding
Bony pain (mets)
---
ix:
DRE
PSA
TRUS guided needle biopsy = histopath GLEASON SCORE
bone scan
CT-staging - TNM
T1/2 = normal life expectancy = use watchful waiting
T3-4 = 70% 10 yr
N or M = 5% 5 yr
Prognostic = stage, grade (gleason), PSA value and doubling time
MRI-spine
CT-pelvis
ALP

-
---
mx:
watchful waiting (short expectancy)
PSA, DR surveillance
Brachytherapy
radical prostatectomy
T1/2 - watchful waiting
T3/4 -
N or M
- external beam radical orchiectomy
- orchiectomy
- GnRH(+) leuprolide
- E2(+) diethystilbestrol
- androgen(-) bicalutamide
- chemo - taxel
_______________________________________
PC:
18 yo M presents with a burning sensation during urination and urethral discharge.
He recently had unprotected sex with a new partner.

---
1o -
---
ddx:

Urethritis
Cystitis
Prostatitis
---
OE:
gonorrhoea or chlamydia, ecoli
UTI, unprotected sex, catheter, immunocompromise

---
ix:
Genital exam
DRE
Grain stain + culture discharge
PCR

Gonorrhoea - urine PCR, culture, gram stain GN diplococci, yellow-grey discharge


Chlamydia - >4 PMN/field, urine PCR/culture
---
mx:
Gonococcal - ceftriaxone
Other - azithromycin
_______________________________________
PC:
45 yo diabetic F presents with dysuria, wrose on filling, better with voiding

---
1o -
---
ddx:
interstitial cystitis (mucosal defect, epithelial permable)
Acute pyelonephritis
Nephrolithiasis
Lower UTI (cystitis, urethritis)
Renal cell carcinoma
Radiation/chemical/eosinophilic/TB cystitis
---
OE:
- bladder pain - worse with filling, relieved with empty
- glomerulations (bladder hemorrhages)
- urinary urgency
- negative UA/UTI MCS cytology
, chronic urgency and frequency without other reasonable causation

---
ix:
Hx
Exam
UA + microscopy
cystoscopy iwth bladder hyperdistension for Dx
---
mx:
1o - conservative, lifestyle, diet, stress mx
2o - amitriptyline, cimetidine, intravesical DMSO/lidocaine
3o - urinary diversion
_______________________________________
PC:
45 yo diabetic F presents with dysuria, urinary frequency, fever, chills, and
nausea for the past 3 days. There is left CVA tenderness on exam.

---
1o -
---
ddx:

Acute pyelonephritis
Nephrolithiasis
Lower UTI (cystitis, urethritis)
Renal cell carcinoma
Radiation/chemical/eosinophilic/TB cystitis
---
OE:
G(+) Ent faecalis, Saureus, Ssaprophyticus
G(-) Ecoli, klebsiella, proteus, pseudomonas, enterbacter
Stones, strictures, prostatic obstruction, DM, sickle, PCKD

freq, urgency, hematura


NOT dysuria unless also cystitis
fever, chills, nausea, vomigin, myalgia, malaise
CVA flank tenderness
---
ix:
UA MCS
BC
KFT - BUN/Cr
FBC + differential = leukocytosis
USS - abd, pelvic, renal
CT
DMSA nuclear med scan
---
mx:
Admit
IVF
IV-ciprofloxacin + ceftriaxone
Nephrectomy
Stenting if obstruction
_______________________________________
PC:
55 yo F presents with urinary leakage after exercise.
She loses a small amount of urine when she coughs,
laughs, or sneezes. She also complains of vague low
back pain. She has a history of multiple vaginal deliveries,
and her mother had the same problem after
the onset of menopause.

---
1o -
---
ddx:

Stress incontinence
Mixed incontinence
Urge incontinence
Overflow incontinence
Functional incontinence
UTI
Diabetes mellitus
---
OE:
Urge incontinence = 1. spastic bladder = UMNL = detrusor overactivity; 2. decr
bladder compliance/fibrosis
PANS (L2-4) = detrusor contraction + sphincter relaxation
SANS (T10-L2) - hypogastric plexus - relax detrusor + contract sphinter
Somatic (S2-4) - pudendal nn - voluntary distal sphincter
Afferent - bladder wall - pelvic + hypogastric nn
Pontine micturition center - detrusor sphincter dyssnergy - painful inability to
pass urine

detrusor overacting = CNS/UMNL/PANS overactivity, inflamm/infection (cystitis,


stone, tumor), bladder neck obstruction (tumor, stone), bph
decrease bladder compliance = fibrosis

Lower Urinary Tract Symptoms (LUTS)


Storage (FUND)
Frequency
Urgency
Nocturia
Dysuria
Voiding (SHED)
Stream changes
Hesitancy
Incomplete Emptying
Dribbling
---
ix:
- Hx - bladder diary
- reversible causes of urinary incontinence = DIAPERS
.delirium - temperature/MMSE
.inflamm/infe - UA, bladder scan, renal USS
.atrophic vaginitis
.pharmaceutical M(+) or BB, psychological - drug history
.Excess urine - bladder scan
.restricted mobiilty, retention M(-)
.stool impaction - DRE, AXR
- Urodynamics
CBC
Electrolytes, BUN/Cr,
glucose
UA, urine culture
Urodynamic testing
IVP
Cystourethroscopy
---
mx:
Conservative
.lifestyle - fluid restriction, stress)
.bladder training
M(-) oxybutynin
B3(+) phenylpropanolamine
Botox injectinos
_______________________________________
PC:
33 yo F presents with urinary leakage. She is unable
to suppress the urge to urinate and loses large
amounts of urine without warning. She has a history
of UTIs and a family history of diabetes mellitus.
She drinks 8 cups of coffee per day. She has been under
stress since her sister passed away a few months
ago.

---
1o -
---
ddx:

Urge incontinence
Mixed incontinence
Stress incontinence
Overflow incontinence
Functional incontinence
UTI
Diabetes mellitus

---
OE:
Urge incontinence = spastic bladder = UMNL)
(versus overflow incontience = atonic bladder = LMNL)

Stress urinary incontinence


-weak pelvic floor (normally the bladder neck is raised and separated from the
pelvic floor, so the abdominal muscles that squeeze the bladder, also squeeze the
neck and stop leakage. BUT if there is descent of the bladder neck, the abdominal
muscles only act on the bladder and can push the urine out through the uncompressed
neck = leakage)

-sphincter deficiency

---
ix:
Hx - bladder diary
- reversible causes of urinary incontinence = DIAPERS
.delirium - temperature/MMSE
.inflamm/infe - UA, bladder scan, renal USS
.atrophic vaginitis
.pharmaceutical M(+) or BB, psychological - drug history
.Excess urine - bladder scan
.restricted mobiilty, retention M(-)
.stool impaction - DRE, AXR
UA MCS
BUN/Cr
urodynamic stress test (bear down and cough)
Cystourethroscope
---

mx:
Conservative:
Diary
Weight loss
Kegel, weighted vaginal cones
fluid restrictions
Estrogen vaginal pessary/cream in postmenopausal
Abx if indicated
Bulking agents
Surgx - tension free vaginal tape, sphincter

_______________________________________
PC:
47 yo M presents with impotence that started 3
months ago. He has hypertension and was started on
atenolol 4 months ago. He also has diabetes and is
on insulin.

---
1o - beta blcokers for hypertension
---
ddx:

Drug-related ED
Testosterone deficiency
ED caused by hypertension
ED caused by diabetesmellitus
Psychogenic ED = 10% time
Peyronies disease (curvature of penis due to fibrous thickening of tunica
albuginea)

(IMPOTENCE)
Iatrogenic: pelvic surgery, pelvic radiation
Mechanical: Peyronies, post-priapism
Psychological: depression, stress, anxiety, PTSD, widower syndrome
Occlusive: arterial HTN, DM, smoking, hyperlipidemia, PVD, impaired veno-
occlusion
Trauma: penile/pelvic, bicycling
Extra factors: renal failure, cirrhosis, COPD, sleep apnea, malnutrition
Neurogenic: CNS (e.g. Parkinsons, MS, spinal cord injury, Guillain-Barr, spina
bifida, stroke), PNS
(e.g. DM, peripheral neuropathy)
Chemical: antihypertensives, sedatives, antidepressants, antipsychotics,
anxiolytics, anticholingerics,
antihistamines, anti-androgens (including 5- reductase inhibitors), statins, GnRH
agonists, illicit drugs
Endocrine: DM, hypogonadism, hyperprolactinemia, hypo/hyperthyroid
---
OE:

---
ix:
Hx - sexual, medical, psychosocial
Self administers questionare - sexual health inventory for men

Genital exam
Rectal exam
Glucose, BSL, HbA1c, cholesterol profile
TSH, CBC UA, testosterone free+total, prolactin, LK
CBC
Testosterone level
---
mx:
conservative -
.cease etoh, alcohol
.counselling
.vacuum device + ring
Mx DM, CVD, HTN, endocrinopathies
PDE(-) sildenafil
intercavernosus vasodilatory injection
.triple therapy = papaverine, phentolamine, PGE1
Penile implant
_______________________________________
PC:
40 yo F presents with amenorrhea, morning nausea
and vomiting, fatigue, and polyuria. Her last menstrual
period was 6 weeks ago, and her breasts are full
and tender. She uses the rhythm method for contraception.

---
1o -
---
ddx:

Pregnancy
Anovulatory cycle
Hyperprolactinemia
UTI
Hypothyroidism
---
OE:
-amenorrhea, N/V, breast tenderness, urinary frequency, fatigue
-GTPAL =
.gravidity - total # pregnancies of any gestation including current
pregnancy/ectopic/abortion/moles
.Total # term infants delivered
.Premature #
.Abortions <20 wk
.Liver chilren #
-year
-sex
-weight
-gestational age
-mode of delivery
-length of labor
-complications
1st prenantal visit:
- gestational age = naegels rule = 1st day of LMP + 7d - 3mo eg 1 April 2014
-> Jan 2015
- dating 12 wk ultrasound
- bleeding ?
- N/V
- previous pregnancies GTPAL
- fhx - genetics, birth defects, twins, consanguinous
- shx - smoking, alcohol, drugs, domestic violence
- BP + weight

---
ix:
Urine hCG
U/Sabdomen/pelvis
Pelvic exam
CBC
UA, urine culture
Prolactin, TSH
Baseline Pap smear,
cervical cultures,
rubella antibody,
HIV antibody,
hepatitis B surface antigen,
VDRL/ RPR

1st visit:
.bloods
-CBC, group + Rh
-antibodies
-infection screen
.rubella
.HBsAg
.VDRL
.pap smear
.gonorrhoea/chlamydia
.HIV
.TB testing
.varicella/vax
.toxoplasmosis
.UA MCS - bacteriria + proteinura
.pelvic exam - papsmear
---
mx:
Arrange follow up
Folic acid supplementation
Iron supplement
Vitamins
refer IF
-insulin dependent GDM
-VBAC
-HTN
-twns
-mlpresentaiton
-PPROM
-tears
_______________________________________
PC:
23 yo obese F presents with amenorrhea for 6
months, facial hair, and infertility for the past 3
years.

---
1o -
---
ddx:
Polycystic ovary syndrome
Thyroid disease
Hyperprolactinemia
Pregnancy
Ovarian or adrenal malignancy
Premature ovarian failure
---
OE:

---
ix = identify PCOS, rule out pituitary/adrenal disease

Labs:
Urine hCG
17-hydroxyprogesterone
LH/FSH, TSH, fT4, prolactin
Pelvic exam
free Testosterone, DHEAS
Pelvic USS
f-BSL, 75g OGTT, HbA1c

rotterdam: USS polycystic, androgen excess/hair, amenorrhoea


---
mx:
conservative -
.wt loss, exercise [decr peripheral estrogen formation]
.mechanical hair removal
pharm
.OCP [prevent endometrial hyperplasia from unopposed estrogen]
.metformin
.transexamic acid [menorrhagia]
.spironolactone [inhibit steroid receptors]
.finasteride [reduce hair], mechanical hair removal
_______________________________________
PC:
35 yo F presents with amenorrhea, galactorrhea, visual field defects, and headaches
for the past 6 months.

---
1o -
---
ddx:

prolactinoma
Pregnancy
Thyroid disease
Premature ovarian failure
Pituitary tumor
---
OE:
galactorhoea
infertility
hypogonadism
amenorrheoa
erectile dysfunction
---
ix:
Urine hCG
LH/FSH, TSH, prolactin
MRIbrain
Pelvic and breast exams
---
mx:
DA(+) bromocriptive, cabergolin (elevates LFTs)
transphenoidal surgery

_______________________________________
PC:
48 yo F presents with amenorrhea for the past 6 months accompanied by hot flashes,
night sweats, emotional lability, and dyspareunia.

---
1o -
---
ddx:

Menopause
Pregnancy
Pituitary tumor
Thyroid disease
---
OE:
(failing theca cells = no estrogen response to FSH and LH = incr FSH + LH = LB
stimulation causes incr androgens)
no menses 1 yr (premature if <40 yo)
estrogen deficiency:
-vasomotor instbaility = flushes, sweats, sleep disturbance, fornication,
nausea, palpitations
-vaginal atrophy = dysparaeunia, pruritis, dryness, bleeding, urinary
frequency, urgency, incontinence
-OP
-decr breast size, skin thinning
-irritable, fatigue, libido, memory loss
---
ix:
Urine hCG
incr FSH on d3
estradiol levels
LH/FSH (incr FSH and LH but FSH>LH)
TSH, prolactin
Testosterone, DHEAS
Pelvic exam
CBC
MRIbrain
---
mx:
vasomotor = 1o - HRT, 2o - ssri
atrophy = estrogen cream
lifestle = wt loss, bladder retrain
OP = wt bearing exercise, cease smoking, bisphosphonates, raloxifene, vit D/Ca
Libido = vaginal lubrication, counselling
CVD = mx

_______________________________________
PC:
35 yo F presents with amenorrhea, cold intolerance, coarse hair, weight loss, and
fatigue. She has a history of abruptio placentae followed by hypovolemic
shock and failure of lactation 2 years ago.

---
1o -
---
ddx:

Sheehans syndrome
Premature ovarian failure
Pituitary tumor
Thyroid disease
Ashermans syndrome
---
OE:

---
ix:
Triple bolus test = rapid IV infusion insulin, GnRH, TRH (stimulates all anterior
pituitary hormones if normal)

Urine hCG
LH/FSH, prolactin
CBC
Pelvic exam
TSH, FT4
ACTH
MRIbrain
Hysteroscopy
---
mx:
?
_______________________________________
PC:
18 yo F presents with amenorrhea for the past
4 months. She is 5 feet, 6 inches (167.6 cm) and
weighs 90 lbs (40.9 kg). She has a history of exercise
and heat intolerance.

---
1o -
---
ddx:

Anorexia nervosa
Pregnancy
Hyperthyroidism
---
OE:

---
ix:
everything is low except the G+Cs which are hi (growth hormone, glucose,
cholesterol, calcium)
Urine hCG
CBC
TSH, FT4
LH/FSH
BMI/weight
OP - BMD w dexa
ser calcium, CBC, creatinint, ALP, TSH
25OF-vit D after 3 mo supplementation
lateral + thoracic XR
FRAX 10 year fracture risk tool
---
mx:
family psychotherapy

OP:
- calcium
- exercise
- cease smoking
- reduce caffeine
- bisphosphonates
- RANKL-inhib denosumab
- PTH hormone - teriparitide
- Calcitonin (inhibits osteoclasts)
- SERM raloxifene
- HRT
_______________________________________
PC:
29 yo F presents with amenorrhea for the past 6 months. She has a history of
occasional palpitations and dizziness. She lost her fianc in a car accident in
which she was a passenger.

---
1o -
---
ddx:

Anxiety-induced
amenorrhea
Posttraumatic stress
disorder
Depression
Hyperthyroidism
---
OE:

---
ix:
CBC
TSH, FT4
Urine cortisol level
Progesterone challenge
test
LH/FSH, estradiol levels
---
mx:

_______________________________________
PC:
17 yo F presents with prolonged, excessive menstrual bleeding occurring irregularly
within the past 6 months.

---
1o -
---
ddx:

Dysfunctional uterine bleeding


Coagulation disorder (eg, von Willebrands disease, hemophilia)
Cervical cancer
Molar pregnancy
Hypothyroidism
Diabetes mellitus
---
OE:
# pads > 80 cc per cycle, or cycle > 8d, affects QoL
PALM (structural) -COEIN (non structural)
polyps
adenomyosis
leiomyoma
malignancy and hyperplasia
COEIN
coagulopathy
ovulatory dysfuntion
endometrial
iatrogenic
not yet classified
---
ix:
Urine hCG
Pelvic exam
Cervical culture
Pap smear
CBC
ESR
Glucose
PT/PTT
LH/FSH, TSH, prolactin
U/Spelvis

PALMS = [TV-USS, sono-hysterography + saline, MRI, endometrial Bx to exclude


cancer]

COEIN =
C = cbc, coagulopathy, vWF, f8
O = bHCG, ferritin, PRL, FSH, LH, fTest, DHEA, prog, 17-hydroxy progesterone,
TSH, fT4, pelvic USS
E = endometrial Bx
I = TV -USS, OCP/HRT

---
mx:
resuscitate + IVF
transexamic acid
mirena IUD
endometrial ablation/hysterectomy
_______________________________________
PC:
61 yo obese F presents with profuse vaginal bleeding
for the past month. Her last menstrual period was
10 years ago. She has a history of hypertension and
diabetes mellitus. She is nulliparous.

---
1o -
---
ddx:

Endometrial cancer
Cervical cancer
Atrophic endometrium
Endometrial hyperplasia
Endometrial polyps
Atrophic vaginitis
---
OE:
post menopausal bleeding

Risk Factors for Endometrial Cancer


COLD NUT
Cancer (ovarian, breast, colon)
Obesity
Late menopause, LYNCH SYNDROME (hereditary non-polyposis colorectal cancer)
Diabetes mellitus
Nulliparity
Unopposed estrogen: PCOS, anovulation, HRT
Tamoxifen: chronic use
---
ix:
Pelvic exam
Pap smear
Endometrial curettage
Colposcopy
D+C + Hysteroscopy
Endometrial biopsy
U/Spelvis

FIGO staging
1. confied to corpus
2. into myometrium
3. local spread, serosa, adnexa
4. bladder, bowel, distant

---
mx:
surg + RadTx
Chemo
_______________________________________
PC:
45 yo G5P5 F presents with postcoital bleeding. She is a cigarette smoker and takes
OCPs.

---
1o -
---
ddx:
Cervical cancer
Endometrial cancer
Cervical polyp
Cervicitis
Trauma (eg, cervical laceration)

cerivcal lesions ddex:


cancer
Nabothian cyst/inclusion cyst
no treatment required
endocervical polyps
treatment is polypectomy (office procedure)
---
OE:
RF
.HPV infection
.smoking
.multiple partners, male to male
PC
.post coital, post menopausal bleeding
.pelvic, back pain
.bladder, bowel sx
.water to brown/red discharge
---
ix:
Pelvic exam
Pap smear
Colposcopy and biopsy
HPV testing
Endometrial biopsy

acetic acid visualisation


endocervical curettage
diagnostic excision - LEEP or cone biopsy
FIGO staging =
.pelvic exam under anaesthesia
.cervical biopsy + cone biopsy
.proctoscopy
.USS liver + kidneys
.CXR
.LFTs + KFT
CT/MRI
Bloods
---
mx:

_______________________________________
PC:
28 yo F who is 8 weeks pregnant presents with lower abdominal pain and vaginal
bleeding.

---
1o -
---
ddx:

Spontaneous abortion
Ectopic pregnancy
Molar pregnancy
---
OE:
decr fetal movement
maternal weight not increasing
absent fetal heart tones on doppler
hi MS-AFP, maternal serum alpha feto protein
---
ix:
Urine hCG
Quantitative serum hCG
U/Sabdomen/pelvis - fetal heart beat
Pelvic exam
CBC, HbA1c, fBSL, TSH, Kleihauer Betke, VDRL, ANA, CBC, anticardiolipins, INR/PTT,
cervical/vaginal cultures, TORCH screen
PT/PTT
---
mx:
<12 wk = D+C
>12 wk = induction of labor (PGE2 causes myometrial contractions)
monitor for maternal coagulopathy
psych support
_______________________________________
PC:
32 yo F presents with sudden onset of left lower abdominal pain that radiates to
the scapula and back and is associated with vaginal bleeding. Her last menstrual
period was 5 weeks ago. She has a history of pelvic inflammatory disease and
unprotected intercourse.

---
1o -
---
ddx:

Ectopic pregnancy
Ruptured ovarian cyst
Ovarian torsion
Pelvic inflammatory disease

DDx of Lower Abdominal Pain


Urinary tract: UTI, kidney stones
GI: diverticulitis, appendicitis
Gyne: endometriosis, PID, fibroid (degenerating, infarcted, torsion), ovarian
torsion, ovarian neoplasm, ovarian cyst, pregnancy-related
---
OE:
Ampulla > isthmic > fimbrial

suspect if:
-bHCG(+)
-abdominal pain
-vaginal bleeding

Clinical Features of Ectopic Pregnancy 4Ts and 1S


Temperature >38C (20%)
Tenderness: abdominal (90%) rebound (45%)
Tenderness on bimanual examination, cervical motion tenderness
Tissue: palpable adnexal mass (50%) (half have contralateral mass due to lutein
cyst)
Signs of pregnancy (e.g. Chadwicks sign,Hegars sign)

Chadwick's sign is a bluish discoloration of the cervix, vagina, and labia


resulting from increased blood flow. It can be observed as early as 6 to 8 weeks
after conception, and its presence is an early sign of pregnancy

Hegar's sign is a non-sensitive indication of pregnancy in women its absence does


not exclude pregnancy. It pertains to the features of the cervix and the uterine
isthmus. It is demonstrated as a softening in the consistency of the uterus, and
the uterus and cervix seem to be two separate regions

RFs
-previous ectopic
-IUD
-PID
-IVF
-pelvic aurgx
-smoking
-leiomyoma
---
ix:
Urine hCG
Quantitative serum hCG (<20% rise per 2/daus)
U/Sabdomen/pelvis, transvaginal USS (fetal heart rate)
Pelvic exam
Cervical cultures
check blood group + Rh
---
mx:
MTX
Rhogam
monitor
rupture = laparotomy + salpingostomy (salvageable)/salpingectomy(removal)
Follow up HCG
_______________________________________
PC:
1. 28 yo F presents with a thin, grayish-white, foulsmelling
vaginal discharge.

2. 30 yo F presents with a thick, white, cottage cheese


like, odorless vaginal discharge and vaginal itching.

3. 35 yo F presents with a malodorous, profuse, frothy,


greenish vaginal discharge with intense vaginal itching
and discomfort.

---
1o -
---
ddx:

1. Bacterial vaginosis (gardnerella vaginalis, mycoplasma hominis, bacteroides)

2. Vaginitiscandidal

3. Vaginitistrichomonal

ddx - Cervicitis (chlamydia,gonorrhea)


---
OE:

RF
sexually active, unprotected sex, previous sti, new sexual partner, homeless,
susbtance abuse
Hx - Hx , paps, communicaiton partner
Sexual History 5 Ps
Partners (numbers, gender)
Practices (vaginal, oral, anal insertive/
receptive)
Protection
Past history of STIs
Pregnancy prevention
PC sti general
- burning, itching, discharge, sores/vesicles, testicular pain, dysurina, abd
pain
-fever, LNpathy, arthralgia
- bacterial vaginosis
.d/c = grey, thin diffuse, fishy, ph>4.5
.wet mount = squaous epithelial cells with coccobacilli (gardnerella),
no wbc, no lactobacilli
.positive whiff test
- trichomoniasis (trichomonas vaginalis= flagellated protozoan)
.d/c = yellow-green, malodorous, diffuse, frothy
.petechiae (vagina, cervix), dysuria, frequency, post coital bleeding
.pH>4.5
.wet mount = motile flagellated, +++wbc, inflamm cells PMN
- candidiasis
.d/c = white, cottage cheese
.pruritis, swollen inflamed, vulvar burning, dysuria, dyspaeunia
.pH<4.5
.wetmount = hypae, spores
- chlamydia trachomatis
.mucopurulent d/c
.dysuria, frequency, pyuria, pelvic pain, postcoital bleed,
intermentrual bleed
.NAAT

---
ix:
- screen chlamydia, gonorrhea, hepB, hiv, syphilic
.urine PCR, endocervical swab for gram stain + culture
.vaginal swab for wet mount (rule out trichomonas)
.pharyngal/rectal swabs
.wet mount for vulvovaginitis (candidiasis, bacterial vaginosis,
trichomoniasis)
- Pap if not done in last 12 mo
- pelvic exam
---
mx:
chlamydia - azithromycin
gonorrhea - ceftriaxone + azithromycin
hep - hep B vaccine
HPV - guardasil, imiquimod, podophylin, cryotherapy,
bacterial vaginosis - metronidazole PO or topical or clindamycin intravaginally,
probiotics
trichoniasis - metrodizole + treat partners
candidiasis - clotrimazole suppository, or fluconazole PO
_______________________________________
PC:
54 yo F c/o painful intercourse. Her last menstrual period was 9 months ago. She
has hot flashes.
---
1o -
---
ddx:

Atrophic vaginitis
Endometriosis
Cervicitis
Depression
Domestic violence
---
OE:
-thinning of vaginal epithelium
-incr pH <> 5
-reduced secretion
- short and narrow of vaginal annal, loss of distensibility
- loss of rugae

PC-
Vaginal dryness
Vaginal burning or irritation
Decreased vaginal lubrication during sexual activity
Dyspareunia, including vulvar or vaginal pain (at the introitus or within the
vagina)
Vulvar or vaginal bleeding (eg, postcoital bleeding, fissures)
Vaginal discharge (leukorrhea or yellow and malodorous)
Pelvic pressure or a vaginal bulge
Urinary tract symptoms (eg, urinary frequency, dysuria, urethral discomfort,
hematuria
---
ix:
pelvic exam
- vaginal pH > 5
- wet mount - maturation index - decr superficial cells
ser hormone
LH/FSH
Wet mount, KOH prep
Cervical cultures
---
mx:
vaginal moisturising agents
vaginal lubricants
vaginal estrogen
_______________________________________
PC:
37 yo F presents with dyspareunia, inability to conceive,
and dysmenorrhea.

---
1o -
---
ddx:

Endometriosis
Cervicitis
Vaginismus
Vulvodynia
Pelvic inflammatory disease
Depression
Domestic violence
ectopic pregnancy
---
OE:
Classic Triad of Endometriosis
Dysmenorrhea + cyclic sx + back ache
Dyspareunia (cul-de-sac, uterosacral ligament)
Dyschezia (uterosacral ligament, cul-desac,rectosigmoid attachment)
Infertility
Mass/endometrioma
---
ix:
- Dx = laparoscopic visualisation of lesions = gold standard + Bx
.mulberry spots, endometrioma, white lesions
-elevated CEA125

Pelvic exam
Wet mount, KOH prep
Cervical cultures
U/Spelvis
Laparoscopy
Endometrial biopsy
---
mx:
1o - OCP + NSAID, or mirena IUS
2o - GnRH(+) (leuprolide) + Mirena IUS
3o - laparoscopic Dx + tx (laser/cautery/ablation, adhesion lysis)
.definitive = bilateral salpingo-oophorectomy + hysterectomy
4o - pain support
5o - MDT
_______________________________________
PC:
1. 28 yo F c/o multiple facial and bodily injuries. She claims that she fell on the
stairs. She was hospitalized for physical injuries 7 months ago. She presents with
her husband.

30 yo F presents with wrist pain and a black eye after tripping, falling, and
hitting her head on the edge of a table. She looks anxious and gives an
inconsistent story.

2. 30 yo F presents with multiple facial and physical injuries. She states that she
was attacked and raped by 2 men.

---
1o -
---
ddx:

1. Domestic violence
Osteogenesis imperfecta
Substance abuse
Consensual violent sexual behavior

2. Rape
---
Establish confidentiality; directly question about physical, sexual, or emotional
abuse and about fear, safety, backup plan; history of frequent accidents/injuries,
mental illness, drug use; firearms in the home.

OE: DO NOT TRAUMATISE


- remove clothes and put in bag
- document abrasions, bruises, lacerations, torn frenulum/broken teetch
Pelvic exam + specimens
-bf urination/defecation
-trauma, semina stains
-seminal specifment
-pubic hair combing/cutting
-speculum exam
ensure privacy
- who when, where, did penetration occur +/- weapons
- post assault - urination, defecation, change clothes, shower, douche
- gyn hx - pregnancies, contraception, last voluntary intercourse
- med hx
---
ix:
1. XRskeletal survey
CTmaxillofacial
Urine toxicology
CBC

2.
Forensic exam (sexual assault forensic evidence [SAFE] collection kit)
Pelvic exam
Urine hCG
Wet mount, KOH prep
Cervical cultures
Chlamydia and gonorrhea testing
XRskeletal survey
CBC
HIV antibody
Viral hepatitis serologies
ser bHCGVDRL
---
mx:
1. treat injuries and document findings
ask about sexual assault and children at home (encourage notification of
police)
safety plan with good follow-up with family doctor/social worker

2. MDT + sexual assault team


- suture lacerations
- tetanus
- gyn consult
- assume positive gonorrhea/chlamydia = aziothromycin + ceftriazone
- hep B prophylaxis + HIV testing
- pregnancy prophylaxis = levonorgestrel
- psych support
_______________________________________
PC:
30 yo F secretary presents with wrist pain and a sensation of numbness and burning
in her palm and the first, second, and third fingers of her right hand. The pain
worsens at night and is relieved by loose shaking of the hand. There is sensory
loss in the same fingers. Exam reveals a positive Tinels sign.

---
1o -
---
ddx:

Carpal tunnel syndrome


Median nerve - compression in the forearm or arm (where nerve passes through
pronator teres = forearm pain/sensory loss involving entire lateral palm, sensory
loss over thenar (spared in CTS), weakness of thumb flexion, wrist flexion, arm
pronation
Radiculopathy of nerve roots C6 and C7 in the cervical spine
De Quervains tenosynovitis
---
OE:
numb first 3+ 1/2 digits - sensory testing
radiation to elbow
worse at night
tinels sign
phalens test
thenar muscle wasting

---
ix:
dx = clinical
EMG
NCS (slowing)
MRI
USS
---
mx:
Sx relief = splinting, corticosteroid injections, NSAID
DOC = surgical decompression
TENS, yoga
_______________________________________
PC:
28 yo F presents with pain in the interphalangeal joints of her hands accompanied
by hair loss and a rash on her face.

---
1o -
---
ddx:

Systemic lupus
erythematosus (SLE)
Rheumatoid arthritis
Psoriatic arthritis
Parvovirus B19 infection
---
OE:
MD SOAP BRAIN:
malar rash
discoid rash, drug induced (hydralazine, isoniazid, procainamide)
serositis (pleuritis, pericarditis)
oral ulcers
arthritis
photosensitivity
blood - pancytopenia, anemia, leukopneia, thrombocytopenia
renal - nephropathy - proteinuria +3, >0.5g/d, casts -
rbc/hb/granular/tubular
ANA - most sensitive
immune - anti-dsDNA, anti-Sm, antiphospholipid Ab, false positive VDRL
.HLA-B8/DR3
.famly history
neurologic - psych, seizures

---
ix:
1o - ANA screen (hi Sn)
2o - Dx (hi Sp) = anti-dsDNA, anti-Sm = can monitor disease
3o - low C3/C4 complement, antiphospholipid Ab

ESR, low bloods


incr PTT

---
mx:
1. derm = avoid sun, use sunscreen, topical steroids
2. msk =
NSAID + omeprazole
hydroxychloroquine
anti-OP = bisphosphonates, calcium, vit D, calcitonin, denosumab
3. severe =
prednisone PO, methylpred IV
steroid sparing azathioprine, MTX, mycophenolate
4. cerebritis, lupus nephritis
cyclophosphamide IV

_______________________________________
PC:
28 yo F presents with pain in the metacarpophalangeal joints of both hands. Her
left knee is also painful and red. She has morning joint stiffness that lasts for
an hour. Her mother had rheumatoid arthritis.

---
1o -
---
ddx:

Rheumatoid arthritis
SLE
Disseminated gonorrhea
Arthritis associated with
inflammatory bowel disease
---
OE:
chronic symmetrical erosive polyarthritis + morning stiffness > 1hr, >6 wk,
elevated CRP/ESR
Effusions
tenosynovitis
subcut nodules
joint deformities
crepitus
wrists, MCP, MTP
HLA-DR4/DR1
atlanto-axial / subaxial subluxation, C-spine instability, neurological impingeent

RA is an independent RF for atherosclerosis and CVD

Extra-articular:
general
fatigue
depression
myalgia
weight loss
loss of function
skin
nodules,periungal infarctions, palapble purpura
Ocular
scleritis, keratoconjunctival sicca
H+N
xerostomia, hashimotos thyroiditis
cardiac
pericarditis, valve dz, conduction defect
pulmonary
fibrosis, effusion
neurologi
peripheral
stocking/glove
mononeuritis multiplex
heam
splenomegaly, neutropenia (felty's)
renal
amyloidosis

---
ix:
ESR incr
Platelets incr
Hb decr
RF(+)
anti-CCP(+) (t-cell signalling/cytokines induce enzymes converting arginine to
citrulline eg and cigarette smoke = immune reactivity to neo-epitopes)
incr TNF levels incr osteoclasts and decr osteoblasts + upregulation of RANK-L
XR/USS/MRI - joint space narrowing, erosions, absence bone repair

[XRhands, left knee


ANA, anti-dsDNA, ESR,
RF, CCP
CBC
Cervical culture
Arthrocentesis and
synovial fluid analysis]
---
mx:
Behavioural modification - isometrics, water, job, PT/OT
DMARD
1o - MTX
2o - add on hydroxychloroquine (blocks Toll-like receptors on dendritic cells
preventing subsequent presentation ot antigen to Tcells), or sulfasalazine
(metabolised by bacteria to sulfapyridine which is absorbed and mesalazine 5ASA
which remains in colon;, or leflunomide (inhibits dihydroorotate dehydrogenase to
inhibit pyrimidine synthesis)
3o - add on anti-TNF (infliximab) - check for TB (interferon gold test)
4o - pain mx - NSAIDs, low dose prednisone
5o - OP mx - DEXA, bisphosphonates, calcium, vit D, calcitonin, denosumab
6o - surgery for joint damage, synovectomy, joint replacement/fusion

_______________________________________
PC:
18 yo M presents with pain in the interphalangeal
joints of both hands. He also has scaly, salmon-pink
lesions on the extensor surface of his elbows and
knees.

---
1o -
---
ddx:

Psoriatic arthritis
Rheumatoid arthritis
SLE
Gout
---
OE:
seronegative arthritis with psoriasis
psoriasis = well demarcated erythematous plaques with silver scale
- nail pitting, oncholysis
MSK arthritis - various patterns
ophthalmic - conjunctivitis, anterior uveitis
cardiac/resp - AR, lung fibrosis, cauda equina
Radiological
-syndesmophytes
-pencil in cup appearance at IP joints
-osteolysis, periostitis
---
ix:
ANA, ESR, RF, CCP
CBC
XRhands
XRpelvis/sacroiliac
joints
Uric acid

CASPAR criteria for PsA


1. psoriasis
2. nail involvement
3. negative RF
4. dactylitis
5. radiological
---
mx:
Psoriasis: s
mild = 1o calcipotriene + topical steroids
severe = PUVU + retinoids (pregnancy test), MTX, cyclosporin, apremilast
- adalimumab, - etanercept, - infliximab (all anti
TNF)
- tacrolimus (IL-2 inhibitor)
Arthritis:
- exercise, PT, OT, behavioural modification, podiatrist/orthotics
- weight reduction
- patient eucation
-NSAID, steroids + OP prophylaxis
- DMARDS:
1o - MTX, leflunomide
2o - infliximab (+TB check)
_______________________________________
PC:
65 yo F presents with inability to use her left leg or
bear weight on it after tripping on a carpet. Onset of
menopause was 20 years ago, and she did not receive
HRT or calcium supplements. Her left leg is externally
rotated, shortened, and adducted, and there is
tenderness in her left groin.

---
1o -
---
ddx:

Hip fracture
Hip dislocation
Pelvic fracture
---
OE:
acute onset pain
cannot weight bear
SHORTENED + EXTERNALLY ROTATED
painful ROM

types:
subcapital #
intertrochanteric
subtrochanteric

NOF garden classes:


I. no displacement + not completely thru = internal fixation
II. no displacement but complete = internal fixation
III. inferiorly displaced + complete + varus alignment = ORIF or arthroplasty
IV. complete displacement + full extent + varus = ORIF or arthroplastic

---
ix:
XRhip/pelvis
CT or MRIhip
CBC, type and cross
Serum calcium and vitamin D
Bone density scan (DEXA)
---
mx:
DVT prophylaxis - LMWH/enoxaparin
_______________________________________
PC:
40 yo M presents with pain in the right groin after a motor vehicle accident. His
right leg is flexed at the hip, adducted, and internally rotated.

---
1o -
---
ddx:

Hip dislocation
traumatic Hip fracture
---
OE:
mechanism ?
- blow to knee with abducted hip = anterior hip dislocation
.shortened, abducted, externally rotated
- force to knee with hip flexed and adducted (knee to dash board in MVA) =
posterior hip dislocation***most common type
.shortened, adducted, internally rotated limb

Vehicle(s) involved: weight, size, speed, damage


Location of patient in vehicle
Use and type of seatbelt
Ejection of patient from vehicle
Entrapment of patient under vehicle
Airbag deployment
Helmet use in motorcycle collision

Complications:
-OA
-AVN
-#femoral head, neck, shaft
-sciatic nerve palsy
-DVT
---
ix: trauma survey ABCDEFG - MIST AMPLE (MOA, injuries sustained, signs, treatment
and trends in vital signs, allergies, medications, past medical history, last ate +
LMP, events)
Initial imaging ;
-non contrast CT heat/face/C-spine
-CXR
-FAST or CT abdomen
-pelvic XR
XRhip
CT or MRIhip
CBC, type and cross
PT/PTT
Urine toxicology and blood alcohol level
Blood type and cross match
---
mx:
closed rochester
ORIF
_______________________________________
PC:
56 yo obese F presents with right knee stiffness and pain that increases with
movement. Her symptoms have gradually worsened over the past 10 years. She has
noticed swelling and deformity of the joint and is having difficulty walking.

---
1o -
---
ddx:

Osteoarthritis
Pseudogout
Gout
Meniscal or ligament damage
---
OE:
joint line tenderness
effusions
bony enlargement of joints
mal-alignment
ROM
crepitus
muscle atrophy

pain with motion, relived by rest


instabilit/buckling
locking
loss of function
---
ix:
XRknee
Joint space narrowing
Subchondral sclerosis
Subchondral cysts
Osteophytes
Bloods
.normal CBC, ESR, CRP
.neg RF, ANA
Normal Knee arthrocentesis synovial fluid analysis (cell count, Gram stain,
culture, crystals)
Uric acid
MRIknee
---
mx:
-behviour modification + weight loss + PT + splints/cane/bracing
-Panadol/NSAID, glucosamine/chondroitin
-neuropathic pain - amitriptyline (or ssri, gabapentin)
-jont injections - short term steroid injections, hyaluronic acid
-topical capsaicin, NSAID
-surgical debrideent, osteotomy, joint replacement
_______________________________________
PC:
45 yo M presents with fevers and right knee pain with swelling and redness.

---
1o -
---
ddx:

Septic arthritis
Gout
Pseudogout
Lyme arthritis
Trauma
Reiters syndrome (reactive arthritis)
---
OE:
MEDICAL EMERGENCY - rapid joint destruction + 15% mortality
Usually gram positive cocci - gonococcal 75%
-other include Saureus, strept, spneumoniae, kingella, haemophilus,
salmonella (sickle cell)
Usually hematogenous spread t/f look for other sourc of infection - skin/pneumonia

RF:
-multiple partners, unprotected sex
-bacteremia,joint surgery, immunocompromise

PC:
- Disseminated gonococcal infection triad:
.migratory arthralgia
.tenosynovitis next to inflamed joint
.dermatitis - pustular skin lesion
- fever, malaise, chills
- joint swelling, warmth, pain, inabiilty to beart weight, decr ROM

---
ix:
CBC + BC
Endocervical, urethral, rectal, oropharyngeal swabbing
Knee arthrocentesis and synovial fluid analysis (CBC + differential, Gram stain,
culture + sensitivitiy, crystals)
.infectious wbc > 15000/mm3, PMN>90%
Blood, urethral cultures
XRknee - use as baseline
Uric acid
Lyme titersIgG and IgM
---
mx:
monitor with XR knee + joint aspiration/cultures

empiric - ceftriaxone + vancomycin


gonococcal - ceftriazone + azithromycin
Streptococcus - ceftriaxone
S aureua - ceftriaxone
GNB - piperacillin/tazobactam ie tazocin
(enterobacter/ecoli/salmonella/shigella/campyl/yersinia, klebs, legionella,
pseudomonas, haemoph)

Surgical drainage if - persistent positive joint culture, hip involvement,


prosthetic joint

PT
_______________________________________
PC:
65 yo M presents with right foot pain. He has been training for a marathon.

---
1o -
---
ddx:

Stress fracture
Plantar fasciitis
Foot sprain or strain
---
OE:
local pain/tenderness
---
ix:
XRfoot
Bone scanfoot
MRIfoot
ABI doppler - neurovascular compromise

Ottawa ankle/foot rules = XR series only if their is:


- malleolus pain PLUS
- cannot weight bear
- pain in:
.posterior lateral or medial malleolus pain, or
- tenderness plantar foot at base of 5th metatarsal
- tenderness at navicular
---
mx:

_______________________________________
PC:
65 yo M presents with pain in the heel of the right foot that is most notable with
his first few steps and then improves as he continues walking. He has no known
trauma.

---
1o -
---
ddx:

Plantar fasciitis
Heel fracture
Splinter/foreign body
---
OE:
RF - athletes/runners/dancer, obesity, DM, arthritis

insidious onset heel pain - worse when getting out of bed


Pain subsides with walking
worse at end of day with prolonged standing
swelling + tenderness over sole
---
ix:
plain XR - ankle/foot series - rule out fractures
Look for bony exostoses
bone scan
---
mx:
conservative
-stretchiing, rest, ice, NSAID, steroid
-PT - achilles/plantar fasic stretching
-orthotics
surgical
-fascial release
_______________________________________
PC:
55 yo M presents with pain in the elbow when he plays tennis. His grip is impaired
as a result of the pain. There is tenderness over the lateral epicondyle as well as
pain on resisted wrist dorsiflexion (Cozens test) with the elbow in extension.

---
1o -
---
ddx:

Tennis elbow (lateral


epicondylitis)
Stress fracture
---
OE:
lateral epicondylitis - inflamm common extensore tendon insertion
-point tenderness over humeral epicondyle
-pain on resisted wrist extension (cozen test_)
medial epicondylitis (golders) - inflamm common flexor insertion into medial
epicondyle
-pain on resisted wrist flexion
---
ix:
XRarm
Bone scan
MRIelbow
---
mx:
consrevtive:
-rest, ice, NSAID
-brace/strap
-PT stretching
-steroid injection
_______________________________________
PC:
27 yo F presents with painful wrists and elbows, a swollen and hot knee joint that
is painful on flexion, a rash on her limbs, and vaginal discharge. She is sexually
active with multiple partners and occasionally uses condoms.

---
1o -
---
ddx:

Disseminated gonorrhea
Rheumatoid arthritis
SLE
Reiters syndrome (reactive arthritis)
---
OE:
systemic - fever, malaise, chills
joint - swelling, warmth, pain, no weight bear, decr ROM
disseminated gonococcal triad:
.migratory arthralgia
.tenosynovitis
.dermatitis - pustular lesion
Look for lesions or other sources of infections
---
ix:

Knee arthrocentesis and synovial fluid analysis (cell count, Gram stain, culture)
ANA, anti-dsDNA, ESR, RF, CCP
CBC
Blood, cervical cultures
XRknee
---
THIS IS A MEDICAL EMERGENCY
mx:
daily monitor with athrocentesis + cultures until sterile
empiric - ceft + vanc
gonococcal - azithro + ceft
PT
_______________________________________
PC:
60 yo F presents with pain in both legs that is induced
by walking and is relieved by rest. She had
cardiac bypass surgery 6 months ago and continues
to smoke heavily.

---
1o -
---
ddx:

Peripheral vascular disease (intermittent


claudication)
Leriche syndrome (aortoiliac occlusive disease)
Neurogenic claudication = Lumbar spinal stenosis (pseudoclaudication)
Complex regional pain syndrome
Osteoarthritis
---
OE:
PAD - 6 Ps
-pain, pallor, paresthesia, paralysis/power loss, poiliothermia,
pulselessness
-critical limb ischemia = rest pain, tissue loss/gangrene/ulceration, wakes
from sleep, relieved by hanging off bed / DEPENDECY, reduced ankle pressure
-hair loss, hypertrophic nails, atrophic muscle, ulceration, slow cap refill,
prolonged pallor with elevation (BURGERS TEST) and rubor on dependency
- embolus = sudden onset, loss of fn/sensation, NORMAL limb pulses, no
claudicaiton, no atrophy
- thrombus = slow onset, absent contralateral limb pulse, atophic,
claudication
-ulceration = deep champagne bottle, dry(not wet), cool
PVD (calf muscle pump dysfuction, valvular incompetence due to phlebitis,
variscosities, DVT)
-pain releived by ELEVATION
-pruritus
-brown hyperpigmentatio (hemosiderin deposit)
-stasis dermatitis
-ulceration (shallow, medial malleolus, weeping, painless, irregular)
-signs of DVT/variscosity
---
ix:
PAD
bedside
-ABI
-ECG,

Bloods
- troponin
-CBC
-PT/INR, PTT (if anticoagulated, + baseline)
- BSL, lipids
Imaging
-Echo (valvular, aortic dissection)
- CT-angio (atherosclerosis, aneurysm, dissection)

PVD
Doppler USS
---
mx:
PAD
Conservative:
-RF modification: stop smoking, control HbA1C (metformin), htn tx
(ACEI+diuretics), hyperlipidemia (statin), antiplatelet therapy (ASA)
-exercise 30min 3x/wk to improve collateral circulation and O2 extraction
-foot care/podiatrist

Pharm CHRONIC LIMB


- antiplatelets: ASA, clopidogrel
- cilostazol (CAMP PDE inhibitor - vasodilator)
- revascularisation -
.angioplasty + stenting
.endarterectomy (remove plaque)
.amputation

Pharm CRITICAL LIMB


-heparin
-surgx revascularisation
-embolus = embolectomy
-thrombus =thrombectomy + bypasss graft + endovascular therapy
-monitor for compartment syndrome (fasciotomy)
-renal failure due to toxin release from ischemic muscle

PVD
conservative:
-compression stocking, ambulation, avoid prolonged standing
-ulcers - compression + abx

surgical:
-ligation/stripping
-radiofrequency ablation or sclerotherapy
_______________________________________
PC:
dry/sore eyes, dry mouth + difficulty swallowing, hands saw

---
1o - Sjogrens
---

---
OE:
lymphocytic infiltration ofg salivary and lachrymal glands -> glandular fibrosis,
exocrine failure

triad: dry eyes (keratoconjunctivitis), dry mouth (xerostomia ->dysphagia),


arthrtisis (non erosive, small joint, asymm), dental caries

increased risk of non-hodgkins lymphoma


---
ix:
Schirmer tear test (rate of tear flow)
Bengal rose staining = punctate epithelial abn on conjunctiva
Lip Bx = lymphoctyic infiltrate of minor salivary glands
Elevated ESR + hypergammaglobulinemia
Antibodies = ANF, RF, anti RO, anti LA

Lymph node biopsy to exclude malignancy


---
mx:
conservative
Dental check up
Eye check up
Synthetic tears = hypromellose
Soft contac tlenses
Oral hyiene
Vaginal lubricant
pharm
pilocarpine M(+) stimulate salivary flow
Hydroxychloroquine (Toll like receptor inhibitor)
Corticosteroids

_______________________________________
PC:
55 yo M presents with crampy bilateral thigh and calf pain, fatigue, and dark
urine. He is on simvastatin and clofibrate for hyperlipidemia.

---
1o -
---
ddx:

Rhabdomyolysis due to statins


Polymyositis
Inclusion body myositis
---
OE:
aching, weakness, decreased exercise tolerance
leg pain
fatigue
dark urine
hx statin + fibrate
---
ix:
serum creatinine kinase *** + resolution on statin withdrawal
Muscle biopsy

CBC
Phosphate, potassium,
BUN/Cr, glucose,
calcium, uric acid
CPK
Aldolase
UA
Urine myoglobin
TSH
vit D levels
---
mx:
Stop statin therapy
Rehydration (excrete myoglobin)
vit D supplement
_______________________________________
PC:
45 yo F presents with low back pain that radiates to the lateral aspect of her left
foot. The straight leg raise is positive. The patient is unable to tiptoe.

---
1o -
---
ddx:

Disk herniation
Lumbar muscle strain
Tumor in the vertebral canal
---
OE:
protrusion of nucleus pulpous through annulus fibrosus - posterolateral herniation
especially L5-S1

Back pain/leg pain


Tenderness between spinous processes
Muscle spasm
Loss of lumbar lordosis
Motor weakness
-L2/3/4 - femoral = hip flexion (iliopsoas), knee extension (quads_
-L5 - sciatic = hip extension (glut Max)
- tibial nn/deep peroneal = ankle inversion/dorsiflexion (tibial mm)
-S1 - sciatic = knee flexion = hamstrings
Decr reflexes
Diminished sensation
Positive straight leg raise + contralateral
Caudaequina syndrome
---
ix:
XRL-spine
MRIL-spine
---
mx:

_______________________________________
PC:
45 yo F presents with low back pain that started after she cleaned her house. The
pain does not radiate, and there is no sensory deficit or weakness in her legs.
Paraspinal muscle tenderness and spasm are also noted.

---
1o -
---
ddx:

Lumbar muscle strain


Disk herniation
Vertebral compression fracture
---
OE:
dull backache aggravated by activity and prolonged standing
morning stiffness
no neurological signs
---
ix:
XRL-spine
MRIL-spine
---
mx:
symptomatic (analgesics, physiotherapy)
prognosis: symptoms may resolve in 4-6 wk, others become chronic
_______________________________________
PC:
45 yo M presents with pain in the lower back and legs during prolonged standing and
walking. The pain is relieved by sitting and leaning forward (eg, pushing a grocery
cart).

---
1o -
---
ddx:

Lumbar spinal stenosis


Lumbar muscle strain
Tumor in the vertebral canal
Peripheral vascular disease
---
OE:
bilateral back/leg pain
neurogenic claudication
motor weakness
normal back flexion
poor back extension (kemp sign)
positive straight leg raise

Red Flags = BACKPAIN


Bowel or bladder dysfunction
Anesthesia (saddle)
Constitutional symptoms/malignancy
Chronic disease
Paresthesias
Age >50 and mild trauma
IV drug use/Infection
Neuromotor deficits

Inspection:
curvature, posture, gait
Palpation:
bony deformities, tenderness, paraspinal muscles bulk/tenderness, rigger
pionts, percussion causes pain (fracture or infection)
ROM
Neuro exam - L4-5, S1 - power reflexes, sensation
Straight leg raise + dorsiflection + then do other leg

L4
-quads (femoral n) = knee extension
-tibialis = ankle inversion + dorsiflexion - tibial nn
-sensory = medial malleolus
-screen = squat and rise
-reflex = knee/patellar
L5
-extensor hallucis longus (big toe extension) - deep peroneal nn
-gluteus medius (hip abduction)
-sensory = 1st dorsal webspace and laterla leg
-screen = heel walking
S1
- peroneus longus/brevis (ankle eversion) - superficial peroneal nn
- gastroc + soleus (plantar flexion) - tibial nn
-screen walking on toes

---
ix:
MRIL-spine (preferred)
XRL-spine
CTL-spine
Ankle-brachial index

CBC, ESR

Indications for Lumbar Spine X-Ray


No improvement after 6 wk
Fever >38oC
Unexplained weight loss
Prolonged corticosteroid use
Significant trauma
Progressive neurological deficit
Suspicion of ankylosing spondylitis
History of cancer (rule out metastases)
Alcohol/drug abuse (increased risk of osteomyelitis, trauma, fracture)
---
mx:
Analgesia: acetaminophen, NSAID + PPI, lo dose TCA,
PT + rehab
Relaxation
Massage therapy
MDT chronic pain team
_______________________________________
PC:
17 yo M presents with low back pain that radiates to the left leg and began after
he fell on his knee during gym class. He also describes areas of loss of sensation
in his left foot. The pain and sensory loss do not match any known distribution. He
insists on requesting a week off from school because of his injury.

---
1o -
---
ddx:

Malingering
Lumbar muscle strain
Disk herniation
Knee or leg fracture
Ankylosing spondylitis
---
OE:

---
ix:
XRL-spine/knee
MRIL-spine

Malingering The essential feature of malingering is intentionally faking or


exaggerating symptoms for an obvious external benefit (eg, money, housing,
medications, or avoiding work or criminal prosecution) [12]. Malingering is a
behavior and not a psychiatric disorder.
Clinicians should consider the possibility of malingering if any of the following
are present:

History notable for multiple inconsistencies and vague responses



Eagerly or dramatically describing symptoms

Conditional threats (eg, Ill kill myself if you dont admit me)

Inconsistency between the history and mental status examination (eg, patient
appears calm and relaxed while reporting symptoms that are distressing)

Marked discrepancy between the patient's claimed distress or disability and the
objective findings (eg, emergency department patient reports depression but is
joking with the security staff, eating every meal, and sleeps throughout the night)

Using technical medical terms (eg, I have command auditory hallucinations)

Demanding specific medications, such as benzodiazepines and opiates

Nonadherence with diagnostic evaluation or treatment

Antisocial personality disorder

Medical-legal context (eg, patient is referred by an attorney for evaluation)

It is possible to establish feigning only if the patient acknowledges deliberately


producing symptoms, or if other evidence demonstrates a major inconsistency between
reported and observed function (eg, a patient who reports an inability to walk is
subsequently observed on a video recording playing tennis).
The motivation for intentionally producing symptoms in malingering is an external
incentive, whereas in factitious disorder the motivation is to assume the role of
patient.
---
mx:

_______________________________________
PC:
20-day-old M presents with fever, decreased breastfeeding, and lethargy. He was
born at 36 weeks as a result of premature rupture of membranes.

---
1o -
---
ddx:

Neonatal sepsis
Meningitis
Pneumonia
Pyelonephritis
---
OE:
resp distress, cyanosis, apnoea
tachy/brady
lethargy/irrit
poor feed, vomit, abdominal, distension, diarrhoea
hypotonia, seizures, lethargy
jaundice, hepatomegaly, petechiae, purpura
---
ix:
Physical exam
CBC
Electrolytes
Blood culture
LPCSF analysis
CXR
UA, urine culture
---
mx:

_______________________________________
PC:
3 yo M presents with a 2-day history of fever and pulling on his right ear. He is
otherwise healthy, and his immunizations are up to date. His older sister recently
had a cold. The child attends a day care center.

---
1o -
---
ddx: AOM

Acute otitis media (Hinf, Spneum, Mcatarrhalis,


URI
Meningitis
Pyelonephritis
---
OE:
GA - pulling at right ear
VS - fever
HEENT - conductive hearing loss (rinne + weber both louder on one side)
- otalgia, fever
- balance
- tinnitus
- facial nerve paralyss
- otorrhoea
- otoscopy

complications - mastoiditis, meningitis, sinus thrombosis


---
ix:
Physical exam (including pneumatic otoscopy)
CBC
Blood culture
Tympanocentesis culture
LPCSF analysis
UA, urine culture
---
mx:
amox 75 mg x 2 doses
macrolide (clarithromycin, azithromycin) - if penicillin allergy
_______________________________________
PC:
12-month-old M presents with fever for the past 2 days accompanied by a
maculopapular rash on his face and body. He has not yet received the MMR vaccine.
---
1o -
---
ddx:

Measles (maculopapular - from hairline spreads down spare palms/soles - prodrome


cough, coryza, conjunctivitis, koplik spots, desquamation, IgM mealses +) -
unimmunised contact with vaccine within 72 hr or IgG within 6d + report to public
health prevention
Rubella (rubivirus - pink maculopapular - face spreads to neck and trunk + STAR +
IgM rubella - MMR vax + report)
Roseola (HHV6 - fever + blamnching pink maculopapular - supportive
Fifth disease (erythema infectiosum = parvovirus B19 = SLAPPED CHEEKS, STAR sore
throat arthritis rash, aplastic crisis) - supportive Mx
Varicella - virus -
Scarlet fever (type 4 hypersensitivity against GAS pyogenes - fever, sore throat,
strawbetter tongue) - penicillin, amoxicillin
Meningitis
---
OE:

---
ix:
Physical exam
CBC
Viral antibodies/titers
Throat swab for culture
LPCSF analysis
---
mx:

_______________________________________
PC:
4 yo M presents with diarrhea, vomiting, lethargy, weakness, and fever. The child
attends a day care center where several children have had similar symptoms.

---
1o -
---
ddx:

Gastroenteritis (viral, bacterial, parasitic)


Food poisoning
UTI
URI
Volvulus
Intussusception
---
OE:

---
ix:
Physical exam
Stool exam and culture
CBC
Electrolytes
UA, urine culture
AXR
---
mx:

_______________________________________
PC:
1-month-old F is brought in because she has been spitting up her milk for the last
10 days. The vomiting episodes have increased in frequency and forcefulness. Emesis
is nonbloody and nonbilious. The episodes usually occur immediately after
breastfeeding. She has stopped gaining weight.

---
1o -
---
ddx:

Pyloric stenosis
Partial duodenal atresia
GERD
Gastroenteritis
Hepatitis
UTI
Otitis media
---
OE:

---
ix:
Physical exam
CBC
Electrolytes
U/Sabdomen/stomach
Barium swallow
pH probe
Endoscopy
AST/ALT/bilirubin/alkaline phosphatase
UA, urine culture
Tympanocentesis culture
---
mx:

_______________________________________
PC:
3 yo M presents with constipation. The child has had 1 bowel movement per week
since birth despite the use of stool softeners. At birth, he did not pass meconium
for 48 hours. He has poor weight gain. There is a family history of this problem.

---
1o -
---
ddx:
Hirschsprungs disease
Low-fiber diet
Anal stenosis
Hypothyroidism
Lead poisoning

---
OE:
---
ix:
Physical exam
Rectal exam
Stool exam and culture
Barium enema
Suction rectal biopsy
Anorectal manometry
TSH, FT4
CBC
Electrolytes
Serum lead level
---
mx:

_______________________________________
PC:
8-month-old F presents with sudden-onset colicky abdominal pain with vomiting. The
episodes are 20 minutes apart, and the child is completely well between episodes.
She had loose stools several hours before the pain, but her stools are now bloody.

---
1o -
---
ddx:
Intussusception
Appendicitis
Meckels diverticulum
Volvulus
Gastroenteritis
Enterocolitis
Blunt abdominal trauma
---
OE:

---
ix:
Physical exam
Rectal exam, stool for occult blood
CBC
Electrolytes
Contrast enema
U/Sabdomen
CTabdomen
---
mx:

_______________________________________
PC:
7 yo M presents with abdominal pain that is generalized, crampy, worse in the
morning, and seemingly less prominent during weekends and holidays. He has missed
many school days because of the pain. Growth and development are normal. His
parents recently divorced.

---
1o -
---
ddx:
Somatoform disorder
Malingering
Irritable bowel syndrome
Lactose intolerance
Child abuse

---
OE:

---
ix:
Physical exam
CBC
Electrolytes
U/Sabdomen
CTabdomen
Amylase, lipase
Stool exam
---
mx:

_______________________________________
PC:
2-month-old M presents with persistent crying for 2 weeks. The episodes subside
after passing flatus or eructation. There is no change in appetite, weight or
growth. There is no vomiting, constipation, or fever.

---
1o -
---
ddx:
Colic
Formula allergy
GERD
Lactose intolerance
Strangulated hernia
Testicular torsion
Gastroenteritis

---
OE:

---
ix:
Physical exam
Rectal exam, stool for occult blood
U/Sabdomen
U/Stesticular
---
mx:
reassurance - normal stage - dont punish
_______________________________________
PC:
3 yo F presents with a 3-day history of pink eye. It began in the right eye but
now involves both eyes. She has mucoid discharge, itching, and difficulty opening
her eyes in the morning. Her mother had the flu last week. She has a history of
asthma and atopic dermatitis.

---
1o -
---
ddx:

Bacterial conjunctivitis
Viral conjunctivitis
Keratitis
Seasonal allergies
Uveitis
---
OE:

---
ix:
Physical exam
Ophthalmoscopic eye exam
CBC
Electrolytes
Discharge cultures
Slit lamp exam
---
mx:

_______________________________________
PC:

14 yo M presents with short stature and lack of sexual development. His birth
weight and length were normal, but he is the shortest child in his class. His
father and uncles had the same problem when they were young, but they are now of
normal stature.
---
1o -
---
ddx:

Constitutional short stature


Growth hormone (GH) deficiency
Hypothyroidism
Chronic renal insufficiency
Genetic causes
Cystic fibrosis
---
OE:
Vital signs; height, weight; HEENT, heart, lung, abdominal, and neurologic exams.
---
ix:
Physical exam
CBC
Electrolytes
GH stimulation test
IGF-1, IGFBP-3 levels
TSH, FT4
XRhand
U/Srenal and cardiac
Sweat chloride testing
BUN/Cr
Karyotype
---
mx:
_______________________________________
PC:
9 yo M presents with a 2-year history of angry outbursts both in school and at
home. His mother complains that he runs around as if driven by a motor. His
teacher reports that he cannot sit still in class, regularly interrupts his
classmates, and has trouble making friends.

---
1o -
---
ddx:

Attention-deficit hyperactivity disorder (ADHD)


Oppositional defiant disorder
Manic episode
Conduct disorder
Hyperthyroidism
---
OE:
Onset, severity, duration, triggers; physical violence or use of weapons; substance
use, developmental history, changes
in environment or school performance; change in personality, anhedonia.
---
ix:
Physical exam
Mental status exam
TSH, FT4
EEG
---
mx:

_______________________________________
PC:
12 yo F presents with a 2-month history of fighting in school, truancy, and
breaking curfew. Her parents recently divorced, and she just started school in a
new district. Before her parents divorced, she was an average student with no
behavioral problems.

---
1o -
---
ddx:

Adjustment disorder Substance intoxication, abuse, or dependence


Manic episode
Oppositional defiant disorder
Conduct disorder
---
OE:

---
ix:
Physical exam
Mental status exam
Urine toxicology
---
mx:

_______________________________________
PC:
15 yo M presents with a 1-year history of failing grades, school absenteeism, and
legal problems, including shoplifting. His parents report that he spends most of
his time alone in his room, adding that when he does go out, it is with a new set
of friends.

---
1o -
---
ddx:

Substance abuse
Conduct disorder
Oppositional defiant
disorder
Adjustment disorder
---
OE:

---
ix:
Urine toxicology
Mental status exam
Physical exam
---
mx:

_______________________________________
PC:
5 yo M presents with a 6-month history of temper tantrums that last 510 minutes
and immediately follow a disappointment or a discipline. He has no trouble
sleeping, has had no change in appetite, and does not display these behaviors when
he is at day
care.

---
1o -
---
ddx:

Age-appropriate behavior
ADHD
Oppositional defiant disorder
---
OE:

---
ix:
Physical exam
Mental status exam
---
mx:

_______________________________________
PC:
A 36-year-old woman is evaluated in the clinic due to left hand clumsiness. The
patient works as an office secretary and has been having difficulty typing. Six
months ago, she had an episode of vertigo and difficulty walking that improved
after several weeks. The patient also has a history of bipolar disorder and takes
valproic acid. On physical examination, she is asked to close her eyes and stretch
out her arms with the palms facing up. Several seconds later, her left arm
involuntarily drifts downward and the palm turns toward the floor. Which of the
following best explains the findings observed in this patient?

A 25-year-old woman comes to the office due to intermittent double vision,


dizziness, and unsteady gait for the last several days. During the past year, she
has had several episodes of numbness and dizziness, and her symptoms were
attributed to anxiety. She does not use tobacco or alcohol. Her brother died of
subarachnoid hemorrhage. Blood pressure is 130/70 mm Hg and pulse is 76/min. On
attempted left gaze, her left eye abducts and exhibits horizontal nystagmus, but
her right eye remains stationary. When she attempts to look to the right, her right
eye abducts and exhibits horizontal nystagmus, but her left eye remains stationary.
The patient is able to converge both eyes without any associated nystagmus. Which
of the following is the most likely site of the lesion in this patient?

---
1o -
---
ddx:

multiple sclerosis
---
OE:
Onset at .age 15-50
Optic neuritis
Lhermitte sign (electrical spinal sensation on neck flexion)
Internuclear ophthalmoplegia
Fatigue
Uhthoff phenomenon (heat sensitivity)
Sensory symptoms (numbness & paresthesia)
Motor symptoms (paraparesis & spasticity)
Bowel/bladder dysfunction
Cerebellar dysfunction: intenttion tremor, ataxia, and nystagmus
Transverse myelitis: motor and sensory loss below the level of the lesion with
bowel and bladder dysfunction. Patients initially have flaccid paralysis (spinal
shock), followed by spastic paralysis with hyperreflexia.
---
ix:
Relapsing-remitting (majority)
Primary progressive
Secondary progressive
Progressive relapsing

Diagnosis
neurologic deficits disseminated in space and time in women age 15-50
T2 MRI lesions disseminated in time & space (periventricular, juxtacortical,
infratentorial, or spinal cord)
Oligoclonal lgG bands on cerebrospinal fluid analysis
evoked potentials (visual/auditory/somatosensory) - delays

When MS is suspected, magnetic resonance imaging (MRI) of the brain and spine is
typically performed to support the diagnosis. T2-weighted imaging usually
demonstrates multifocal, ovoid, subcortical hypo-/hyperintense lesions located in
the white matter of the CNS, particularly the periventricular and subpial white
matter of the cerebrum (corpus callosum), optic nerves, brainstem, and spinal cord.
Lumbar puncture can be performed after an MRI to document oligoclonallgG bands in
the cerebrospinal fluid (found in 85%-95% of patients with MS),
---
mx:
acute -
.methyprednisone
Disease modifying - (reduce relapse
1o:
.teriflunomide [inhibiting pyrimidine de novo synthesis by blocking the
enzyme dihydroorotate dehydrogenase]
.interferon-beta [not an appropriate treatment for patients with progressive,
non-relapsing forms of multiple sclerosis]
.glatiramer acetate (decoy myelin-antigen to mop up antibodies/Tcells)

2o:
.natalizumab (MAb against CAM alpha 4 integrin)

Symptomatics - spastic, bladder, pain, fatigue, depression, constipation, sexual


dysfunction
.spastic: baclofen (GABA(+)), botox (synaptobrevin(-)
.bladder incontinence: oxybutynin M(-)
.pain: TCA, cabamazepine, gabapentin(voltage gated calcium ion channels (+))
.fatigue: amantadine, modafinil, methylphenidate (amphetamine)
.depression: ssri
.constipation: fibre, coloxy senna
.sexual dysfunction: sildenafil\
Education & counselling
_______________________________________

_______________________________________
PC:
sudden onset scrotal pain and swelling
radiation along cord to flank
scrotal erythema, tender
fever
frequency, urgency, dribbling
relieved by elevating testicles (positive prehn sign)
cremaster reflex present

---
1o - epididymo-orchitis
---
OE:
eti: Gonorrhea, chlaymida, mumps, TB, syphilis,
---
ix:
UA, urinc C/S
urethral d/c - culture
colour flow doppler US (rule out torsion)
EUA in OR
---
mx:
ceftriaxone + doxycycline

Gonorrhea/chlaymydia = ceftriaxone + azithromycin


.urethral swab, urine PCR
_______________________________________
PC:
-const (fatigue, anorexia, night sweats, weight loss)
-chronic, non tender LNpathy
-hx of immigration / travel

---
1o - secondary/reactivation TB
---
ddx:

I - histoplasmosis, blastomycosis, nocardia

histoplasmosis: immunecompro, LNpathy, pancytopenia, HSmegaly, urine antigen


testing, hilar adenopathy, mississipi river

Blastomyces: immunocompro, skin lesion, osteolytic, prostate, mississipi river


---
OE:
RF - low ses, etoh, immunesuppression/HIV

resp - subacute fever, cough, weight loss, apical pulmonary dz, night sweats

lymphadenitis (chronic, non tender)

meningitis, pelurisy, heaptitis, pericardis, osteomyelitis

Adrenal/addisons

renal, ovarian

---
ix:
PPD of IFN-gamma assay screen:
- if >5mm treat those that are
.HIV
.recent contact with TB
.CXR evidence of prev TB
.organ transp
- if > 10 mm
.recent immigrant from TB endemic
.IVDU
.hi risk (prison, nursing home, hospital homeless)
.mycobacteriology lab personnel
.hi reactivation risk (DM, steroids, leukemia, ESRD, malabsorption)
-if >15mm
.healthy individuals

gram stain acid-fast/ aurium (excludes nocardia)


cxr - upper lobe, cavitary

Urine/serum Histoplasma antigen assay

---
mx:
Pulm TBN = RIPE x 2 mo then RIP x 4 mo
extra pulm (...itis's) = RIPE + steroids x 12 mo

RIPE: rifampicin, isoniazid, pyrazinamide, ethambutol

Latent TB:
iso + rifapentine 1 per wk x 3 mo with direct observation

MED SE:
Isoniazid peripheral neuropathy (B6 def)
Isoniazid hepatitis (elevated ALT)
Isoniazid - drug induced SLE
_______________________________________
PC:
hx CHF, SOB, coughing up pink frothy sputum

---
1o - pulmonary edema
---
ddx:

rule out -
4Hs = hypo VOGET (volume, oxygen, glucose, electrolytes, temperature)
4Ts = toxin, trauma, tamponade (cardiac), tension (pneumothorax), thromboembolus,
tearing (dissection), thrombosis (MI)

---
OE:
precipitations of CHF = CAD, htn, valvular, etoh
HFREF = systolic dysfn = ejection fraction < 35% = apex displaced, S3,
cardiomegalty, decr LV-EF, ORTHOPNEOA***, MR, assoc with CAD, MI, htn, DM, etoh,
myocarditis, dilated cardiomyopathy,
-cough, crackles,
HFPEF = diastolic dysfn = htn, normal apex, S4, normal size hear, LVH on echo/ecg,
normal EF,
-peripheral edema, JVP, hepatomegaly, pulsatile liver S4,
NYHA: 1. no sx w normal activity; 2. sx on activity; 3. reduced fn; 4. at rest
---
ix:
CBC, UEC+CMP, BUN/Cr, fasting BSL, HbA1c, lipids, LFT, serum TSH, ferritin, BNP***,
uric acid
ECG: arrhythmia, hypertrophy
CXR - HERB-B = heart enlarge, effusion, re-distributed edema, B-lines kerly,
bronchiolar-laveolar cuffing
Echo: ventricular dysfunction
---
acute mx:
1o LMNOPP
lasix (furosemide 500 mg IV)
morphine 4 mg (decr preload)
nitroglycerin - GTN patch
oxygen - if hypoxic
Positive airway pressure - CPAP/BiPAP (decr preload)
Position - sitting up with legs hanging down
2o
nitroprusside
hydralazine
SANS(+) =
.dopamine (lo-med = D1-B1 = renal vasodil - inotropic)
.dobutamine B1(+)
.phosphodiesterase inhib (milrinone) = SM relax = decr SVR
3o mechanical vent

long term mx =
1o - mx HFREF(EF<35) = (mortality benefit with BB, spironolactone)
.MDT
.Lifestyle = diet, exercise, DM, smoking, decr etoh, education, restrict
fluids / salt
.ACEI (ARB) (slows LV dysfn)
.BB (if LVEF < 40%)
.spironolactone (eplerenone)
.furusemide 40 - 500 mg daily = for fluid overload and to prevent the
hyperkalemia of BB/ACEI/ARBs/spiro
.digoxin if in sinus but still symptomatic on ACEI, or AFib
.amiodarone if CHF + arrhythmia
.warfarin to prevent VTE

_______________________________________
PC:
immobilised patient + hx cancer
dyspnoea, pleuritic chest pain
tachy
haemoptysis
cyanosis
hypoxia
fever

---
1o - pulmonary embolism
---
rule out -
4Hs = hypo VOGET (volume, oxygen, glucose, electrolytes, temperature)
4Ts = toxin, trauma, tamponade (cardiac), tension (pneumothorax), thromboembolus,
tearing (dissection), thrombosis (MI)
---
OE:
immobilised patient + hx cancer
dyspnoea, pleuritic chest pain
tachy
haemoptysis
cyanosis
hypoxia
fever
---
ix:
wells score

if Wells = Lo (0-2)
1o - D-dimer (neg = ruled out) - but if pos do CTPA
2o - CTPA (pos = ruled in)

if Wells > Lo
1o - CTPA

Dx/Sp = CT-PA (pulm angiogram is gold standard)


ECG = sinus tachy + RV strain + S1-Q3-T3inv
CXR = atelecatsis/collapse, effusions
V/Q scan (hi Sn) = do it if there is no COPD and CXR normal or contr to CT
Venous dopplers
baseline INR
---
mx:
1o - anticoagulation 6 mo if unprovoked (3mo if transient)- LMWH eg enoxaparin 1.5
mg/kg SC q24hr + warfarin started at same time 5mf PO OD + baseline INR; IVC filter
if anticoag contraindicated
2o - thrombolysis if > moderate PE (eg massive) or compromise/shock
3o - admit if unstable, requiring O2, comorbiditis, lack social supports, unable to
ambulate

Wells criterion:
clinical DVT
immobilisation or surgery within 1 mo
previous PE/DVT
HR > 100
hemoptysis
malignancy

PERC = PE rule out criteria = rules out if all met


<50yo
HR<100
O2sat>95
no hemoptysis
no E2
no prev DVT/PE
no unilateral leg swell
No surg/trauma req hospital w/in last 4 wk
_______________________________________
PC:
b/g > 20 pack year smoking
normally = wheeze, productive cough
last PFT = irreversible with bronchodilator, lo FEV/FVC
meds - salbutamol and prednisone
acute SOBOE, incr productive, desat, tachnypnoic, cyanotic

---
1o - acute exacerbation of COPD
---
ddx:

V - MI, PE
I - URTI, pneumonia
N -
D -
I -
C - MI, CHF
A -
T -
E -
M -
S -
---
OE:
acute exacerbation =
>48hr
worse dyspnoea / cough/ sputum production leading incr use medications
purulent or non-purulent

chronic COPD = GOLD classification for severity COPD


1. fev > 90%
2. fev1 bw 50-80
3. severe = fev1 30-50
4. v severe = fev1<30

---
ix:

---
mx:

mx acute COPD
ABCs
O2 target 88-92 for CO2 retainers
bronchodilators SABA + M(-) ipratropium bromide x 3 back to back every 15 min
systemic corticosteroids (IV methyl prednisone or oral prednisone)
abx if (incr sputum + purulence)
-simple exacerbation = amoxicillin (or ceftr)
-complex exacerb = 1 of (FEV1<50 , exacerv > 4/yr, IHD, home O2 use,
chronic oral steroid use) = amoxicillin/clavulanate ie fluoroquinolone/B-lactam +
B-lactamase inhibitor
consider ICU admission IF cannot maintain O2sats, LOC, low GCS and
ventilation required
follow up with exercise rehab

mx chronic COPD
1o (these all prolong survival) =
smoking cessation (NRT, bupropion, varenicline),
vax (influenza, pneumococcal),
home O2 (>15 h/d) indic if paO2<55 or <60+corpulmonale

2o (for sx only) =
ICS + LABA (Fluticasone + salmeterol)
Bronchdilators = M(-) ipratropim bromide + B(+) SABA

3o Nurse educator = education, avoid irritants,


PT + OT = exercise rehab improve endurance

4o Review past admission notes + contact GP + review medications

prognosis COPD = BODE index


BMI < 21 = 1pt
Obstructive FEV1 50-64 = 1pt, 36-49 = 2, <35 = 3pt
Dyspnoea : walks slow + stops = 1, stops = 2, too breathless at rest for adls
= 3
Exercise capacity (6 min walk) = 250-349 = 1pt, 150-249 = 2pt, < 149m = 3pt

_______________________________________
PC:

---
1o - cirrhosis
---
ddx:

constrictive pericarditis
budd-chiari syndrome
portal vein thrombosis
schistosomiasis
sarcoidosis
---
OE:
bruises
hepatic flap
caput medusa
esophageal bleed
ascities
juandice
encephalophaty/coma
xanthelasma
scleral icertus
fetor hepaticus
spider angioma
gynceomastia
muscle wasting
ankel edema
palmar erythema
dupuyretns contracture, asterixis anemia
clubbing, luekonychia
splenomegaly
hemorrhoids
spontaenous bacterial peritonitis
ATN, hepatorenal syndrome
---
ix:
1o biopsy
2o platelet decr, INR incr, albumin decr, bilirubin incr, glucose decr
3o USS***
CT (varices, nodular liver, splenomegaly, ascites)
Ultrasound elastography
4o liver transplant if Childs PUgh score >6/B

Child Pugh score = 1/2 yr mortality


bilirubin, albumin, INR, ascites, encephalotphy
---
mx:
cease etoh, hepatotoxic (paracetamol/NSAID)
HAV/HBV vax
monitor complications (varices, ascites, HCC)
HCC: ALP, bilirubin, AFP, triphasic CT, USS
tx = surgical, chemo, RF ablation, ethanol injection
_______________________________________
PC:
Right flank, RUQ pain
Jaundice, ascites, caput medusa, esophageal bleed
CHF

---
1o - hepatitis, HCV
---

---
ix:
1o serum HCV-RNA (also correlates to disease activity)
2o HALT = elevated ALT more then AST
3o (+) serum anti-HCV IgG
4o HAV and HBV serology
5o liver biopsy (fibrosis, cirrhosis, necrosis, inflamm)
---
mx:
Avoid etoh
vax HVA, HVB
sofosbuvir/ledipasvir (interferon free regimen
alternative = IFN + ribavirin + protease inhibitor
monitor for HCC, cyroglobulinemia (leading to membrano proliferative GN), non
hodgkin lymphoma

_______________________________________
PC:
depression, insomnia, craving, myalgia, nausea, chil, autonomic instability
(lacrimation, rhinorrhea, piloerection)

---
1o - opiate withdrawal
---
---
OE:

---
ix:
UA drug screen
---
mx:

opiate toxicity
(shallow resp, miosis, brady, hypothermia, LOC)
ABC
IV glucose
Dx = naloxone 2 mg IV
Mx - intubation/ventilation, nalozone IV
opiate withdrawal
methadone (long acting opioide)
clonidine
opiate rehab
methadone or buprenorphine
suboxone (PO naloxone + buprenorphine = naloxone has low bioavailability but
when injected will cuase withdrawal)

Cocaine intox/OD = htn, tachy, tonic/clonic seizure, dyspniea, vT/VF,


elation/euphoria, pressured speech, restless, mydriasis, sweating
diazepam IV (seizures)
(contra BB bc unopposed alpha)
Cocain withdrawal = incr sleep + apetite, fysphoria, fatigue, irritable, bad
dreams, psychomotor agitaiton
supportive
_______________________________________
PC:
IVDU
sepsis, murmur, embolic infarcts

---
1o - infective endocarditis
---
ddx:
---
OE:

"From Jane"
fever, roth, osler, mumur, janeway lesions, aneamia, nailbed/splinter hemorrhage,
emboli

insp - fever, wt loss, pallor/anaemia, fatigue, petechiae


hands - spinterhemorrhage, clubbing, painful oslers nodes finger pulp, janeway
lesions
arms - injections sites
eye - pale/anemia, retinal/conunctival spot - roth
heart -
dyspnea, chest pain, clubbing
murmur - regurg (MR, AR)
tricuspid = IVDU, (non IVDU = MV >> AV > TV > PV)
signs of CHF (edema, pulm edema)
abd - splenomegaly
neuro - focal neuro bc embolic stroke
UA - heamturia, GN (immunde complex)

pathogen:
native = strept/viridans, saureus, enterococcus
ivdu = saures, streptococcus/viridan
prosthetic = saurea, epidermidis
---
ix:
DUKE criterion = 2 Mj or 1 Mj + 3 mn or 5 Mn
Mj
.BC x 2 sites
.(+) echo for valvular mass and regurg
Mn
.predisposition - congenital/IVDU
.fever
.vascular sx - arterial emboli, pulm infarcts, mycotic aneurysm, ICH,
conjunctival hemorrhag, janeway lesion
.immune - GN, RF, oslers, roth
.(+) BC no meeting major criterion

3 sets BC (1 anaerobic, 1 aerobic from different sites 1 hour apart)


CBC, differential (normochromic, normocytic aneamia)
ESR incr
RF(+)
BUN/Cr incr
UA - protein, hematurai, RBC casts, urine C+S
ECG - prolonged PR (perivalvular abscess)
Echo (TEE best) - vegetations, regurg, abscess

---
mx:
1o take BCs
2o empiric
native. vanc + gent/ceftriazone
prosthetic. vanc + gent + cefepime + rifampin
3o monitor complications
CHF
block
pulm embolus
stroke
abx toxic - nephritis
_______________________________________

AEIOU
A - anticipated problems
E - psychoscoial
I - prevent further injury
O - optimise care
U - treat underlying cause

AFlutter:
- acute unstable : electrical cardioversion
- acute stable :
. rate - BB, CCB (diltiazem, varapil), digoxin
. chemical cardioversion - sotalol, amiodarone, type 1 (flecainide)
- long term:
. sotalol, amiodarone
. RF ablation

MAT:
- underlying cause (COPD, hypoxemia, sepsis, theophylline, digitalis)
- CCB
(contra - BB in COPD, cardioversion, antiarrhythmics, ablation)

AFib - mx: RACE


- rate: BB, CCB, digoxin/amiodarone
- anticoag: warfarin or DOACs
- cardioversion w/o anticoag if <24, w anticoag if >24 + 4 wk after
- eti - "Atrial Fib"
Alcohol Abuse
Thyroid Disease
Rheumatic Heart Disease
Ischemic Heart Disease
Atrial Myxoma
Lung (Pulmonary Embolism, COPD)
Pheochromocytoma
Idiopathic
Blood Pressure (Hypertension)

AVNRT:
acute
1. valsalva/carotid + metoprolol, digoxin, diltiazem, EC version
2. adenosine
long:
1. BB, diltiazem , digoxin
2. type 1c (flecainide, propafenone)
3. ablation

AVRT:
acute: valsalva/carotid + metoprolol, EC version
long: flecainide, procainamide, ablation

Torsade:
IV Mg
pacing
isoproterenol
EC version

Stable angina: AEIOU


Injury = Prevent further injury:
- reduce MyoC O2 demand
- incr O2 supply
- reduce RF (life-style, diet, exercise, statins, hypertension)
- prevent plaque rupture and thrombosis
Optimise current therapy:
1st. ASA or clopidogrel
BB (improves survival) - B1 selective metoprolol, atenolol
2nd. CCB
Nitrates
HTN/DM - ACEI

PRINZTMETAL vasospasm angina


- nitrates + CCB

NSTEMI
acute
- BEMOAN (+1 and 8 hr - telemetry+troponin-I)
betablocker
enoxaparin (LMWH) + clopidogrel
morphine
o2
nitrates sublingual then IV
aspirinv
- clopidogrel + LMWH
- stress test (exercise or dipyradamole/adenosine + radiolabelled sestamibi)
(+) or Trop (+) or pain or LVEF <40%
- abciximab + CTCA + revascularisation

STEMI
acute
- MONA BasH (+1 and 8 hr - telemetry+troponin-I)
morphine
o2
nitrates sublingual then IV
aspirin
BB metoprolol
Heparin

- PCI w/i 4 hr pain OR anytime if unstable = abciximab + CTCA + PCI


- no PCI w/i 4hr of pain = thrombolysis tPa, then stress testing
. if stress test positive then => ticagrelor + CTCA + coronary
revascularisation

Anticipate: MI, ecg, echo

long
1. Educate + RF modification (PT, exercise, diet)
2. dual antiplatelet = ASA + ticagrelor (ADP blocker)
3. BB (met/atenolol) or in context of COPD use CCB
4. Nitrates (Sx)
5. ACEI
6. Aldosterone antagonist IF LVEF < 40 and CMF or DM - Eplerenone
7. monitor for complications MI CRASH PAD (MI, cardiac rupture, arrhythmia,
shock, htn/heart failure, pericarditis/pulmonary emboli, aneurysm, DVT

PE= LMNOP
-lasix
-morphine
-nitrates
-oxygen
-position + CPAPA

CHF
- O2, bed rest, elevate head
- lifestyle - diet, exercise, DM, smoking, etoh, education, salt/fluid
restriction
- MDT
- cardiac rehab

ABCDE (acei, bb, anticoag, antiarrhythmics, diuretic, aldosterone receptor


inhibitor, inotrope)
- ACEI (HFREF, post MI, NYHA>1)
- BB (esp cardelilol)
- Diuretic (furosemide + metolazone)
- Digoxin, amiodarone (anti-arhtyhmics)
- Eplerenone or spironolactone

Dilated cardiomyopathy - tx for CHF

HOCM:
AAAABCDD
A. - avoid incr obstruction - hypovolaemia
A. - avoid competitive sport
A - alpha agonist - phenylepphrine for shcok
A - ablation, myectomy, dual chamber pacing
B - BB
CCB - verapamil
D - disopyramide
D - defib - ICD***
D - dual chamber pacing
BESTS - ICD***

restrictive cardiomyopathy
- investigate infiltrative (amyloid - paraproteins)
- investigate non-infiltrative (scleroderma scl-70)
- investigate storage dz (haemachromatosis, Fabry diz, gaucher, glycogen
storage)
- CXR
- ECG
- echo
- CT
- biopsy
- Mx underlying
- Mx CMF + arrhythmias

Effusion/tamponade
- hypotension, pulsus paradoxus, muffled heart sounds, JVP
- ECG
- echo
- cardiocentesis + echo

Constrictive pericarditis (coxsackie, TB, radiation, collagen vascular disease)


- ecg (lo, flat T, Afib)
- CXR (calcification, effusion)
- echo/CT
- diuretic + salt restriction
-----------------------------------------------------------
PC: female, noticed hard, non tender lump on breast. Mother died of breast cancer,
sister has ovarian cancer.

Dx = Breast Cancer

RF =
incr risk - (+)hx, 1st degree, BRCA1/2, chest wall radiation
RF - hi breast density, nulliparity, 1st pregnancy >30, menarche <12, menopause
>55, radiation, >5yr HRT, >10yr OCP, BRCA 1/2, alcohol, obesity, sedentary

Ix:
- mammogram (calcification, distortion, changes)
- USS cystic vs solid
- MRI
- galactogram
- mets workup (bone scan, abd USS, CXR, CT head
- Dx - FNA or core needle biopsy or excisional
- Genetic BRCA1/2 for <35 yr, bilateral < 50 yr
- receptor biology (ER, PR, Her2)

Mx:
- DCIS - lumpectomy + RadTx (ductal epithelial contained with breast ducts)
- LCIS -
- Invasive ductal carcinoma

Stage 0 = insitu
1 = <2 cm, LN0, M0
2 = LN+, M0
3 = invasion, AxN
4 = mets

(mastectomy is an alternative to conservative sx)

St 0/insitu = conserving sx + RTx + tamoxifen ER + trastuzumab HER2


St 1-2 = conserving + AxN + RTz +/- chemo/tamoxifen
St 3-4 = mastectomy + AxN + RTx + Chemo

F/U
-q 3mo exam for 2 yr then annual
-q6 mammography if conservtive sx
-gync surveillance endometrial cancer if tamoxifen used
- breast reconstruction
- psychosocial support
- mets surveillance Bones>lungs?pleura> liver>brain....palliate w hromone therapy,
chemoherapy, radiation
---
CML:
PC - systemic, splenic (early fullness, LUQ, shoulder tip pain, splenomegaly,
anemia, bleeding platellet dysfn, pruritus, PUD, leukostasis, priapism,
encephalopathy)
Sx - allopurinol, antihistamines
chronic - imatinib mesylate (2ndgen dasatinib, nilotinib) + INF + hydroxyurea
Curative ? HSCT
---
Prostate cancer screening:
SCREENING IS NOT RECOMMENDED
hi risk:
- Men with one or more first-degree relatives diagnosed <65 years of age
- Men with a first-degree relative with familial breast cancer (BRCA1 or
BRCA2)
screening with PSA > 4 ng/L - SCREENING IS NOT RECOMMENDED
-not routine if asymptomatic - any age
-only on demand (risk of overdiagnosis, erectile dysfn, incontinence,
suicide, CVD)
DRE is not recommended as it is not sensitive
---
COlorectal cancer screening:

AVERAGE RISK
asymptomatic or
1 first or second degree relative diagnosed > 55 yo
= FOBT (fecal immunochemical test)from 50 yo until 75 yo - every 2 yr
.not colonoscopy or DRE

MODERATE RISK
asymptomatic with 1 first degree < 55 yr or
2 first degree on same family side, or
1 first + 1 second degree on same family side
= Colonoscopy (or sigmoidoscopy) q5 yr from 50yo, or 10 yr younger than first
diagnosed in family
PLUS
FOBT in intervening years

HIGH RISK
Asymptomatic PLUS 3+ first/second on same side dx with CRC (hereditary non-
polyposis, ovarian cancer), or
2+ first/second on same side with CRC PLUS multiple CRC in one person, CRC aged<50
yo, lynch syndrome cancer, or
1+ first/second with CRC + many adenomas (FAP), or
Fhx(+) hi risk mutation APC or MSH gene
= genetic testing + colonscopy
FAP = surveillance from age 12
Lynch = surveillance from age 25
Aspirin prophylaxis

---
Breast cancer screening guidelines:

AVERAGE:
1st degree > 50 yo, or
1+ 1st/2nd > 50yo on different side
= mammogram + breast awareness, q2 yr, age 50-74

MODERATE:
1st degree < 50 yo, or
2+ 1st degree same side
2+ 2nd degree same side, 1+ < 50 yo
= mammogram + breast awareness, q1 yr, age 50-74 + referral to cancer clinic

HIGH RISK:
2+ 1st/2nd, same side - breast/ovarian - PLUS they were dx < 40 yo, bilateral,
breast + ovarian in same woman, ashkenazi male relative
1+ 1st/dend dx < 45 yo PLUS another dx sarcoma < 45 yo
Mutation
=individualised - MRI + USS + breast exam

Potrebbero piacerti anche